Vous êtes sur la page 1sur 61

Articles

They are probably the simplest words. They are probably Correct : There are hundred centimetres in a metre.
the most used words. A, an and the. These three words are In the vague sense of a certain.
known as articles. It is difficult to master these (particularly I saw a book on the table.
the) without sufficient practice. Let us see what we can do in It means I do not know exactly which book I saw.
the few pages that we have. To make a common noun of a proper noun.
When NT Ramarao died, the media said of Lakshmi
Indefinite Article Parvati: “Will she do a Jayalalitha?
A or an is called the indefinite article because it leaves If you see Jayalalitha as a proper noun, it makes no
indefinite the person or thing spoken of. For example, sense. The writer here is not concerned with Jayalalitha the
There shall come a day when everyone will be rich. person but with the traits she carries. Jayalalitha thus becomes
The writer of the above sentence is indefinite about the day. a common noun: characteristics particular of Jayalalitha.
He cannot specify a particular day. It could be any day. In such conversions a (or an) is used.
In the sense of each, every or per.

K
The difference between a and an is only phonetic
(of sound). Both of them serve the same function. The doctor said “Take the pill twice a day.”

Rule: An is used before a word beginning with a vowel In the sense of any, to single out an individual as the
sound. representative of a class.
Incorrect : The English were a enemy of the French. A computer saves time.
The word enemy begins with a vowel (e) sound. So, use an. That is, saving time is the characteristic of any and every
Correct : The English were an enemy of the French. computer.
Note : There are five vowels in the English alphabet — a, e, i, Rule: When two subjects are thought of as a single unit,
o, u. the rest are consonants. article is used before the first subject.
Incorrect : He is a MP. Incorrect: He introduced me to his friend who is a singer

KUNDAN
MP stands for Member of Parliament. But when you read the and a scientist.
above sentence, MP is read as empee. So the word begins Correct: He introduced me to his friend who is a singer
with a vowel (e) sound. The correct indefinite article is and scientist.
therefore an. The same person is a singer as well as a scientist. So
Correct : He is an MP. article is used before the first subject.
Rule: A is used before a word beginning with a consonant
sound.
Incorrect : An girl is waiting for me. Use of the definite article
Girl begins with a consonant (g) sound, So, it should be When we speak of a particular person or thing, or one
preceded by a. already referred to.
Correct : A girl is waiting for me. Incorrect : He has been causing trouble since a day he came.
Incorrect : She is an university student. See how things become clear if we split the given sentence
You are tempted to use an when you see the u in university. into two parts.
But pronounce it and it is yuniversity. That is, it begins with (1) He came one day.
a y sound. y is a consonant. So, (2) He has been causing trouble since then.
Correct : She is a university student. Now, what would you do if you have to use the noun day
instead of then in (2)? Which article would come before day?
Use of the indefinite article It has to be the definite article the because the day has already
been referred to in (1). So, after substitution we have
In its original numerical sense of one. (2) He has been causing trouble since the (that) day.
Incorrect : There are hundred centimetres in the metre. Combining (1) and (2).
If you know the metric system well, 1m = 100 cm. Thus, there Correct : He has been causing trouble since the day he came.
are hundred centimetres in one metre. If we have to substitute When a singular noun is meant to represent a whole class.
one with an article, it is a, not the. So, The elephant is a huge animal.
Exceptions: Man and woman are not preceded by an article Omission of the article
when used in the general sense.
Before the names of physical features in geography. We are so much in the habit of using the ( or even a/an) that
The Gulf of Cambay; the Volga (river); the Arabian Sea; it becomes imperative to know where not to use them.
the Indian Ocean,
Before a common noun used in its widest sense.
the West Indies (group of islands); the Himalayas
Incorrect : The science has developed much in the past
(mountain range).
hundred years.
Before the names of religious and mythological books.
The above sentence would be correct if a particular science
Incorrect : The story is there in Bible.
were being talked about. Since this is not mentioned in any
Correct : The story is there in the Bible.
form, we assume science to be used in its general sense. So,
However, when these books are preceded by the name of
the article the should not be used.
their authors, the is not used. So, you might have read the
Correct : Science has developed much in the past
Aeneid, but you cannot read Virgil’s the Aeneid. It is Virgil’s
hundred years.
Aeneid.
Before the noun following kind of.
Before common nouns which are names of things unique of
Incorrect : What kind of a hobby is this?
their kind.
When you talk of a particular thing, it is a kind of general
The earth goes round the sun.
thing (a common noun in its widest sense). Here hobby has

K
Before a common noun to give it the meaning of an abstract been used in this general sense. By the previous rule, there
noun. should be no article before it. So,
The devil in him begins its misdeeds now and then. Correct : What kind of hobby is this?
Here the devil becomes an abstract noun which means the Note : The above sentence is different from the exclamatory
devilness or the devilish spirit. “What a hobby!” You exclaim thus about one hobby, so it is
Before superlatives. correct to use a.
Incorrect : Kumble is best bowler in the country. Before names of materials.
The moment you use the superlative, you are pointing to a Incorrect : An iron is a useful metal.
particular person or thing. When you use best for someone Here we are talking of iron the material. So, it is incorrect to
you are specific about him. So, the definite article is used. use an article.

KUNDAN
Correct : Kumble is the best bowler in the country. Correct : Iron is a useful metal.
Before ordinals. But it is correct to say
Incorrect : They won the match in fortieth over. An iron is a useful gadget.
Fortieth (40th) is an ordinal because it tells you which (in because here we are not talking of the material iron but the
terms of number) over. By thus singling out this over, it is object which is used to make clothes smooth.
being specified. Hence, the should be used. Before proper nouns.
Correct : They won the match in the fortieth over. India is a big country.
Before Comparatives. Before abstract nouns used in a general sense.
Incorrect: He is a better of the two boys. Incorrect : The strength is what makes a man successful.
Correct: He is the better of the two boys. Which strength? Any particular one? No. So, do not use the.
Correct : Strength is what makes a man successful.
Note: If “than” is used to express comparative degree, the But it is correct to say
article “the” is not used with comparative degree. The strength of a team lies in its ability to maintain
harmony. The adjective phrase thus makes strength particular.
Incorrect: He is the better than the two boys.
Correct: He is better than the two boys. Before names of one’s relations.
Mother would like to see you.
Before musical instruments.
The writer of the above sentence is telling the reader that his
Amjad Ali Khan plays the sarod.
mother would like to see him (the reader). But if someone
Before an adjective when it is understood a noun. else’s mother is being talked about, he should write.
Let us enter the world of the bold and the beautiful. The mother would like to see you.
Here the adjectives actually contain the noun persons or
people within them. Articles are not used with these common expressions if they
are used for primary purposes. For example,
(1) to go to school/college/church/home/market/bed Incorrect: I will meet you at the dusk.
court/hospital/prison. Correct: I will meet you at dusk.
(2) At home, at fault, at down, at night, at noon, at church,
at hand, all day, all right. Articles are not used with certain idiomatic phrases
(3) By road/land/air/ train/sea/bus/ship. like — in danger, in demand, on fire, by mistake, in problem,
Incorrect: I went to school to drop my sister. in difficulty, by bus, in hope, in detail etc.
Correct: I went to the school to drop my sister. Incorrect: The house was on a fire.
Here school is not used for primary purpose that to Correct: The house was on fire.
study there. Dropping someone to school becomes secondary
No article is used before most proper nouns (names of
purpose. So the should be used before school.
persons, countries, cities, continents).
No article is used after such verbs as appoint, elect, crown,
Correct: My friend will come from India.
make.
Correct: My friend will come from the USA.
Incorrect : They elected him a chairman
Note: Certain countries like USA, US, UAE take the article
Correct : They elected him chairman.
“the” before them.
No article is used before sunset, sunrise, daybreak,
dawn, dusk, night, noon, today, yesterday.

K
EXERCISE

Directions: Read each sentence to find out whether there 13. The robbers were (a)/ caught just as they (b)/ were about
is any error in it. The error, if any, will be in one part of the to escape from the jail. (c)/ No error (d)
sentence. The number of that part is the answer. If there is 14. The man is mortal (a)/ and ultimately he has to leave
no error, tick “No error” as your answer. (Ignore the errors everything here (b)/ yet how irrationally he remains
of punctuation, if any) attached to his worldly possessions. (c)/ No error (d)
1. It is not my business (a)/ to give an advice to those who 15. In a mountaineering expedition (a)/ higher you (b)/ go
are not sensible enough (b)/ to deal with their own from the base camp the colder you feel. (c)/ No error (d)
problems. (c)/ No error (d) 16. He leads a very busy life (a)/ so he goes everywhere (b)/

KUNDAN
2. You have been learning Tamil (a)/ for last one year (b)/ but by a car. (c)/ No error (d)
you show no improvement whatsoever. (c)/ No error (d) 17. We should spend a good part of the day (a)/ in outdoor
3. The English (a)/ defeated (b)/ French in the battle of activities and games (b)/ because staying indoors all the
Waterloo. (c)/ No error (d) time is bad for health. (c)/ No error (d)
4. A first step (a)/ in a rational solution to any problem (b)/ 18. The trees in a forest must be properly counted and
is the recognition that a problem exists. (c)/ No error (d) numbered (a)/ and proper entries be made (b)/ in the
5. I am not wealthy (a)/ so I cannot afford (b)/ to buy a register. (c)/ No error (d)
expensive car. (c)/ No error (d) 19. The principal asked the girls to return (a)/ to the hostel
6. As soon as the teacher entered (a)/ everyone in the class (b)/ before the sunset. (c)/ No error (d)
(b)/ fell in a silence. (c)/ No error (d) 20. He is the best artist of his time (a)/ but unfortunately (b)/
7. On my request (a)/ Lalit introduced me to his friend (b)/ least recognised. (c)/ No error (d)
who is singer and a scientist. (c)/ No error (d) 21. Brahmaputra is (a)/ one of the largest rivers (b)/ that
8. To perform this experiment (a)/ drop little sugar (b)/ into originate in the Himalayas. (c)/ No error (d)
a glass of water and see the change. (c)/ No error (d) 22. In a hour’s time (a)/ when I had finished the work, (b)/ I
9. Even now when I (a)/ see the spot (b)/ I am reminded of got up and slipped out unnoticed. (c)/ No error (d)
an unique accident that took place several years ago. 23. He was fascinated by insects (a)/ and the more he studied
(c)/ No error (d) their habits (b)/ greater was his fascinations. (c)/ No
10. With little imagination and enterprise (a)/ the tournament error (d)
could have been transformed (b)/ into a major attraction. 24. We all (a)/ must respect (b)/ a honest and hardworking
(c)/ No error (d) person. (c)/ No error (d)
11. Page after page (a)/ of Gita were read (b)/ and it gave 25. By all standards (a)/ he is a best soldier (b)/ our military
great consolation to his mind. (c)/ No error (d) school has produced so far. (c)/ No error (d)
12. One should (a)/ be true to one’s promises in order to 26. He did not go (a)/ to the city on foot (b)/ he went there.
earn (b)/ name & fame in the life. (c)/ No error (d) by the train. (c)/ No error (d)
27. A person I met (a)/ in the theatre yesterday (b)/ was the 32. I don’t like (a)/ that type of a man (b)/ who does nothing
playwright himself. (c)/ No error (d) but find out faults in others. (c)/ No error (d)
28. Both the civilians (a)/ and armymen (b)/ joined the First 33. Little knowledge (a)/ of music that he possessed proved
World War. (c)/ No error (d) to be a boon for him (b)/ when he was utterly helpless.
29. My friends insisted (a)/ that I should see (b)/ the movie (c)/ No error (d)
from beginning to the end. (c)/ No error (d) 34. It was (a)/ by a mistake (b)/ that he caught her hand (c)/
30. He took to (a)/ reading Times (b)/ for better knowledge No error (d)
of the facts. (c)/ No error (d) 35. After returning (a)/ from the jail, he has abandoned (b)/
31. All the staffs (a)/ of this office hold the view (b)/ that Mr. all the criminal activities and is determined to lead a simple
Sinha is a very wise and an intelligent officer. (c)/ No life. (c)/ No error (d)
error (d)

Answers and Explanations


1. b; Omit the article “an” before “advice” as “advice” is 17. d; No error.
singular uncountable noun. 18. d; No error.
2. b; Put “the” before “last” is an ordinal. 19. c; Omit “the” before “sunset”.

K
3. c; Put “the” before “French”. 20. d; Put “the” before “least” as it is a superlative.
4. a; Replace ‘a’ with “the” as the word “first” is an 21. a; Put “the” before “Brahmaputra”
ordinal. 22. a; Replace ‘a’ with “an”.
5. c; Replace ‘a’ with “an”. 23. c; Put “the” before “greater”.
6. c; Omit ‘a’ before “silence” as it is uncountable noun. 24. c; Replace ‘a’ with “an”.
7. c; It should be “a singer and scientist”. 25. b; Replace ‘a’ with “the”.
8. b; Put ‘a’ before “little”. 26. c; Omit “the” before “train”.
9. c; Replace “an” with “a”. 27. a; Replace “a person” with “the person”.
10. a; Put “a” before “little”. 28. b; Put “the” before “armymen”.
11. b; Put “the” before “Gita” as it is the name of a 29. c; Put “the” before “beginning”.
30. b; Put “the” before “Times”.

KUNDAN
religious text.
12. c; Omit “the” before “life”. 31. c; Omit “an” before “intelligent”
13. c; Omit “the” before “jail” as it has been used for primary 32. b; Omit “a” before “man”.
purpose. 33. a; Put “the” before “little”.
14. a; Omit “the” before “man”. 34. b; Omit “a” before “mistake”.
15. b; Put “the” before “higher” as it is a comparative. 35. b; Omit “the” before “jail”.
16. c; Omit “a” before “car”.

Preposition
A preposition is a word placed before a noun (or a pronoun) about, above, across, along, amidst, among, around,
to show in what relation the person or thing denoted by it before, behind, below, beneath, beside, between,
stands to something else. beyond, inside, outside, underneath, within, without.
3) Phrase Prepositions : When a group of words is used
Kinds of Preposition with the force of a single preposition, it is known as a
phrase preposition. For example,
1) Simple Prepositions : They are the simple words used He succeeded in spite of obstacles.
as prepositions. For example, at, by, for, from, in, of, off, 4) Participial Prepositions: When the present participle
on, out, over, through, till, to, under, up, with. of a verb acts as a preposition, it is known as a participle
2) Compound Prepositions : They are generally formed by preposition. For example,
prefixing a preposition (usually a = on or be = by) to a There have not been too many communal riots in recent
noun, an adjective or an adverb. For example, times barring those in the aftermath of Dec 6.
Some Rules Correct: There were few incidents of irregularity during
the Emergency years.
Rule : In is used with names of countries and large towns; Rule: The preposition “off” denotes “separation”, “at a
at is more often used when speaking of small towns distance from” or “far from” whereas the
and villages. preposition “of” denotes cause, origin, quality,
Incorrect: He lives at Germany. possession.
Correct: He lives in Germany. Incorrect: He put of his coat.
Rule : In and at are used in speaking of things at rest; to Correct: He put off his coat. (Separation)
and into are used in speaking of things in motion. Incorrect: He died off cancer.
Incorrect : This week the song jumped at the top of He died of cancer. (Cause)
the charts. Rule : The word compare is followed by to when it shows
Correct: This week the song jumped to the top of that two things are alike. It is followed by with when
the charts. we look at the ways in which two things are like
Rule : Till is used of time and to is used of place. and unlike each other.
Incorrect: The office will remain open to six in the Incorrect: Sanath Jayasuriya's batting may be
evening. compared with the sales of a useful book:
Correct: The office will remain open till six in the they score fast right from the beginning.
evening. Correct: Sanath Jayasuriya's batting may be

K
Rule : With often denotes the instrument used by the agent. compared to the sales of a useful book:
Incorrect: The ball was hit by the bat. they score fast right from the beginning.
Correct: The ball was hit with the bat. Rule : When we simply speak of a thing having got better,
Rule : Both since and from are used before a noun denoting we talk of an improvement in it. When we compare
some point of time. However, while since is preceded two things, the second of which is better than the
by a verb in the perfect tense, from is used with the first, we talk of an improvement on the first thing.
non-perfect tense. Incorrect: There has been an improvement on the
Incorrect: India has been playing Test cricket from weather.
1932. Correct: There has been an improvement in the
Correct: India has been playing Test cricket since weather.
1932.

KUNDAN
Rule : In means at the end of, while within means before
Rule : While since or from is used with a point of time, for the end of, the said time duration.
is used with a period of time. Incorrect: We left at 5 o'clock expecting to come
Incorrect: The parcel has been lying here since 10 back in an hour and watch the 6 o'clock
days. movie.
Correct: The parcel has been lying here for 10 days. Incorrect: We left at 5 o'clock expecting to come
Rule : Beside means by the side of while besides means in back within an hour and watch the 6
additions to. o'clock movie.
Incorrect: He sat besides the chair.
Correct: He sat beside the chair. Some important points about prepositions
Rule : Between is used for only two things or persons while
1. The words “superior”, “inferior”, “senior”, “junior” etc.
among is used for more than two.
take the preposition “to” with them.
Incorrect: You have to choose among tea and coffee.
Incorrect: Ram is senior than Mohan.
Correct: You have to choose between tea and coffee.
Correct: Ram is senior to Mohan.
Rule : Above and below merely denote position while over
2. The words “prefer”, “preferrable”, “preferred” also take
and under also carry a sense of covering or
the preposition “to” with them
movement.
Incorrect: I prefer tea than coffee.
Incorrect: The bird flew above the lake.
Correct: I prefer tea to coffee.
Correct: The bird flew over the lake.
3. The words “inspite of” and “despite” share the same
Rule : During is used when we are talking about the time
meaning. The only difference is that inspite takes the
within which something happens. For is used when
preposition “of” whereas despite does not take any
we are talking about how long something lasts.
preposition.
Incorrect: There were few incidents of irregularity
for the Emergency years. Incorrect: Despite of bad weather, he went to school.
Correct: Despite bad weather, he went to school.
In the same way, the word “consist” takes the preposition Incorrect: The classroom comprises of twenty students.
“of” whereas “comprise” does not take any preposition. Correct: The classroom comprises twenty students.

EXERCISE

Directions: Choose the most appropriate preposition in 14. I am not obsessed _______ such ideas as you are.
each of the following sentences. a) in b) for
1. The peasant refused to grovel _______ the feet of his c) on d) with
master. 15. The thunder was accompanied _____ heavy rains.
a) on b) about a) with b) by
c) upon d) at c) up d) through
2. He was killed _______ a highway man ______ a dagger. 16. We travelled _____ a boat to reach Dhaka.
a) by; for b) by; with a) by b) on
c) in; for d) with; for c) for d) about
3. Give an example pertinent ________ the case. 17. Rata is married ______ Bobby.
a) with b) for a) with b) to
c) on d) to

K
c) at d) between
4. Are not these slums a disgrace ______ the civic
18. I brought him _____ with great difficulty.
authorities.
a) about b) round
a) for b) to
c) towards d) on c) up d) in
5. Take this medicine and you will get rid _______ the bad 19. Good sleep is necessary ______ good health.
cold. a) of b) for
a) from b) over c) at d) from
c) at d) of 20. Exercise is beneficial _____ health.
6. President Saddam Hussain has lived _______ the gun a) towards b) for
all his life. c) to d) in
21. I have been teaching in this school a)/ since several

KUNDAN
a) with b) for
c) by d) on years b)/ but have never met such a hopless class as
7. He had to repent _______ what he had done. this. c)/ No error d)
a) at b) of 22. I have an appointment a)/ on the 9th of September b)/ on
c) over d) for five O’clock. c)/ No error d)
8. The commentator seems to be enamoured _______ it. 23. Despite of a good monsoon a)/ this year, the production
a) with b) over of food grains in the b)/ country did not go up. c)/ No
c) for d) on error d)
9. The man died _______ fever last week. 24. Ordinarily, when in difficulty a)/ Renu prefers keeping
a) with b) from her counsel b)/ than running about taking advice. c)/ No
c) of d) by error d)
10. He has not been cured _______ his disease. 25. The invitation card a)/ says that Satish b)/ marries with
a) of b) from Sunita next month. c)/ No error d)
c) by d) through 26. She looked very different from a)/ the photograph b)/ I
11. He is a descendant ______ the Mughal royalty. had of her. c)/ No error d)
a) of b) from 27. The bride said a)/ that she would not b)/ marry the man
over thirty five. c)/ No error d)
c) at d) for
28. The reason a)/ of his failure b)/ is due to his negligence.
12. Kabir was born ________ humble parents.
c)/ No error d)
a) of b) from
29. The captain and his wife were invited a)/ for the cultural
c) by d) with b)/ function at my home. c)/ No error d)
13. She was a devoted wife and looked _____ her husband. 30. Yesterday, while a)/ crossing the road b)/ he was run out
a) after b) at by a truck. c)/ No error d)
c) for d) to 31. Beside chocolates a)/ the parents also b)/ bought toys
for the child. c)/ No error d)
32. None of the students a)/ saw him b)/ worthy for the 34. I am vexed a)/ at him b)/ about what he has done. c)/ No
reward he received. c)/ No error d) error d)
33. All the players a)/ except Sam b)/ were present on the 35. Mr. Smith was accused for murder a)/ but the court did
playground. c)/ No error d) net find him guilty b)/ and acquitted him. c)/ No error d)

Answers and Explanations


1. d 2. b 3. d 4. b 5. d 27. c; Replace “over” with “above”
6. c 7. d 8. a 9. c 10. a 28. b; Replace “of” with “for”
11. a 12. a 13. a 14. d 15. a 29. b; Replace “for” with “at”
16. a 17. b 18. c 19. b 20. c 30. c; Replace “run out” with “run over”
21. b; Replace “since” with “for”. 31. a; Replace “beside” with “besides”
22. c; Replace “on” with “at”. 32. c; Replace “worthy for” with “worthy of”
23. a; Omit “of” 33. c; Replace “on” with “in”
24. c; Replace “than” with “to” 34. b; Replace “at” with “with”
25. c; Replace “with” with “to” 35. a; Replace “accused for”“ with “accused of”

K
26. d; No error.

Conjunction
A conjunction is a word which merely joins together Note : "No sooner A ... than B" is very similar to "Scarcely
sentences or clauses, and sometimes words. A... when B" and has almost the same meaning.
1. That is a beautiful place and I have been there. Rule : Do not use seldom or ever in place of seldom or
2. That is the beautiful place where I have been to.

KUNDAN
never.
Incorrect: The national network seldom or ever
Classes of Conjunctions telecasts good programmes.
Conjunctions are divided into two classes: 1) Co-ordinating Correct: The national network seldom or never
and 2) Sub-ordinating. telecasts good programmes.
1. Co-ordinating conjunction : It joins together clauses of Rule : Correlative conjunctions should be followed by
equal ranks. For example. the same part of speech.
He is slow, but he is sure. Incorrect: He neither agreed to my proposal nor to
2. Sub-ordinating conjunction : It joins together clauses in his.
which one is dependent on the other. For example, Correct: He agreed neither to my proposal nor to
his.
Since you wish to excel in life, you must learn English.
Rule : Neither is followed by nor, not by or.
Some rules Incorrect: The phone neither went dead or worked
properly.
Rule: Scarcely should be followed by when, not by than. Correct: The phone neither went dead nor worked
Incorrect: Scarcely had he arrived than he had to properly.
leave again. Rule : The conjunction that is not used before
Correct: Scarcely had he arrived when he had to i) a sentence in direct narration.
leave again.
ii) an interrogative adverb or interrogative pronoun
Rule : No sooner is followed by than, not by but. in the indirect narration.
Incorrect: No sooner had we reached the station but Incorrect: He said that, "it is none of my business".
the train left. Correct: He said, "It is none of my business."
correct: No sooner had we reached the station than
Incorrect: He asked me that which was the way to
the train left.
the station.
Correct: He asked me which was the way to the Incorrect: They not only gave me standing ovation
station. but also a cash prize of `1000.
Rule : Although goes with yet or a comma (,) in the other Correct: They gave me not only standing ovation
clause. but also a cash prize of `1000.
Incorrect: Although Manohar is hardworking, but Rule : Such...as is used to denote a category whereas
he does not get a job. such...that emphasises the degree of something by
Correct: Although Manohar is hardworking, yet he mentioning its consequence.
does not get a job. Incorrect: Each member of the alliance agrees to take
Note: Though/Although is used with contradictory such action as it deems necessary.
sentences (sentences expressing opposite views). Correct: Each member of the alliance agrees to take
Incorrect: Since she was sick, she attended the party. such action that it deems necessary.
Correct: Though she was sick, she attended the party. Rule: The conjunction “lest” is followed by “should”.
Rule : Nothing else should be followed by but, not by than. Incorrect: Work hard lest you would fail.
Incorrect: Mr Bureaucrat! This is nothing else than Correct: Work hard lest you should fail.
red-tapism. Note: “Lest” is a negative word. There should be no use of
Correct: Mr Bureaucrat! This is nothing else but “not”, “nothing”, “never”, “no” with “lest”.

K
red-tapism.
Incorrect: Word hard lest you should not fail.
Rule : The correlative conjunctions indeed... but are used Correct: Word hard lest you should fail.
to emphasis the contrast between the first and the
second parts of the statement. Rule: The conjunction so—as is used for comparison in a
Incorrect: I am indeed happy with my school but it negative sentence whereas—as is used in a positive
produces famous men. or affirmative sentence.
Correct: I am indeed happy with my school that it Incorrect: He is not as tall as his brother.
produces famous men. Correct: He is not so tall as his brother.
Rule: The conjunction, “both” is followed by “and” and
Rule : In a "not only ... but also ... "sentence, the verb should not by “as well as” or “along with”.
agree with the noun or pronoun mentioned second, Incorrect: Both the boys as well as the girls were

KUNDAN
that is, the one after but also because this is the part asbent.
being emphasised. Correct: Both the boys and the girls were absent.
Incorrect: Not only the students but also the teacher Rule: The conjunction “unless” denotes condition
were responsible for what happened whereas “until” denotes time.
in the class. Incorrect: Until you work hard, you will not succeed.
Correct: Not only the students but also the teacher Correct: Unless you work hard, you will not
was responsible for what happened in succeed.
the class. Note: “Unless” and “Until” are negative words. There
Rule : Not only ... but also always comes before the same should be no use of “not” “nothing”, “never” with
part of speech. unless and until.

EXERCISE
Directions: In each of the following questions, find out 4. It is not (a)/ such a good book (b)/ that I expected. (c)/
which part of the sentence has an error. The error may be No error (d)
grammatical or idiomatic. If there is no mistake, the answer 5. The downfall of this kind (a)/ is to be attributed to (b)/
is ‘No error’. nothing else than pride. (c)/ No error (d)
1. No sooner did I open the door (a)/ when the rain, heavy 6. Unless he does not return (a)/ all the library books (b)/
and stormy, rushed in (b)/ making us shiver from head to he will not be relieved from the service. (c)/ No error (d)
foot. (c)/ No error (d) 7. You must either tell me (a)/ the whole story or, at least,
2. Be smart (a)/ not only in dress (b)/ and also in action. (c)/ (b)/ the first half of it. (c)/ No error (d)
No error (d) 8. The old woman has had the best medical facilities
3. Hardly had I reached the airport (a)/ where I learnt (b)/ available (a)/ but she will not be cured (b)/ unless she
about the powerful bomb explosion. (c)/ No error (d) does not have a strong desire to live. (c)/ No error (d)
9. My books have been (a)/ missing from my room (b)/ till 23. Ranjeet is as (a)/ fast as or (b)/ perhaps faster than Rohit.
yesterday. (c)/ No error (d) (c)/ No error (d)
10. The manager of the bank was busy; (a)/ so he asked 24. Though she was sick (a)/ but she (b)/ attended the party.
them to come and (b)/ see him between two to three in (c)/ No error (d)
the afternoon. (c)/ No error (d) 25. Neither the famine (a)/ or the subsequent fire (b)/ was
11. He is as clever if (a)/ not cleverer (b)/ than his brother. able to destory the spirit of the poeple. (c)/ No error (d)
(c)/ No error (d) 26. No sooner did (a)/ the Chairman begin speaking, some (b)/
12. No sooner did the sun rise (a)/ when we took a hasty participants started shouting slogans. (c) No error (d)
breakfast (b)/ and resumed the journey. (c)/ No error (d) 27. The Management Committee has requested me (a)/ not
13. Hardly had the children (a)/ left the school (b)/ than it to take any action (b)/ unless and until I don’t see all the
began to rain. (c)/ No error (d) documents. (c)/ No error (d)
14. Because he is physically strong, (a)/ therefore he was 28. As he is rich, (a)/ so he spends (b)/ lavishly. (c)/ No error
selected (b)/ for the school boxing team (c)/ No error (d) (d)
15. The reason for (a)/ his failure is because (b)/ he did not 29. I am but (a)/ a humble seeker after truth (b)/ and bent
work hard. (c)/ No error (d) upon finding it. (c)/ No error (d)
16. Bharat asked him (a)/ that which was the way (b)/ to the 30. Hardly had he arrived (a)/ when the house caught fire (b)/
post-office. (c)/ No error (d) and everything was reduced to ashes. (c)/ No error (d)
17. Unless you do not listen to his advice (a)/ I am not going 31. Make haste (a)/ lest you might (b)/ miss the train. (c)/ No

K
(b)/ to help you. (c)/ No error (d) error (d)
18. How do you say (a)/ that neither he or (b)/ Ravi has 32. Neither did he (a)/ accept the punishment (b)/ or
qualified in the examination? (c)/ No error (d) apologised (c)/ for his mistake. (d)/ No error (e)
19. Being a strict vegetarian (a)/ he depended on milk and 33. No sooner did the jeep arrive at the station (a)/ than a young
fruit (b)/ and had to cook for himself. (c)/ No error (d) police officer (b)/ jumped out of it. (c)/ No error (d)
20. Even though the shirt is rather expensive (a)/ but I wish 34. If you hate me, (a)/ then you should (b)/ leave my house.
to (b)/ purchase it with my own money. (c)/ No error (d) (c)/ No error (d)
21. Unless Ashish does not (a)/ return all the library books, 35. The managing director well as the Board members (a)/
(b)/ he will not be awarded the pass certificate. (c) No was in favour of taking strict action (b)/ against the
error (d) striking workers. (c)/ No error (d)
22. Mohan could not (a)/ go to the picnic (b)/ for his mother

KUNDAN
(c)/ was not well. (c)/ No error (d)

Answers and Explanations


1. b; Replace ‘with’ by ‘than’ 19. c; Replace ‘and’ with ‘or’
2. c; Replace ‘and’ with ‘but’ 20. b; Remove ‘but’
3. b; Replace ‘where’ with ‘when’ 21. a; Replace ‘unless’ with ‘if’
4. c; Replace ‘that’ with ‘as’ 22. d; No error
5. c; Replace ‘than’ with ‘but’ 23. b; Remove ‘as’
6. a; Replace ‘does not return’ with ‘returns’ 24. b; Replace ‘but’ with ‘yet’
7. d; No error 25. b; Replace “or” with “nor”
8. c; Replace ‘does not have’ with ‘has’ 26. b; Add ‘than’ before ‘some’
9. c; Replace ‘till’ with ‘since’ 27. c; Remove ‘don’t
10. c; Replace ‘to’ with ‘and’ 28. b; Remove ‘so’
11. a; Add ‘as’ after ‘clever’ 29. c; Remove ‘and’
12. b; Replace ‘when’ with ‘than’ 30. d; No error
13. c; Replace ‘than’ with ‘when’ 31. b; Replace “might” with “should”
14. b; Remove ‘therefore’ 32. c; Replace ‘or’ with ‘nor’
15. b; Replace ‘because’ with ‘that’ 33. d; No error
16. b; Remove ‘that’ 34. b; Remove ‘then’
17. a; Remove ‘do not’ 35. a; Add ‘as’ before ‘well’
18. b; Replace ‘or’ with ‘nor’
Noun
Words are divided into different kinds or classes according Incorrect: The jury were unanimous in their opinion.
to the work they do in a sentence. These kinds or classes are Correct: The jury was unanimous in its opinion.
known as Parts of Speech. They are eight in number: 1. Noun Rule: A collective noun takes a plural verb and is
2. Pronoun 3. Verb 4. Adjective 5. Adverb 6. Preposition 7. substituted by a plural pronoun when the
Conjunction 8. Interjection. individuals of which it is composed are thought of.
What is noun? Noun is a word used to name a person, place net behaving as a single unit
or thing. Incorrect: They jury was divided in its opinions.
For example: Correct: The jury were divided in their opinions.
Rama was an epic king.
His valour brought him laurels. 4. Abstract Noun
An abstract noun is usually the name of a quality, action or
Kinds of Noun state considered apart from the object ( which is usually a
common noun and often qualified by an adjective) to which it

K
1. Common Noun belongs. For example:
Innocence is integral to a child.
A common noun is a name given in common to every person Love knows no bound.
or thing of the same class or kind. For example;
I write with a pen. 5. Material Noun
The student asked me a few questions.
A material noun is the name of a material ( matter from which
2. Proper Noun a thing is made). For example:
Cotton is the source of natural fibre.
A proper noun is the name of a particular person or Milk constitutes a nutritious food.
place. Rule: A material noun does not take an article (a, an, the)

KUNDAN
For example: before it.
Ramu is a wise fellow. Incorrect: The tobacco is a cash crop.
Patna lacks basic civic amenities. Correct: Tobacco is a cash crop.
Rule: Proper nouns are always written with a capital Rule: It takes articles when treated as common nouns.
letter at the beginning. Incorrect: They offered me glass of water.
Incorrect: Earlier the capital of india was in calcutta. Correct: They offered me a glass of water.
Correct: Earlier the capital of India was in Calcutta.
Rule: Proper nouns are sometimes used as common nouns. Numbers
Incorrect: LK Advani is today regarded as Sardar
Patel of India. Numbers are of two kinds-
Correct: LK Advani is today regarded as the Sardar (1) Singular Number
Patel of India. (2) Plural Number
Generally while changing singular noun to plural we add
Rule: In this case proper nouns are always preceded by s, es in the end.
articles. For example
She is a Lata Mangeshkar. cow cows
Kalidas is the Shakespeare of India. chair chairs
school schools
3. Collective Noun
Mango Mangoes
A Collective noun is the name of a group of persons or
On the surface some nouns appear to be singular but in
things. For example army, committee, crowd, fleet, flock, herd,
meaning they are treated as plural.
jury, mob, parliament, team.
For example,
Rule: A collective noun usually takes a singular verb and
cattle, clergy, people, children, police, poultry
is substituted by a singular pronoun.
peasantry peasants of a country Certain other nouns - caves, riches (valuables),
admirality the Government department that alms, funds, proceeds (profit), annals (record).
controls navy Incorrect : I bought a trousers yesterday.
infantry soldiers who fight on foot Correct :I bought a pair of trousers yesterday.
cavalry horse soldiers Some nouns are always singular and used as singular
gentry people belonging to high social class only. For example
nobility people belonging to aristocratic Poetry, information, machinery, dust, furniture, dirt,
class traffic, money, music.
Incorrect: The peasantry has always stood as Some nouns have one meaning in singular form and
rockstars for the development of a country. another in plural. For example,
Correct: The peasantry have always stood as Singular Plural
rockstars for the development of a country. Advice – counsel Advices – information
Some nouns look like plural but are always used as Air – wind Airs – affected manners
singular. For example, Colour – hue Colours – appearance, the
Subjects - Mathematics, economics, physics, ethics, flag of regiment
politics, classics. Compass – limit, Compasses – an instrument for
Rule: When these subjects denote qualities or property, range drawing circles
they are used in plural sense. For example, Custom – habit Customs – duties levied on

K
Incorrect: His mathematics is weak. imports
Correct: His mathematics are weak. Force – strength Forces – troops
Diseases - Rickets, AIDS, diabetes, mumps, measles. Good – profit Goods – movable property
Some other nouns - News, gallows, summons, Pain – suffering Pains – careful effort
innings. People – men & Peoples – nations
Some nouns are plural in form and in meaning, normally women
they are instruments or dress articles which have two parts Spectacle– sight Spectacles – eye-glasses
and make a kind of pair. Do not use a/an before these nouns. Manner – method Manners – correct behaviour
For example, Letter – a unit Letters – letters, literature
Spectacles, tongs, scissors, pincers, bellows, of alphabet
trousers, drawers, breaches, pants, knickers, Ground – Earth Grounds – reasons, enclosed

KUNDAN
pantaloons. land

EXERCISE

Directions: Find out the error in each of the following name and fame not by (b)/ labour but by chance. (c)/
sentences, if any. If there is no error, the answer is ‘d’. No error (d)
1. The sceneries (a)/ of Kashmir is more beautiful (b)/ than 8. Not to speak of failure (a)/ even a tragedy (b)/ does not
that of Darjeeling. (c)/ No error (d). affect his high spirits. (c)/ No error (d)
2. One of the reason (a)/ for his failure (b)/ in the examination 9. Running a five-stars (a)/ hotel needs much more (b)/
is his carelessness. (c)/ No error (d). money than what we have in our account. (c)/ No error
3. All his sister-in-laws (a)/ are extremely beautiful to look (d)
at, so the question (b)/ of selection is very puzzling. (c)/ 10. All the Chiefs Ministers (a)/ are responsible for the (b)/
No error (d). pitiable condition of their states. (c)/ No error (d)
4. The tables’s legs (a)/ were broken by my (b)/ angry 11. The wall of this magnificent (a)/ building is (b)/ fifty foot
brother. (c)/ No error (d) high and its paintings are very striking. (c)/ No error (d)
5. There were only (a)/ certain boy that were sincere and 12. He gave me two important informations (a)/ I had been
diligent that (b)/ could do well in the examination. (c)/ No waiting for (b)/ for the previous two months. (c)/ No
error (d) error (d)
6. These type of books (a)/ are certainly helpful to the 13. All the girls students (a)/ of the college are advised (b)/
students (b)/ preparing for the Banking Service to set in the Girls’ Common Room. (c)/ No error (d).
Examinations. (c)/ No error (d) 14. One of the problem (a)/ with you is that you do not (b)/
7. You are among those (a)/ luckiest man who attained come in time. (c)/ No error (d)
15. When he returned (a)/ from America, he informed me (b)/ 25. Cattles are grazing (a)/ in the field (b)/ near my house.
that he had come here to do urgent work. (c)/ No error (d) (c)/ No error (d)
16. I do my best (a)/ but I don’t understand why I do not get 26. He gave me (a)/ a five-rupees note (b)/ and asked me to
expected (b)/ marks in the examination. (c)/ No error (d) get him a few cigarettes. (c)/ No error (d)
17. The pretty woman (a)/ sitting in the car (b)/ is the 27. I have not read (a)/ the first and second (b)/ chapter of
daughter of one of the headman of the village. (c)/ No the book. (c)/ No error (d)
error (d) 28. These kind (a)/ of shirts (b)/ are rather expensive for him
18. Owing to his heavy engagement (a)/ he could not write to buy. (c)/ No error (d)
even two-third (b)/ of the book of which the publications 29. All the furnitures have been (a)/ sent to the new (b)/
was expected before January. (c)/ No error (d) house located in the village. (c)/ No error (d)
19. You will not be allowed (a)/ to go anywhere else (b)/ to 30. The Cotton Textile Company cannot work properly (a)/
play outdoor games. (c)/ No error (d) unless it employs (b)/ a talented sale representative. (c)/
20. The beautiful surrounding (a)/ of the palace (b)/ attracted No error (d)
a large number of tourists coming from abroad. (c)/ No 31. The poultries (a)/ situated near my house (b)/ are for
error (d) sale. (c)/ No error (d)
21. He did not know (a)/ that the headquarter (b)/ of the 32. They left (a)/ their luggages (b)/ at the railway station
army was in Delhi. (c)/ No error (d) and went to save the child. (c)/ No error (d)
22. Liberalisation of economy (a)/ has given birth to (b)/ 33. The disclosure of the synopsis of (a)/ the Thakkar

K
many uncommon phenomenon in the society. (c)/ No Commission Report (b)/ has raised several vital question.
error (d) (c)/ No error (d)
23. Thousand of young (a)/ protesters were standing (b)/ at 34. I don’t think (a)/ it is your house (b)/ it is somebody’s
Jantar Mantar and supporting Anna Hazare. (c)/ No error else. (c)/ No error (d)
(d) 35. The table’s wood (a)/ is infested with (b)/ termites and I
24. It is very difficult (a)/ to chase (b)/ a huge score in the am likely to dispose it off. (c)/ No error (d)
fourth inning. (c)/ No error (d)

Answers and Explanations


1.
2.
3.
4.

5.
6.
7.
8.
a;
a;
a;
a;

b;
a;
b;
d;
KUNDAN
Replace “sceneries” with “scenery”.
Replace “reason” with “reasons”.
Replace “sister-in-laws” with “sisters-in-law”
Replace “the table’s legs” with “the legs of the
table.”
Replace “boy” with “boys”.
Replace “type” with “types”
Replace “man” with “men”
No error
19. e;
20. a;
21. b;
22. c;
23. a;
24. c;
25. a;
26. b;
27. c;
No error
Replace “surrounding” with “surroundings”
Replace “headquarter” with “headquarters”
Replace “phenomenon” with “phenomena”
Replace “thousand” with “thousands”
Replace “inning” with “innings”
Replace “cattles” with “cattle”
Replace “rupees” with “rupee”
Replace “chapter” with “chapters”
9. a; Replace “stars” with “star”. 28. a; Replace “kind” with “kinds”
10. a; Replace “Chiefs” with “Chief”. 29. a; Replace “furnitures” with “furniture”
11. c; Replace “foot” with “feet” 30. c; Replace “sale representative” with “sales
12. a; Replace “informations” with “information” representative”
13. a; Replace “girls” with “girl” 31. a; Replace “poultries” with “poultry”
14. a; Replace “problem” with “problems” 32. b; Replace “luggages” with “luggage”
15. d; No error 33. c; Replace “question” with “questions”
16. d; No error 34. c; Replace “else” with “else’s”
17. c; Replace “headman” with “headmen” 35. a; Replace “The table’s wood” with “The wood of the
18. d; Replace “third” with “thirds” table”.
Pronoun
A pronoun is a word used instead of a noun. Rule: When two singular nouns joined by and are preceded
Amit is absent because he is ill. by each or every, the pronoun must be singular.
Incorrect: Every teacher and every student got their
Kinds of Pronoun dress ready.
Correct: Every teacher and every student got his
1. Personal Pronouns dress ready.
Personal pronouns are those which stand for three persons. Rule: When two or more singular nouns are joined by or,
You may ask: which three? Well, here are the three persons either... or or neither... nor, the pronoun is generally
grammar knows: singular.
(1) First Person: the person speaking. Incorrect: Neither Mani nor Guni has brought their
Pronouns used: I, we. identity card.
(2) Second Person: the person spoken to. Correct: Neither Mani nor Guni has brought his
Pronoun used: you. identity card.

K
(3) Third Person: the persons other than the above two. The 123 rule of joint pronoun: When a pronoun
Pronouns used: He, she, it, they. refers to more than one noun or pronoun of different
Rule: A personal pronoun must be of the same number, persons, it must be of the first person plural in
gender and person as the noun for which it stands. preference to the second, and of the second person
Incorrect: Every man must love their own country. plural in preference to the third.
Correct: Every man must love his own country. Incorrect: You and I, husband and wife, have to look
after your home.
Rule: A pronoun must agree with its antecedent in person,
Correct: You and I, husband and wife, have to look
number and gender.
after our home.
Incorrect: I am not one of those who believe
Note : 123 is a shorthand to remember the above rule, 1
everything I see.

KUNDAN
stands for first person, 2 for second person, and 3
Correct: I am not one of those who believe everything
for third person.
they see.
Rule: When a pronoun stands for a collective noun, it 231 manners: Generally the second person should
must be in the neuter gender and come before the third and the third person before
i) singular number, if the collective noun is viewed as a the first.
whole. Incorrect : I, you and he had forgotten our roots.
ii) Plural number, if the collective noun conveys the Correct : You, he and I had forgotten our roots.
idea of separate individuals.
Rule: When expressing a negative idea the sequence of
Incorrect: The committee decided the matter without
personal pronouns should be 123.
leaving its seats.
Incorrect: you, I and he are guilty of committing
Correct: The committee decided the matter without
theft.
leaving their seats.
Correct: I, You, and he are guilty of committing
Rule: When two or more singular nouns are joined by and theft.
refer to Rule: The complement of the verb “to be”, when it is
i) separate persons, the pronoun used for them must be expressed by a pronoun, should be in the nominative
plural. case.
ii) the same person, the pronoun used for them must be Incorrect: It is him whom we are looking for.
singular. Correct: It is he whom we are looking for.
Incorrect: Both Manas and Appu showed his
fondness for his father. Rule: If a pronoun is used as the object of a verb or of a
Correct : Both Manas and Appu showed their preposition, it should be in the objective case.
fondness for their father. Incorrect: Let you and I take a dip into holy water.
Correct: Let you and me take a dip into holy water.
2. Reflexive Pronouns indefinite pronoun. For example, one, none, all, some,
somebody, nobody, few, many, any, other, anybody, anyone,
When the action done by the subject reflects or turns back
everyone. In sentences like.
upon the subject. In other words, the subject and the object
They say India will reach on top of the world one day.
refer to the same person or thing.
they is an indefinite pronoun.
He killed himself.
Emphatic pronouns: For the sake of emphasis.
Rule: In referring to anybody, everybody, everyone,
I myself went to finalise the deal.
anyone,each etc, the pronoun he or she is used
Rule: When the following verbs are used reflexively a according to the context. But when we are not
reflexive pronoun must be put after them: absent, talking categorically of the females, the pronoun of
acquit, apply, avail, enjoy, exert, forget, over-reach, the masculine gender is used.
pride, resign. Incorrect: Each can buy whatever she wants with
Incorrect : I absented from the class. her own money.
Correct : I absented myself from the class. Correct: Each can buy whatever he wants with his
Rule: The following verbs are usually not followed by a own money.
reflexive pronoun: bathe, break, burst, feed, form, Rule: The indefinite pronoun “one” should be used
gather, hide, keep, lengthen, make, move, open, throughout, if used at all.
qualify, rest, roll, speed, steal, stop, turn. Incorrect: One should take care of his house.

K
Incorrect : I bathed myself in the river. Correct: One should take care of one's house.
correct : I bathed in the river.
Rule: None is used in the singular or plural as the sense
Rule: A reflexive pronoun can not be used as a substitute may require.
for the subject. Incorrect: Each boy was accompained by an adult
Incorrect : Mohan and myself decided to join politics. but there were none with the orphan.
Correct: Mohan and I decided to join politics. Correct: Each boy was accompained by an adult but
there was none with the orphan.
3. Demonstrative Pronouns Rule: Anyone should be used when more than two persons
The pronouns that are used to point out the objects to which or things are spoken of.

KUNDAN
they refer, are called demonstrative pronouns. For example, Incorrect: I could not meet either of my three
this, these, that, those, such. brothers at home.
This is a book. Correct: I could not meet anyone of my three
brothers at home.
Rule: When two things which have already been
mentioned are refered to, “this” refers to the thing
last mentioned, “that” to the thing first mentioned. 5. Distributive Pronouns
Incorrect: India and England shared a relationship The pronouns that refer to persons or things one at a time
of exploitation; this was the slave and are called distributive pronouns. For example, each, either,
that the master. neither.
Correct: India and England shared a relationship of each everyone of a number of persons or
exploitation; this was the the master and things taken singly
that the slave. either the one or the other of two
Rule: The pronoun “it” comes before the phrase or clause neither not the one nor the other of two
to which it refers whereas “this” follows the phrase Rule: A distributive pronoun is used as a singular.
or clause it refers to. Incorrect: Either of the choices fit the answer.
Incorrect: This is true that bell-bottoms were once Correct: Either of the choices fits the answer.
in fashion. Rule : Each may come after the object of the verb only if
Correct: It is true that bell-bottoms were once in the object contains a number.
fashion. Incorrect: The boys narrated stories each.
Correct: Each of the boys narrated stories.
4. Indefinite Pronouns
When a pronoun refers to a person or a thing in a general 6. Relative Pronouns
way, but not to any person or thing in particular, it is called an A relative pronoun is a pronoun that refers or relates to some
noun going before, which is called its antecedent. Besides, it Incorrect : I have an uncle who I love and is a
is a pronoun that serves as a conjunction as well because it management guru.
connects two statements. For example; Correct: I have an uncle whom I love and who is a
She worked in the kitchen which was like a bedroom. management guru.
Rule: The relative pronoun “that” has the same form in Rule: The relative pronoun should be placed as near as
the singular and the plural. possible to its antecedent.
Incorrect: They those worship money only are Incorret: The office was located in the heart of the
deprived of life's other rewards. town which had state-of-the-art interior
Correct: They that worship money only are deprived decoration.
of life's other rewards.
Correct : The office which had state-of-the-art
Rule: That may be used as an adverbial accusative interior decoration, was located in the
meaning on which, in which or at which. heart of the town.
Incorrect : I remember the day which he came. Rule: No conjunctions should be placed before the
Correct: I remember the day which he came on. relative pronoun except to join together two relative
or, clauses referring to the same antecedent.
I remember the day that he came. Incorrect : He had a cow, and which gave him milk.
Rule: The relative pronoun that is preferred to who or Incorrect : He had a cow, which gave him milk.

K
which Rule: A pronoun (or noun) in the possessive case should
i) after adjectives in the superlative degree. not be used as the antecedent to a relative pronoun.
ii) after all, same, any, none, nothing, only. Incorrect: India became dependant on his batting
iii) after interrogative pronouns who, what. who scored runs with a passion.
iv) after antecedents, one denoting a person and Correct: India became dependant on batting of him
the other an animal or a thing. who scored runs with a passion.
Incorrect: It was the best food which we ever tasted. Rule: The relative pronoun is sometimes wrongly omitted
when it is the subject of a clause.
Correct: It was the best food that we ever tasted.
Incorrect: He has a resilience would see him through
Rule: The word but after a negative, often has the force of

KUNDAN
anything.
a relative pronoun and is equivalent to who... not,
which... not. Correct: He has a resilience that would see him
through anything.
Incorrect: There is no city but does not have a huge
population. Rule: The relative pronoun “what” is used without an
antecedent.
Correct: There is no city but has a huge population.
Incorrect: I mean that what I say.
Rule: As the relative pronoun refers to a noun or pronoun,
it must be of the same number and person as its Correct: I mean what I say.
antecedent (that is, the noun or pronoun referred Rule: When the antecedent is the same, the consequent
to). should be as or that; when the antecedent is such,
Incorrect: I, who is your captain, shall never desert the consequent should be as.
you. Incorrect: The book was such that he could read.
Correct: I, who am your captain, shall never desert Correct: The book was such as he could read.
you. Rule: Pronoun of the third person plural should not be
Rule: The case of the relative pronoun depends upon its used as antecedent to who and that.
relation to the verb in the clause in which it occurs. Incorrect: They who are students of the school
Incorrect: I was the boy whom they thought need not buy tickets.
committed the theft. Correct: Those who are students of the school need
Correct: I was the boy who, they thought, committed not buy tickets.
the theft.
Rule: If a relative pronoun is used in both subjective and 7. Interrogative Pronouns
objective cases, we cannot put one pronoun in place The pronouns used for asking questions are called
of two. interrogative pronouns.
Rule: Which implies selection, that is it implies a question Incorrect: What of the pictures do you like best?
concerning a limited number. It is used of both Correct: Which of the pictures do you like best?
persons and things.

EXERCISE

Directions: In the following questions, some of the 17. There is no question a)/ of my failing b)/ in the
sentences have errors and some have none. Find out which examination. c)/ No error d)
part of the sentence has an error. If there is no error, mark 18. The number of vehicles a)/ plying on this road b)/ is
your answer as ‘No error’. more than on the main road. c)/ No error d)
1. The interviewer asked the candidate a)/ to what district 19. Being a destitute, a)/ I admitted him b)/ to an old people’s
b)/ he came from. c)/ No error d) home. c)/ No error d)
2. The master did not know a)/ who of the servants b)/ 20. This is not the a)/ sense which b)/ concerns us here. c)/
broke the glass. c)/ No error d) No error d)
3. The ruling party stood a)/ for implementation of the Bill 21. One should make a)/ his best efforts b)/ if one wishes to
b)/ and was ready to stake their political existence. c)/. achieve success in this organisation. c)/ No error d)
No error d) 22. May I a)/ know who you want b)/ to see please. c)/ No

K
4. This is the boy a)/ whom I think had won b)/ the gold error d)
medal in the Dance competition. c)/ No error d) 23. Our is the only country in the world a)/ that can the
5. Wherever they go, a)/ the Indians easily adapt to b)/ boast of b)/ unity in diversity. c)/ No error d)
local circumstances, c)/ No error d) 24. Last summer he went a)/ to his uncle’s village, b)/ and
6. He is a)/ taller b)/ than me. c)/ No error d) enjoyed very much. c)/ No error d)
7. It is not easy for anyone to command a)/ respect from 25. If I were him, a)/ I would have taught b)/ those cheats a
both one’s friends and critics b)/ as Dr. Johnson did for lesson. c)/ No error d)
his integrity and honesty. c)/ No error d) 26. Those sort of people a)/ usually do not b)/ earn fame in
8. Mahatma Gandhi taught us a)/ that one should respect society. c)/ No error d)
b)/ the religions of others as much as his own. c)/ No 27. He could not plan his strategy a)/ until he knew b)/ whom
error d) his opponents could be. c)/ No error d)

KUNDAN
9. He, you and I a)/ will manage b)/ this problem together. 28. Being a very a)/ hot day I b)/ remained indoors. c)/ No
c)/ No error d) error d)
10. Was it him a)/ who got injured b)/ in an accident this 29. Concurrent with this programme, a)/ educational
morning. c)/ No error d) institutions may be urged b)/ to inculcate patriotism c)/
11. I suppose a)/ you have heard the latest news b)/ John in each and every one of its pupils. c)/ No error d)
has married that girl. c)/ No error d) 30. Inspite of the doctor’s stern warning, a)/ he continued
12. As soon as he a)/ saw his mother b)/ he ran to her and taking b)/ sugar in his tea. c)/ No error d)
embraced. c)/ No error d) 31. Had I come a)/ to know about b)/ his difficulties I would
13. Due to me being a newcomer a)/ I was unable to get a have certainly helped. c)/ No error d)
house b)/ suitable for my wife and me. c)/ No error d) 32. One of them a)/ forget to take their bag b)/ from the
14. The audience a)/ are requested b)/ to be in its seats. c)/ school. c)/ No error d)
No error d) 33. Mr. Sunil Pawar, our representative, a)/ he will attend the
15. The scientist must follow a)/ his hunches and his data meeting b)/ on our behalf. c)/ No error d)
b)/ wherever it may lead. c)/ No error d) 34. If the teacher is good a)/ the students will respond b)/
16. He is the general a)/ whom the reporters agree b)/ was positively to them. c)/ No error d)
the most popular with the troops. c)/ No error d)

Answers and Explanations


1. b; Replace ‘to what’ with ‘which’ 3. c; Replace ‘their’ with ‘its’
2. b; Replace ‘who’ with ‘which’ 4. b; Replace ‘whom’ with ‘who’
5. b; Add ‘themselves’ after ‘adapt’ 20. b; Replace ‘which’ with ‘that’
6. c; Replace ‘me’ with ‘I’ 21. b; Replace ‘his’ with ‘one’s’
7. b; Add ‘one’s’ before ‘critics’ 22. b; Replace ‘who’ with ‘whom’
8. c; Replace ‘his’ with ‘one’s’ 23. a; Replace ‘our’ with ‘ours’
9. a; The correct form is ‘You, he and I’ 24. c; Add ‘himself’ after ‘enjoyed’
10. a; Replace ‘him’ with ‘he’ 25. a; Replace ‘him’ with ‘he’
11. a; Add ‘that’ after ‘suppose’ 26. a; Replace ‘Those’ with ‘That’
12. c; Add ‘her’ after ‘embraced’ 27. c; Replace ‘whom’ with ‘who’
13. a; Replace ‘me’ with ‘my’ 28. a; Add ‘It’ before ‘being’
14. c; Replace ‘its’ with ‘their’ 29. c; Replace ‘its’ with ‘their’
15. c; Replace ‘it’ with ‘they’ 30. c; Remove ‘his’
16. b; Replace ‘whom’ with ‘who’ 31. c; Add ‘him’ after ‘helped’
17. d; No error 32. b; Replace ‘their’ with ‘his’
18. c; Add ‘that’ after ‘than’ 33. b; Remove ‘he’
19. a; Add ‘He’ before ‘being’ 34. c; Replace ‘them’ with ‘him’

K
person or thing.
For example,
The girls were singing,
The watch has been found.
Verb
A verb is a word used to tell or assert something about some Verbs of Incomplete Predication
The intransitive verb which requires something to make the
sense complete is called a verb of incomplete predication.
For example be, become, seem, grow, taste, appear.
That which is required to make the sense (of these verbs)
Kinds of Verb complete is known as the complement of the verb or the

KUNDAN
completion of the predicate.
Transitive and Intransitive Verbs For example,
A Transitive verb is a verb that denotes an action which The family appears.
passes over from the doer or agent to an object. Transitive Rule: When the subjective complement is a noun (or a
means passing over. For example, pronoun), it is in the same case as the subject, that is,
India won the World Cup. in the nominative case.
An Intransitive verb is a verb that denotes an action For example
which does not pass over to an object, or that expresses a The lawyer called the witness a liar.
state of an object, or that expresses a state of being.
Intransitive means not passing over. For example, Rule: When the objective complement is a noun it is in the
The sun shines brightly. objective case in agreement with the object.

Rule: When an intransitive verb is used in a causative Subject-Verb Agreement


sense, it becomes transitive.
1. He sat in a chair. Language has its base in logic and harmony. The subject and
2. He sat me on chair. the verb are the essential parts of a sentence. It is imperative
Rule: Verbs like, fall, lie, rise and sit are made transitive by that the verb agree with the subject in number and person.
a slight change in their spellings. The transitive is Rule: Two or more singular subjects connected by and
the causative of the corresponding intransitive verb. usually take a verb in the plural.
Incorrect: The poachers were caught yesterday when Incorrect: Computer and telecommunication has
they fell trees illegally. metamorphosed information technology.
Correct : The poachers were caught yesterday when Correct: Computer and telecommunication have
they felled trees illegally. metamorphosed information technology.
Rule: If two singular nouns refer to the same person or Rule: When a plural noun denotes some specific quantity
thing, the verb must be singular. or amount considered as a whole, the verb is
Incorrect : The poet and critic have been honoured. generally singular.
Correct : The poet and critic has been honoured. Incorrect: Five hours are too short a time to judge
Rule: If two subjects together express one idea, the verb is one's character.
in the singular. Correct: Five hours is too short a time to judge one's
Incorrect: Bread and butter are essential for one's life. character.
Correct: Bread and butter is essential for one's life.
Rule: If the singular subjects are preceded by each or every,
The Infinitive
the verb is usually singular. Usually a verb is limited by the number and person of its
Incorrect: Every man and woman in a family are subject.
responsible for the upbringing of a child. For example,
Correct: Every man and woman in a family is He plays cricket.
responsible for the upbringing of a child. They play cricket.
A verb, which is not limited by person and number as a
Rule : Two or more singular subjects connected by or, nor, verb that has a subject, is called the verb infinite or the
either ... or, neither... nor take a verb in the singular. infinitive. Infinitives are of two kinds. (1) Simple Infinitive
Incorrect: Either Manoj or Madhukar have the key

K
and (2) Gerundial or Qualifying Infinitive.
to this problem. (1) Simple Infinitive - when the infinitive is used like a noun.
Correct: Either Manoj or Madhukar has the key to (i) as the subject of a verb.
this problem. To name is to give an identity.
Rule: When the subjects, joined by or or nor are of different (ii) as the object of a verb.
numbers, the verb must be plural, and the plural I have decided to leave the place.
subject must be placed next to the verb. (iii) as the complement of a verb.
Incorrect : Neither Rekha nor her friends was present Her hobby is to dance.
at the party. (iv) as the object of a preposition.
Correct : Neither Rekha nor her friends were present He is about to go.
at the party. (v) as the complement of the object.

KUNDAN
Rule: When the subjects, joined by or or nor are of different She heard her weep.
persons, the verb agrees in person with the one nearest (2) Gerundial or Qualifying Infinitive-when the infinitive
to it. is used for the following purposes:
Incorrect : Either she or I pays the fees. (i) to qualify a verb.
Correct : Either she or I pay the fees. He slept to refresh himself.
(ii) to qualify an adjective.
Error of Proximity: The verb is made to agree in number Tea is easy to prepare.
with a noun near it instead of with its proper subject. (iii) to qualify a noun.
This is wrong and should be avoided. He has a point to make.
Incorrect : The behaviour of the students were not proper. (iv) to qualify a sentence.
Correct: The behaviour of the students was not proper. To be very frank, I hate your smile.
Rule: Words joined to a singular subject by with, as well as, Rule: The infinitive is used without to after verbs like help,
etc. are parenthetical. The verb should therefore be watch, bid, dare, hear, let, make, need and see.
put in the singular. Incorrect : I bid him to quit the organisation.
Incorrect: The guru as well as his disciples are Correct : I bid him quit the organisation.
committed to celibacy. Rule: If, however, the structure is do + dare or do + need,
Correct: The guru as well as his disciples is the infinitive is used with to.
committed to celibacy. Incorrect: You do not need worry about us.
Rule: Either, neither, each, everyone, many a must be Correct: You do not need to worry about us.
followed by a singular verb. Rule: The following verbs are followed by the infinitive.
Incorrect: Many a man have resigned in crisis. agree, arrange, attempt, care, cease, consent, decide,
Correct: Many a man has resigned in crisis. determine, endeavour, fall, forget, hesitate, hope, learn,
manage, neglect, prepare, promise, propose, refuse,
regret, remember, seem, swear, undertake.
Incorrect: I agree for helping you out in trouble. Auxiliary Verb
Correct: I agree to help you out in trouble.
Rule: Prepositions but and except take the infinitive without Auxiliary literally means giving help, So an auxiliary verb
to. is one that helps the main verb form in tenses, moods, voices
Incorrect: There is no alternative but to help him out. etc. The following verbs are auxiliaries: is, are, am, was, were,
Correct : There is no alternative but help him out. be, can, could, dare, do, does, did, have, has, had, may,
might, must, need, ought, shall, should, will, would, used to.
Rule: Expressions would rather, would sooner, rather than,
sooner than, had better are followed by infinitive 1. May implies permission, doubt or possibility.
without to. Ex. May I come in? (permission)
Incorrect : I would rather to go for batting.
It may rain tonight. (possibility)
Correct : I would rather go for batting.
2. Might is the past form of “may” and it implies more doubt
Participle than “may”.

(1) Present Participle Ex. If the clouds are salted, the rains might come.
(2) Past Participle 3. Can is used to express ability.
(3) Perfect Participle Ex. He can do the work.
Rule: Present participle is used when all the parts of a

K
4. Could is the past form of can but it does not necessarily
sentence have the same subject; represent past time. Often it implies a more uncertain
Incorrect: Opened the drawer, he took out his gun. condition.
Correct: Opening the drawer, he took out his gun.
Ex. He could refuse, but she never does.
Rule: When a sentence has two different subjects, avoid
using present participle. Instead a clause should be 5. Should expresses the idea that something must be done
used in such instances. or is important.
Incorrect: While going to school, a dog jumped Should and ought to have similar meanings, but ought is
at me. followed by “to” “Ought to” has a more objective force
Correct: While I was going to school, a dog jumped and is used when we are talking about laws, duties and
at me. regulations.

KUNDAN
Rule: Some past participles are used only as adjectives and Ex. We ought to see her tomorrow.
qualify a noun. They can be used as past participles
6. Must is used to give strong advice or orders.
with changed spellings. For example, shrunken,
drunken, sunken, graven, molten. Ex. He really must stop drinking.
Incorrect: He and his brother have drunken wine 7. Do is used to make question and negative forms of
last night. ordinary verbs.
Correct: He and his brother have drunk wine last
night. Ex. Do you know him?
Rule: Perfect participle is used when one activity is I don’t like swimming.
completely over. “Do” is always used in question-tags.
Having worked hard, he felt tired.
Ex. You know painting, do you?

EXERCISE

Directions: In the following questions, some of the 3. People have a right to criticise (a)/ but at the same time
sentences have errors and some have none. Find out which each of them (b)/ have to remember his duty also. (c)/ No
part of the sentence has an error. If there is no mistake, the error (d)
answer is ‘No error’. 4. The child (a)/ picked up a burned paper (b)/ from the
1. Many a student (a)/ are frustrated (b)/ because of street. (c)/ No error (d)
unemployment. (c)/ No error (d) 5. Never did this exhibition of contempt and anger ceased
2. It is true (a)/ that the poor is unable (b)/ to get nourishing (a)/ so long as British rule remained (b)/ a live thing in
food even today. (c)/ No error (d) India. (c)/ No error (d)
6. The teacher was angry (a)/ when he found that (b)/ you employees (b)/ to be reinstated, has been accepted by
are not there. (c)/ No error (d) the management. (c)/ No error (d)
7. I would like you to (a)/ complete this assignment (b) 22. When he went out (a)/ he left the radio on (b)/ so that his
before you will leave for Mumbai. (c)/ No error (d) parents shall think that he was still in the house. (c)/ No
8. Ordinarily, when in difficulty (a)/ Ruchi prefers keeping error (d)
her counsel (b)/ than running about taking advice. (c)/ 23. The foremost criterion of selection we adopted (a)/ were
No error (d) the number of years of training (b)/ a dancer had received
9. One of the drawbacks (a)/ of modern education are (b)/ that under a particular guru. (c)/ No error (d)
it does not encourage original thinking. (c)/ No error (d) 24. I can’t understand why (a)/ he did not told me (b)/ the
10. I enquired of him (a)/ why he is so negligent (b)/ in his reason for his absence. (c)/ No error (d).
studies. (c)/ No error (d) 25. He very promptly (a)/ withdraw the remarks (b)/ made by
11. There was no doubt (a)/ that the majority of workers (b)/ him on my office note. (c)/ No error (d)
is in favour of the proposal. (c)/ No error (d) 26. He would not return the money (a)/ that he borrowed,
12. He laid unconscious for half an hour (a)/ until he was (b)/ will he? (c)/ No error (d)
seen (b)/ by a passing motorist. (c)/ No error (d) 27. None of my friends (a)/ are applying (b)/ for this job. (c)/
13. None of my friends (a)/ are applying (b)/ for this job. (c)/ No error (d)
No error (d) 28. Bread and butter (a)/ is (b)/ a wholesome food. (c)/ No
14. Rohan was leading (a)/ a happy and leisurely (b)/ life error (d)

K
after his retirement from service. (c)/ No error (d) 29. The future is (a)/ yet to come but you have a (b)/ right to
15. When I went to Maniram’s house, (a)/ his mother told shape it. (c)/ No error (d)
me (b)/ that he was gone out of the country. (c)/ No error 30. I finished my work (a)/ before we reached his place (b)/
(d) with great difficulty because of heavy traffic. (c)/ No
16. One of his many (a)/ good traits that (b)/ come to my error (d)
mind was his modesty. (c)/ No error (d) 31. If I knew (a)/ that my friend had planned to visit the town
17. When the dentist came in (a)/ my tooth was stopped today, (b)/ I would have made his stay comfortable. (c)/
aching (b)/ out of fear that I might lose my tooth. (c) No No error (d)
error (d) 32. More widely popular (a)/ than the hunting of deer or fox
18. The parents scolded the child (a)/ for having broke the (b)/ were the pursuit of the hare. (c)/ No error (d)

KUNDAN
window (b)/ but their words fell (c)/ on deaf ears. (d)/ No 33. The Committee Chief warned the party members (a)/ that
error (e) if they persist in their obstructionist attitude (b)/ they
19. Two-thirds of the book (a)/ were (b)/ rubbish. (c)/ No would be suspended. (c)/ No error (d)
error (d) 34. If the manager acted in time, (a)/ the strike would (b)/ not
20. The chief idea of (a)/ very common type of traveller (b)/ have lasted long. (c)/ No error (d).
is to see as many objects of interest as he possibly could. 35. On hearing the news over the radio that (a)/ India has
(c)/ No error (d) won the hockey match, (b)/ I jumped with joy. (c)/ No
21. The demand of the workers’s union (a)/ that the dismissed error (d)

Answers and Explanations


1. b; Replace ‘are’ with ‘is’ 11. c; Replace ‘is’ with ‘was’
‘Many a’ takes singular subject and singular verb. 12. a; Replace ‘laid’ with ‘lay’
2. b; Replace ‘is’ with ‘are’ 13. b; Replace ‘are’ with ‘is’
Here, ‘poor’ stands for ‘poor people’. 14. a; Replace ‘was leading’ with ‘led’
3. c; Replace ‘have’ with ‘has’ 15. d; Replace ‘was’ with ‘had’
4. b; Replace ‘burned’ with ‘burnt’ 16. d; Replace ‘was’ with ‘is’
5. a; Replace ‘ceased’ with ‘cease’ 17. b; Remove ‘was’
6. c; Replace ‘are’ with ‘were’ 18. b; Replace ‘broke’ with ‘broken’
7. c; Remove ‘will’ 19. b; Replace ‘were’ with ‘was’
8. c; Replace “than” with “to” 20. c; Replace ‘could’ with ‘can’
9. b; Replace ‘are’ with ‘is’ 21. c; Remove ‘to’
10. b; Replace ‘is’ with ‘was’ 22. d; No error
23. b; Replace ‘were’ with ‘was’ 29. a; Replace ‘is’ with ‘has’
24. b; Replace ‘told’ with ‘tell’ 30. a; Replace ‘finished’ with ‘had finished’
25. b; Replace ‘withdraw’ with ‘withdrew’ 31. d; No error
26. c; Replace ‘will’ with ‘would’ 32. c; Replace ‘were’ with ‘was’
27. b; Replace ‘are’ with ‘is’ 33. b; Replace ‘persist’ with ‘persisted’
28. d; No error 34. a; Replace ‘acted’ with ‘had’ acted
35. b; Replace ‘has’ with ‘had’

Adjective
An adjective is a word used with a noun ( or a pronoun) to Rule: When a number together with a unit of measurement
add something for its meaning. The adjective may be used is to be used as an adjective, it is a compound word
(i) attributively: when it is used along with the noun as and the unit of measurement is taken in the singular.
an epithet or attribute; for example, Incorrect : It is a twenty-kilometres walk.
There were dark clouds in the sky. Correct : It is a twenty-kilometre walk.

K
(ii) predicatively : when it forms part of the predicate; Rule: Definite Numeral Adjective always takes plural
for example. countable noun.
The clouds in the sky were dark. Incorrect: When I was in Punjab, I had twenty
Rule : The adjective is correctly used with a verb when some money.
quality of the subject rather than of the action of the Correct: When I was in Mumbai, I had twenty rupees.
verb, is to be expressed. Rule: If in a sentence one noun is used for all ordinals and
Incorrect: Flowers were plucked freshly. the first ordinal has 'the' while, other not, the noun
Correct: Flowers were plucked fresh. will be in the plural.
Rule: Due to and prior to should be used as predicative Incorrect: I have read the literature of the seventeenth
adjectives. When prepositional functions are to be and eighteenth century.

KUNDAN
performed, use because of in place of due to, and Correct: I have read the literature of the seventeenth
before in place of prior to. and eighteenth centuries.
Incorrect : Wickets fell due to superb bowling. (ii) Indefinite Numeral Adjective : It does not denote an
Correct : Wickets fell because of superb bowling. exact number. For example.
There are many petitions lying on the tables.
Kinds of Adjective Rule : Many takes plural noun while "Many a/an singular
1. Adjective of Quality : Also known as Descriptive noun.
Adjective, it shows the kind of quality of a person or a Incorrect: He has observed many an uncommon
thing. For example, phenomena.
He loves her soft skin. Correct: He has observed many an uncommon
2. Adjective of Quantity : It is an adjective which shows phenomenon.
how much of a thing is meant. For example, Rule : The few is used before plural countable noun and the
There is little sense in it. little before uncountable noun. Both follow a clause.
3. Adjective of Number : Also known as Numeral Adjective, Incorrect: I gave the beggar a little coins I had.
it shows how many persons or things are meant, or in Correct: I gave the beggar the few coins I had.
what order a person or a thing stands. For example, (iii) Distributive Numeral Adjective : It refers to each one of
The face of a clock has sixty divisions. a number. Distributive numeral adjectives are the same
Numeral Adjectives are of three kinds: as distributive pronouns (see Pronoun). They only differ
(i) Definite Numeral Adjective: It denotes an exact number. in function. For example,
It may be a cardinal denoting how many; as, one, two, Each time he missed the bus.
three, etc. It may be an ordinal denoting the order, as 4. Demonstrative Adjective : It points out which person or
first, second, third etc. thing is meant. For example,
This table is made of wood.
Rule : The plural forms these and those are often wrongly Correct: Shikha is the better of the two sisters.
used with the singular nouns kind and sort. Rule: Preferable takes to and is not preceded by more.
Incorrect : These kind of clothes are not acceptable. Incorrect: Coffee is more preferable than tea in Brazil.
Correct : This kind of clothes are not acceptable. Correct: Coffee is preferable to tea in Brazil.
5. Interrogative Adjective: When an interrogative pronoun Rule: When two adjectives in different degrees of
is used with a noun to ask a question, it is called an comparison are used in the same sentence, each
interrogative adjective. For example, should be complete in itself.
What options are available after graduation? Incorrect: My father is as rich if not richer than your
6. Exclamatory adjective: It is used with a noun in mother.
exclamatory sentence. For example, Correct: My father is as rich as if not richer than
What a show! your mother.

Confusing Words Rule: If two adjectives are connected by and, they should
be in the same degree throughout.
Later, latter, latest, last: Later and latest refer to time. Incorrect: Raj is more intelligent and most active.
Latter and last refer to position. Correct: Raj is more intelligent and more active.
Incorrect: The later part of the novel is not neatly written. Rule: The comparative in -er is not used when we compare
Correct: The latter part of the novel is not neatly written. two qualities of the same person or thing.

K
Incorrect: What is the last news? Incorrect : Mohan is wiser than strong.
Correct: What is the latest news? Correct : Mohan is more wise than strong.
Elder, older, eldest, oldest: Elder and eldest are used of Rule: When two objects are compared with each other, the
members of the same family. Besides, both of them are latter term of comparison must exclude the former.
used only attributively (that is, before the noun). Older Incorrect : Satish is cleverer than any student in his
and Oldest are used in the general sense. class.
Incorrect : I have an older brother. Correct : Satish is cleverer than any other student
Correct : I have an elder brother.. (Since brother is a in his class.
member of the same family) Rule: The comparative degree is generally followed by than
Incorrect : Mohan is the eldest boy in the town. but the following comparative adjectives are followed

KUNDAN
In a town there may be so many families. We are talking by the preposition to : inferior, superior, prior, anterior,
of boys in general sense. So eldest should be replaced posterior, senior, junior.
by oldest. Incorrect: India's fielding is inferior than South
Correct : Mohan is the oldest boy in the town. Africa's.
Correct: India's fielding is inferior to South Africa's.
Degree of Comparison Rule: Adjectives expressing qualities that do not admit of
different degrees cannot be compared.
Degrees are of three kinds. Incorrect: Our field is more square.
1. Positive Degree Correct: Our field is square.
2. Comparative Degree
3. Superlative Degree Rule: Double comparatives and superlatives are to be
avoided.
Rule: When a selection is made out of the two, and two is
Incorrect: Zarda mangoes are more sweeter than
given in the sentence, the is used before the adjective
Maldah ones.
denoting comparison.
Correct: Zarda mangoes are sweeter than Maldah
Incorrect: Shikha is better of the two sisters.
ones.

EXERCISE
Directions: In the following questions, some of the 1. Firstly you should a)/ think over the meaning of the
sentences have errors and some have none. Find out which words b)/ and then use them. c)/ No error d)
part of the sentence has an error. The letter corresponding 2. The driver tried his best a)/ to avert the accident by b)/
to that part is your answer. If there is no mistake, the answer bringing the car to a suddenly stop. c)/ No error d)
is ‘No error’. 3. The Sunrise Hotel was a)/ fully equipped to offer b)/
leisure stay c)/ to its clients. d)/ No error e)
4. The technician reminded a)/ them to have a thoroughly the statue of the one who had dedicated b)/ his life to the
cleaning of the b)/ machine after each use. c)/ No error d) uplift of the poor. c)/ No error d)
5. I am much glad a)/ that you have b)/ won the prize. c)/ No 21. The car flew off the road a)/ and fell into the valley b)/
error d) because Ashish was driving faster. c)/ No error d)
6. My observation is that a)/ between Vivek and Shashi, 22. He is very blind a)/ to read b)/ smallest of prints. c)/ No
b)/ Vivek is the most intelligent. c)/ No error d) error d)
7. It very often a)/ happens that b)/ a man who talks most 23. What is worst, a)/ this distortion cannot be corrected b)/
does little. c)/ No error d) by either contact lens or glasses. c)/ No error d)
8. From all accounts a)/ I learn that b)/ he is the best and 24. Inspite of working a)/ very neat and careful b)/ he could
honest member of the new Cabinet. c)/ No error d) not win even the third prize. c)/ No error d)
9. He is a)/ too intelligent b)/ to make a mistake. c)/ No error 25. Sohanlal is a)/ the richest b)/ of the two. c)/ No error d)
d) 26. On hearing the news, a)/ he went directly b)/ to the
10. The flood situation this year a)/ is worst than b)/ that Manager’s room. c)/ No error d)
prevailed in the last year, c)/ No error d) 27. Even though it was a)/ raining bad b)/ I went out to get
11. People invent new machines a)/ when they b)/ think some medicines. c)/ No error d)
independent. c)/ No error d) 28. The Headmaster said that a)/ Sachin was capable of doing
12. Geometry and Drawing a)/ are more easier than b)/ b)/ more better work. c)/ No error d)
Geography and Social Studies. c)/ No error d) 29. The night was bright, a)/ the clouds dispersed, b)/ and

K
13. Some people get a)/ used to changes b)/ very easily than the stars were shining brightly in the sky. c)/ No error d)
others do. c)/ No error d) 30. It is the duty of every citizen to do his utmost a)/ to
14. My uncle forbade me a)/ not to go through b)/ the defend the hardly-won b)/ freedom of the country. c)/
contents of his letter. c)/ No error d) No error d)
15. This road is the worst a)/ than any other road in Delhi b)/ 31. Of the two a)/ he was trying the hardest b)/ to attain the
but remains unrepaired. c)/ No error d) highest percentage. c)/ No error d)
16. Of all the friends a)/ I have had, he is the most helpful b)/ 32. Early rising a)/ brings out the better b)/ in a story writer.
and less arrogant. c)/ No error d) c)/ No error d)
17. A man entered the tavern a)/ and asked for some bread 33. At the end of the Second World War a) the USA was
and cheese b)/ with a decided foreign accent. c)/ No stronger b)/ than any country in the world. c)/ No error
error d) d)

KUNDAN
18. Our new playground a)/ is big and cleaner b)/ than theirs. 34. Being the only people there a)/ their presence was b)/
c)/ No error d) most important. c)/ No error d)
19. Watch how careful a)/ the sparrow knits the b)/ straws 35. He can play a)/ as good as b)/ he can write with his left
into one another to form a nest. c)/ No error d) hand. c)/ No error d)
20. It is rather ironically that so much is being spent a)/ on

Answers and Explanations


1. a; Replace ‘Firstly’ with ‘First’ 14. b; Remove ‘not’
2. c; Replace ‘suddenly’ with ‘sudden’ ‘Forbade’ has a negative meaning and double
3. c; Replace ‘leisure’ with ‘leisurely’ negatives are not used.
4. b; Replace ‘thoroughly’ with ‘thorough’ 15. a; Replace ‘the worst’ with ‘worse’
5. a; Replace ‘much’ with ‘very’ 16. c; Replace ‘less’ with ‘least’
6. c; Replace ‘the most’ with ‘more’ 17. c; Replace ‘decided’ with ‘decidedly’
7. c; Replace ‘most’ by ‘much’ 18. b; Replace ‘big’ with ‘bigger’
8. c; The correct form is ‘he is the best and most honest When two qualities are compared, the two adjectives
member....’ should be of the same degree.
9. d; No error 19. a; Replace ‘careful’ with ‘carefully’
10. b; Replace ‘worst’ with ‘worse’ 20. a; Replace ‘ironically’ with ‘ironical’
11. c; Replace ‘independent’ with ‘independently’ 21. c; Replace ‘faster’ with ‘fast’
12. b; Remove ‘more’ No comparison has been made here and so positive
Double comparatives are not used degree should be used.
13. c; Replace ‘very’ with ‘more’ 22. a; Replace ‘very’ with ‘too’
23. a; Replace ‘worst’ with ‘worse’ 29. c; Replace ‘brightly’ with ‘bright’
24. b; The correct form is ‘very neatly and carefully’ 30. b; Replace ‘hardly’ with ‘hard’
25. b; Replace ‘richest’ by ‘richer’ 31. b; Replace ‘the hardest’ with ‘harder’
When a comparison is made between two, 32. c; Replace ‘better’ with ‘best’
comparative degree is used. 33. c; Replace ‘any country’ with ‘any other country’
26. b; Replace ‘directly’ with ‘direct’ 34. c; Remove ‘most’
27. b; Replace ‘bad’ with ‘badly’ 35. b; Replace ‘good’ with ‘well’
28. c; Remove “more”

Adverb
An adverb is a word which modifies the meaning of a verb, an Incorrect : He went only that far.
adjective or an adverb. For example Correct : He went only so far.
Rama runs quickly. Rule : The adverb “too” should not be used instead of “very”
Govinda reads quite clearly.

K
or “much” and vice versa.
Incorrect : She is too intelligent.
Kinds of Adverb
Correct : she is very intelligent.
Rule : Much is used with past participles used as adjectives
(a) According to meaning not with present participles.
1) Adverbs of Time : They are the adverbs which tell us Incorrect : The match became much interesting.
when an action takes place. For example, Correct : The match became very interesting.
I shall meet you tomorrow. Rule : Adverb of degree comes just before the adjective it
Rule : Time adverbs cannot be used in the present perfect, qualifies.
instead the past indefinite is used for them. Incorrect : He too is weak to run.

KUNDAN
Incorrect : I have seen a 3D movie last night. Correct : He is too weak to run.
Correct : I saw a 3D movie last night. They are so honest that they cannot tell a lie.
2) Adverbs of Place : They are the adverbs which tell us 6) Adverbs of Affirmation and Negation : They are the
where an action takes place. For example, adverbs which tell us whether an action is done or
He left his bag there. not. For example,
3) Adverbs of Manner : They are the adverbs which tell us He has not left the place.
how an action takes place or in what manner. For example, 7) Adverbs of Reason : They are the adverbs which tell
Gautam Buddha left his family stealthily. us why an action takes place. For example,
4) Adverbs of Frequency : They are the adverbs which tell She therefore decided to be a rich woman.
us how often an action takes place. For example,
The Delhi Police is always with you . (b) According to use
5) Adverbs of Degree or Quantity : They are the adverbs 1) Simple Adverb : It only modifies some words (verb,
which tell us how much or in what degree or to what adjective or an adverb). All the above examples belong
extent. For example, to this category.
I have pleaded enough and now I give up. 2) Interrogative Adverb : It not only modifies some words
Rule : The adverb “so” as an adverb of degree must not be but also introduces a question. For example,
used absolutely. How did you come here?
Incorrect : He is so rich. 3) Relative Adverb : It not only modifies some words but
Correct : He is very rich. also refers back to some antecedent. For example,
Rule : That “should not be used instead of ” so “as an How can I forget the day when happiness was
adverb”. showered on me from all sides?
Forms of adverb Rule : When there are two or more adverbs after a verb
(and its object), the normal order is; adverb of
Rule : Adjective should not be used for adverb. manner, adverb of place, adverb of time.
Incorrect: The moon shone bright in the sky. Incorrect: I read the proof yesterday meticulously at
Correct: The moon shone brightly in the sky. home.
Correct: I read the proof meticulously at home yesterday.
Rule : Some adverbs have two forms, the form ending in-ly
and the form which is the same as adjective. For Rule : Adverbs of frequency and most of the adverbs of
example, loud, quick, high, hard, near, late, pretty. quantity are
Incorrect: He fought hardly for the No. 1 spot. i) normally put between the subject and the verb if the
Correct: He fought hard for the No. 1 spot. verb consists of only one word;
Similarly, Incorrect: It would hard rain today. ii) put after the first word if there is more than one word
in the verb;
Correct : It would hardly rain today.
iii) put after the verb if the verb is am/are/is/was/were;

Comparison of adverbs
Some more rules
Rule : If the adverb is of one syllable, the comparative is
formed by adding-er and the superlative by adding- Rule : Two negatives should not be used in the same

K
est to the positive. sentence unless we wish to make an affirmation.
Incorrect : The hare ran more fast than the dog. Incorrect: she did not despise none of her lovers.
Correct: She did not despise any of her lovers.
Correct : The hare ran faster than the dog.
Rule : Adverbs ending in -ly form the comparative by Rule : Ever should not be used for never.
adding more and the superlative by adding most. Incorrect: She seldom or ever has sweet talks with
her son.
Incorrect : The petrol price hike issue has been
Correct: She seldom or never has sweet talks with
debated hotlier than the urea scam.
her son.
Correct : The petrol price hike issue has been
debated more hotly than the urea scam. Rule : Else should be followed by but.
Incorrect: It is nothing else than fatigue.

KUNDAN
Correct: It is nothing else but fatigue.
Position of adverbs
Rule : Never should not be used for not.
Rule : Adverbs of manner, place and time are generally Incorrect: She met her friend after a long time and
placed after the verb or after the object if there is her friend never recognised her.
one. Correct: She met her friend after a long time and her
Incorrect: He wrote last year a book. friend did not recognise her.
Correct: He wrote a book last year.

EXERCISE

Directions: Find out the error in each of the following 6. The observers feel that (a)/ the stronger team has to face
sentences, if any. If there is no error, the answer is ‘d’. defeat (b)/ because the players don’t play whole hearted.
1. She behaved in a cowardly manner (a)/ before the (c)/ No error (d)
headmaster (b)/ and hesitated to do the work. (c)/ No 7. Although he is usually (a)/ rude with everyone, (b)/ he
error (d) behaved nice with all of us today. (c)/ No error (d)
2. The girl refused (a)/ not to (b)/ return the articles. (c)/ No 8. He ran so fastly (a)/ that he reached first and (b)/ realised
error (d) that he had (c)/ not earned anything substantial. (d)/ No
3. The real important thing (a)/ of our life is our livelihood (b)/ error (e)
which discriminates us from animals. (c)/ No error (d) 9. In spite of toiling (a)/ very hardly he (b)/ realised that he
4. She is too much beautiful (a)/ so most of the boys (b)/ run had not earned anything substantial. (c)/ No error (d)
after her and want to influence her. (c)/ No error (d) 10. We were very much (a)/ carefully in our approach (b)/
5. The taxi driver who had come (a)/ to receive us at (b)/ the and hence we would complete the complicated task. (c)
airport was speaking fluently French. (c)/ No error (d) No error (d)
11. Hard had he (a)/ thrown the ball (b)/ when it fell on the 23. He was looking impatient (a)/ at the visitor (b)/ who
ground. (c)/ No error (d) showed no signs of leaving the room. (c)/ No error (d)
12. We are confident (a)/ enough to earn (b)/ our livelihood 24. The rabbit runs (a)/ enough fast (b)/ to win the race. (c)/
by toiling hardly. (c)/ No error (d) No error (d)
13. All said and done (a)/ it is the basic values that (b)/ are 25. Your scooter would not (a)/ have given you trouble (b)/
most importance and help you in the long run. (c)/ No if you had maintained it proper. (c)/ No error (d)
error (d) 26. Ajay is the most (a) succeeded of all the businessmen
14. She was so (a)/ emotional stable that (b)/ she was not (b)/ in this city today (c)/ No error (d)
moved by their decision to suspend her. (c)/ No error (d) 27. I was delighted (a)/ to see him (b)/ full recovered. (c)/ No
15. He is sure (a)/ to succeed because (b)/ he is used to error (d)
working very hardly. (c)/ No error (d) 28. The old man continued (a)/ living an independently life
16. He has been working with me (a)/ for the last ten years (b) and never asked anyone for help. (c)/ No error (d)
(b)/ but I cannot claim that I have understood him 29. All of them worked (a)/ very careful (b)/ from the
thorough. (c)/ No error (d) beginning till the finish. (c)/ No error (d)
17. I saw a (a)/suddenly change in the weather (b)/ last night. 30. As the accountant (a)/ came lately (b)/ he was punished
(c)/ No error (d) by the boss. (c)/ No error (d)
18. Some people get (a)/ used to change (b)/ very easily 31. He will (a) have to pay (b)/ dear for his foolishness (c)/
than others do. (c)/ No error (d) No error (d)

K
19. Although it was (a)/ raining bad (b)/ he went out. (c)/ No 32. As he was running fastly (a)/ he was knocked down (b)/
error (d) by a speeding car. (c)/ No error (d)
20. The worker was (a)/ paid poor (b)/ for his dedication and 33. The boy spoke out straightly (a)/ to the principal (b)/ to
hard work. (c)/ No error (d) prove his innocence (c)/ No error (d)
21. Watch how careful (a)/ the sparrows knit (b)/ the straws 34. Rahul wants to stand first (a)/ in the class and so (b) he
into one another. (c)/ No error (d) works hardly (c)/ No error (d)
22. Shekhar comes usually (a)/ to my house (b)/ on Sunday. 35. All the flowers (a)/ in the garden (b)/ smell sweetly. (c)/
(c)/ No error (d) No error (d)

Answers and Explanations


1. d;
2. b;
3. a;
4. a;
5. c;
6. c;
7. c;
8. a;
KUNDAN
No error
Omit the word “not”
Replace “real” with “really”
Replace “too much” with “very”
Replace “fluently French” with “French fluently”
Replace “whole-hearted” with “whole-heartedly”
Replace “nice” with “nicely”
Replace “fastly” with “fast”
19. b;
20. b;
21. a;
22. a;
23. a;
24. b;
25. c;
26. b;
Replace “bad” with “badly”
Replace “poor” with “poorly”
Replace “careful” with “carefully”
Put the word “usually” before “comes”
Replace “impatient” with “impatiently”
Replace “enough fast” with “fast enough”
Replace “proper” with “properly”
Replace “succeeded” with “successful”
9. b; Replace “hardly” with “hard” 27. c; Replace “full” with “fully”
10. b; Replace “carefully” with “careful” 28. b; Replace “independently” with “independent”
11. a; Replace “hard” with “hardly” 29. b; Replace “careful” with “carefully”
12. d; Replace “hardly” with “hard” 30. b; Replace “lately” with “late”
13. c; Replace “importance” with “important” 31. c; Replace “dearly” with “dear”
14. b; Replace “emotional” with “emotionally” 32. a; Replace “fastly” with “fast”
15. c; Replace “hardly” with “hard” 33. a; Replace “straightly” with “straight”
16. c; Replace “thorough” with “thoroughly” 34. c; Replace “hardly” with “hard”
17. b; Replace “suddenly” with “sudden” 35. c; Replace “sweetly” with “sweet”
18. c; Replace “very” with “more”
Time and Tense
Simple Present speaking, but to a phenomenon which occurs regularly; as
in.
To express a habitual action. It rains heavily in equatorial regions and hilly areas.
Incorrect : He is taking a walk every morning. To express a temporary action which may not be actually
It is clear that the action (taking a walk) happens only every happening at the time of speaking.
morning and not continuously. While describing such ac- For example; If you are a teacher but work on a project tempo-
tions as happen at regular invervals or routinely we use the rarily, you say
simple present. So, I am working on a project.
Correct : He takes a walk every morning. This you may say in a conversation even though you are not
working at the moment of speaking. You cannot say
To express general truths.
I work on a project.
Incorrect : Two and two are making four.
because this would give the false impression that you are a
The above incorrect sentence gives the impression, that such
project worker by profession. As things stand in the given
a thing is happening in a particular case. The truth is it hap-

K
circumstances, the appropriate statements would be
pens always. It is a general truth. So,
I teach.
Correct : Two and two make four.
I am working on a project.
In exclamatory sentences beginning with here and there to
express what is actually taking place in the present. To express an action that is planned or arranged to take
Incorrect : Here is coming the bus ! place in the near future. For example,
It is true that the bus is actually coming. But, bearing the He is going to the city tomorrow.
above rule in mind, But it is not good to use the present continuous for slightly
Correct : Here comes the bus! distant future. So, don’t say
He is going to the city next week.
To indicate a future event that is part of a plan or an Rather, use the simple present. So, it is better if you say
arrangement. For example,

KUNDAN
He goes to the city next week.
The Indian team goes to the Caribbean island next month.
Note: It is not, however, incorrect if one uses will go in place When the reference is to a particularly obstinate habit, the
of goes in the above sentence. present continuous is used instead of simple present. An
adverb like always, continually, constantly is also used.
To introduce quotations. Incorrect : It is no use scolding him; he always
For example, does what is forbidden.
Pope says “A little knowledge is a dangerous thing”. Note that his doing what is forbidden has become a die-hard.
Note: It is also correct if said is used in place of says. The habit persists inspite of advice or warning. So, we should
use the present continuous.
When two actions of the future are being talked about, one
Correct : It is no use scolding him; he is always
dependant on the other, the former action is denoted by
doing what is forbidden.
simple present and the latter by simple future.
For example, The following verbs are normally used in the simple present
We shall go when the child comes back home. instead of the present continuous.
a) verbs of appearing - appear, look , seem.
Present Continuous b) verbs of emotion - want, wish, desire, feel, like, love,
hate, hope, prefer, refuse.
To express an action going on at the time of speaking. c) verbs of perception - see, hear, smell, notice, recognise.
For example, if you are sitting indoors, and you lament not d) verbs of possession - belong to, consist of, contain,
being able to go out because rain continues to fall heavily, have, own, possess.
you say. e) verbs of thinking - agree, believe, consider, forget,
It is raining heavily. imagine, know, mean, mind, remember, suppose, think,
If you say trust, understand.
It rains heavily. f) the verb ‘be’
You are not referring to the falling of the rain at the moment of Incorrect : The allegation is appearing to be true.
Since appear here is being used in the sense of look or seem, Note: There are certain verbs which are used in the present
the above usage is incorrect. perfect form even though they function as present perfect
Correct : The allegation appears to be true. continuous for such verbs, referred to the list given at the
But it is correct to say that end of present continuous. (There we saw how those verbs
He is appearing on the stage for the first time. are used in the simple present form even though they carry
because appear here has a meaning different from that the present continuous sense.)
mentioned in the list. It does not mean look or seem. It means Incorrect : I have been knowing him for a long time.
present oneself formally or publicly. As per our discussion above,
Correct : I have known him for a long time.
Present Perfect
Simple Past
To indicate completed activities in the immediate past.
Incorrect : He just came. To indicate an action completed in the past.
The usage of just tells us that the activity of coming has I met her last year.
been completed in the immediate past. So, we use the present For past habits.
perfect instead of the past simple. I visited Jaipur every year.
Correct : He has just come.
After “it is time”.
To express past actions whose time is not given and not It is time we started working.

K
definite.
Incorrect : I wrote three books. Past Continuous
Then the reader would infer that you wrote books in the
past as a profession or hobby. But when you are being so To denote an action going on at some time in the past.
specific as to say “ three books”, we immediately feel the Incorrect : When I saw her, she wrote a poem with
need of a time frame. Since no time frame is mentioned, we full attention.
assume it to be by now (not definite). So, we have something The two-clause sentence obviously gives us the clue
to the effect. that she was doing (past continuous) something when I saw
I have written three books by now. her. So,
This by now is implied and need not be written. So, Correct : When I saw her, she was writing a poem

KUNDAN
Correct : I have written three books. with full attention.
The present perfect is never used with adverbs of past time. For persistent habits in the past.
In such cases the simple past should be used. Incorrect : She always chewed gum.
Incorrect ; India has won the match last week. The use of always tells us that the habit is persistent, or
Now, last week is immediate past. You may therefore be rather was persistent because chewed suggests that it is the
tempted to use the present perfect. But remember that the past, not the present, that is being talked about.
immediate past here does not go un-indicated. Last week in Correct : She was always chewing gum.
being used as an adverb of past time. So,
Correct : India won the match last week. Past Perfect
To describe an action completed before a certain moment in
Present Perfect Continuous the past.
To express an action which began at some time in the past Incorrect : He wrote a novel even before he was 10 years
and is still continuing. old.
Incorrect : They have built the road since March last. Being 10 years old is a moment in the past. The writing of the
Have built is present perfect and implies that the action of novel had been completed (past perfect) before that point of
building is complete. Note, however, that the sentence only time. So,
mentions when the work began. If it were suggested that the Correct : He had written a novel even before he was 10
work was finished at a certain time, say May last, things years old.
would have been different.We would then say If two actions happen in the past, the earlier one is denoted
They built the road between March and May last. In the by past perfect while the later one is denoted by simple past.
given sentence, however, we are not told that the work has For example, you deposited money in the bank. An hour later
ended. So, we assume it is going on. Then your friend came to you to borrow some money. You then
Correct : They have been building the road since March say: I had deposited money in the bank when my friend came
last. to borrow from me.
Past Perfect Continuous We often have to deal with sentences where the tense
of the subordinate clause has to be seen in relation with the
To express an action that began before a certain point in the tense of the principal clause. The following rules should then
past and continued upto that time. be observed.
Incorrect : Kapil Dev had played for more than a decade 1. A past tense in the principal clause should be followed
when Tendulkar entered the Indian team. by a past tense in the subordinate clause.
Note that here we do not have a case where both the actions Incorrect : He hinted that he wants money.
had been completed in the past. Kapil had not bidden good- Correct : He hinted that he wanted money.
bye to cricket then. He was still playing. So, Incorrect : He replied that he will come.
Correct : Kapil Dev had been playing for more than a Correct : He replied that he would come.
decade when Tendulkar entered the Indian Incorrect : I never thought that I shall see him
team. again.
Correct : I never thought that I should see him
Simple Future again.
However,
To express an action that has still to take place. i) when the subordinate clause in the above case expresses
For example, He will play cricket tomorrow. a universal truth, it is in the present tense.
Incorrect : Newton discovered that the apple fell
Future Continuous

K
due to gravity.
Correct : Newton discovered that the apple falls
To express an action as going on at some time in future.
due to gravity.
When I reach there, he will be reading a book.
Incorrect : He said that honesty was the best
The above sentence means that the action of reading will be policy.
going on at that moment in future when I make my entry. Now Correct : He said that honesty is the best policy.
mark its difference from the following: ii) when the subordinate clause is introduced by than in the
When I reach there, he will read a book. above case, it may be in the present tense. This implies that
Here his action of reading is dependant on my reaching there. in this case, even if the subordinate clause is in the past
He will begin reading only when I reach there. tense, it will not be considered incorrect. Both tense are
correct.

KUNDAN
To express future events that are planned.
Correct : He liked you better than he liked me.
For example,
Or Correct : He liked you better than he likes me.
He will be coming here for Durga Puja.
Correct : I saw him often than I saw my children.
Or Correct : I saw him often than I see my children.
Future Perfect 2. A present or future tense in the principal clause may be
To indicate the completion of an action by a certain future followed by any tense in the subordinate clause, as required
time. by the sentence. Thus all of the given sentences are correct.
I shall have read the books when you come next. He thinks that she is there.
He thinks that she was there.
Future Perfect Continuous He thinks that she will be there.
He will think that she is there.
To indicate an action represented as being in progress over He will think that she was there.
a period of time that will end in the future. He will think that she will be there.
She will have been teaching for six years when she gets
married.

EXERCISE
Directions: In each of the following sentences, find out 1. He drank once again a)/ as he was b)/ feeling thirsty. c)/
which part of the sentence has an error. The error may be No error d)
idiomatic or grammatical. If there is no mistake, the answer 2. With a view to help the Third World countries, a)/ the
is ‘No error’ I.M.F. and the World Bank keep sending b)/ their experts
to different regions of the world. c)/ No error d)
3. One of these days a)/ I will be going to b)/ confront you 18. You and I a)/ know each other b)/ for the last six years.
with incontrovertible evidence. c)/ No error d) c)/ No error d)
4. It is necessary a)/ that everybody b)/ must have a house. 19. The eminent speaker’s speech a)/ was broadcasted over
c)/ No error d) b)/ all the major radio stations. c)/ No error d)
5. This small table a)/ will collapse b)/ if you will stand on it. 20. Somewhere along the line a)/ I lost track of b)/ what
c)/ No error d) Ashish said about heart ailments. c)/ No error d)
6. Shobha asked the dealer a)/ what was the price b)/ of 21. When your father inquired a)/ about your marks b)/ you
that bicycle and whether it is really made in Germany? c)/ lied to him. Have you not? c)/ No error d)
No error d) 22. I am sure that a)/ neither the flat nor its contents b)/ is for
7. He refused to answer the question a)/ despite the fact sale. c)/ No error d)
b)/ that his silence will be interpreted as guilt. c)/ No 23. It’s time a)/ you started b)/ working. c)/ No error d)
error d) 24. You are learning Tamil a)/ for the last one year b)/ but you
8. Sadhana was the one person a)/ who could somehow show no improvement whatsoever. c)/ No error d)
manage b)/ to working in that section for a long time. c)/ 25. Many a profound thinker believe a)/ that the march of
No error d) civilisation b)/ has not coincided with real human
9. When we consider all the factors, which are many, a)/ progress. c)/ No error d)
the number of school dropouts b)/ are quite disturbing. 26. The last of the Mughal emperors of India a)/ was
c)/ No error d) imprisoned b)/ and was later sent into exile by the British.

K
10. The lecture to be held a)/ in the auditorium tomorrow b)/ c)/ No error d)
was open to the public and free of charge. c)/ No error d) 27. The programme which a)/ came on television b)/ these days
11. While proceeding on leave a)/ he had committed b)/ that in the evenings is very interesting. c)/ No error d)
he would be resumed after two days. c)/ No error d) 28. On reaching the railway station a)/ he was disappointed
12. Coffee seeds a)/ is dried and roasted b)/ before they are to learn b)/ that the train left. c)/ No error d)
ground into powder. c)/ No error d) 29. Neither India nor Pakistan a)/ have yet acquired b)/ the
13. If there had been a)/ no new thinking, b)/ life would capability to produce nuclear weapons. c)/ No error d)
follow the beaten track. c)/ No error d) 30. When I shall see him a)/ I shall tell him b)/ that you had
14. My father did not a)/ heard from my younger brother b)/ called. c)/ No error d)
who is abroad for over a month. c)/ No error d) 31. I courteously asked him a)/ where was he going b)/ but

KUNDAN
15. While going a)/ through the report b)/ yesterday I find he did not reply. c)/ No error d)
several factual mistakes. c)/ No error d) 32. Nobody believed him a)/ when he said that b)/ his son
16. The meeting adjourned abruptly a)/ by the Chairman was gone out of the country. c)/ No error d)
after b)/ about three hours of deliberation. c)/ No error d) 33. It is high time a)/ that we send b)/ the answer. c)/ No error d)
17. If the streets would have been clearly marked a)/ it would 34. I have had to work a)/ at the fountain for almost b)/ ten hours
not have taken us b)/ so long to find his house. c)/ No before it could start functioning well. c)/ No error d)
error d) 35. My body has long since exhausted all its energy. a)/ but
it went on running b)/ just the same. c)/ No error d)

Answers and Explanations


1. d; No error 13. a; Replace ‘had been’ with ‘was’
2. a; Replace ‘help’with ‘helping’ 14. a; Replace ‘did’ with ‘has’
3. b; Replace ‘will be’ with ‘am’ 15. c; Replace ‘find’ with ‘found’
4. c; Replace ‘must’ with ‘should’ 16. a; The correct form is: ‘The meeting was abruptly
5. c; Remove ‘will’ adjourned...’
6. c; Replace ‘is’ with ‘was’ 17. a; Replace ‘would have’ with ‘had’
7. c; Replace ‘will’ with ‘could’ 18. b; Replace ‘know’ with ‘have known’
8. c; Replace ‘working’ with ‘work’ 19. b; Replace ‘broadcasted’ with ‘broadcast’
9. c; Replace ‘are’ with ‘is’ 20. c; Replace ‘said’ with ‘was saying’
10. c; Replace ‘was’ with ‘is’ 21. c; Replace ‘Have’ with ‘Did’
11. c; Replace ‘be resumed’ with ‘resume’ 22. c; Replace ‘is’ with ‘are’
12. b; Replace ‘is’ with ‘are’ (‘seeds’ is plural) 23. b
24. a; Replace ‘are’ with ‘have been’ 30. a; Remove ‘shall’
25. a; Replace ‘believe’ with ‘believes’ 31. b; The correct form is ‘where he was going’
26. c; Remove ‘was’ 32. c; Replace ‘was’ with ‘had’
27. b; Replace ‘came’ with ‘comes’ 33. b; Replace ‘send’ with ‘sent’
28. c; Add ‘had’ before ‘left’ 34. e; No error
29. b; Replace ‘have’ with ‘has’ 35. b; Replace ‘went’ with ‘goes’

Active and Passive Voice


Voice is that form of a verb which shows whether what is number) with vermin.
denoted by the subject does something or has something She was touched with pity for the animals.
done to it. The poor man was beset (was surrounded) with
difficulties.
A verb is in the active voice when its form shows that the
The master was pleased with the servant.
person or thing denoted by the subject does something, that
The road was lined with people.
is, is the doer of the action. The subject acts.

K
The field was thronged (was crowded) with audience.
A verb is in the passive voice when its form shows that She was surprised at what her mother said.
something is done to the person or thing denoted by the
subject. The subject is not active but passive. It does not act, Rule: A few transitive verbs like eat, read,smell, taste etc,
but suffers or receives some action. even in an active form, are sometimes used in a passive
sense.
Consider the following sentences.
Incorrect: The mangoes are tasted sour.
1. The peon opened the gate.
2. The gate was opened by the peon. It is true that the mangoes are tasted. Mangoes do not taste.
Both the sentences have the same meaning. The difference, Yet the above sentence would imply that
however, lies in emphasis. In the first sentence, the peon The mangoes are tasted when they are

KUNDAN
becomes prominent. In the second sentence, the gate sour.
becomes prominent. So, the function of voice lies in the shifting (Like “Some vegetables are eaten raw.” means “Some
of prominence. vegetables are eaten in their raw state or when they are raw.”)
Sentence 1 is in the active voice because the subject But common knowledge tells us that people in general prefer
(the peon) is the doer of the action (opened). Note that the sweet mangoes. What the writer intends is the meaning
gate is the object here. The mangoes are sour when tasted.
Sentence 2 is in the passive voice because the subject This comes out in the following sentence.
(the gate) is not the doer of the action, but the receiver (was Correct: The mangoes taste sour.
opened). It does not have any object. Note that the object of
the verb becomes the subject in the passive voice. Thus only Incorrect: This fruit is smelt sweet.
transitive verbs can be used in the passive voice because an This would mean that this fruit is smelt when it is sweet. Do
intransitive verb has no object. you know of any such fruit bound by such convention?
What the writer intends to say is that this fruit is sweet when
Rule: The subject of the verb in the active voice comes after smelt. So,
the verb in the passive voice and is usually preceded by Correct: This fruit smells sweet.
the preposition by.
Incorrect: The mouse was killed of the cat. SYNTACTICAL CHANGES IN ACTIVE-PASSIVE
The cat would have been the subject of killed, the verb in the CONVERSION PROCESS
active voice. On the verb being converted to the passive A. TENSE
voice, the cat goes after the verb and is preceded by by. So, (i) Present Indefinite Tense
Correct: The mouse was killed by the cat. Active Voice: Subject + Verb + Object
However, there are a few notable exceptions: Passive Voice: Subject + Helping Verb (is/am/are)
The writer is enamoured (is inspired with love) of what + third form of verb (V3) + by + Object
he writes.
The entire place is infested (is spread over in large
Active Voice Passive Voice (vii) Future Indefinite Tense
(a) I love my children. My children are loved by me. Active Voice : Subject + Helping Verb (shall/will)
(b) They call him a liar. He is called a liar by them. + Verb + Object.
(ii) Present Continuous Tense Passive Voice : Subject + Helping Verb (shall/will) +
be + third form of Verb + by + Object
Active Voice: Subject + Helping Verb (is/am/are)
Active Voice Passive Voice
Verb + ing + Object
(a) He will give the news. The news will be given by
Passive Voice: Subject + Helping Verb (is/am/are)
him.
+ being + third form of Verb + by + Object (b)We will teach him. He will be taught by us.
Active Voice Passive
(a) He is writing a letter. A letter is being written by him. (viii) Future Perfect Tense
Active Voice: Subject + Helping Verb (will/shall) +
(b) The children are The poor are being helped
have + third form of Verb + Object
helping the poor. by the children. Passive Voice: Subject + Helping Verb (will/shall)
(iii) Present Perfect Tense + have + been + third form of Verb + by + Object
Active Voice: Subject + Helping Verb (has /have) Active Voice Passive Voice
+ third form of Verb + Object (a) They will have planted Trees will have been planted
trees. by them.
Passive Voice: Subject + Helping Verb (has/have + been)

K
(b)The teacher will have The students will have been
+ third form of verb + by + Object
called the students. called by the teacher.
Active Voice Passive Voice
(a) He has invited all his All his friends have (B) TWO OBJECTS OF THE VERB
friends. been invited by him. If a verb in the Active Voice takes two objects — a direct
(b)I have lost my watch. My watch has been lost. one and an indirect one after it, either of them may be
(iv) Past Indefinite Tense retained in the Passive such as,
Active Voice: Subject + second form of verb + Object (i) The direct object of the Active Verb:
Passive Voice: Subject + helping verb (was/were) Active Voice Passive Voice
+ third form of verb + by + Object (a) He teaches us Sanskrit. We are taught Sanskrit by
Active Voice Passive Voice him.

KUNDAN
(a) He bought a watch A watch was bought by him (b)He forgave me my fault. I was forgiven my fault by
yesterday. yesterday. him.
(b)They kept us waiting. We were kept waiting by (ii) The indirect object of the Active Verb:
them. Active Voice Passive Voice
(v) Past Continuous Tense (a) He teaches us Sanskrit. Sanskrit is taught to us by
Active Voice: Subject + Helping Verb (was/were) him.
+ Verb + ing + Object (b)He forgave me my fault. My fault was forgiven by
Passive Voice: Subject + Helping Verb (was/were) him.
+ being + third form of Verb + by + Object (C) SENTENCES BEGINNING WITH “LET”
Active Voice Passive Voice Active Voice Passive Voice
(a) Ram was writing A book was being written (a) Let Neha run a show. Let a show be run by Neha.
a book. by Ram. (b)Let me try this question.Let this question be tried by
(b)The horses were A cart was being drawn by me.
drawing a cart. the horses. (D) USE OF PREPOSITION
(vi) Past Perfect Tense (i) Certain intransitive verbs which take a preposition after
Active Voice: Subject +Helping Verb (had) + third form them when changed into the Passive Voice retain the
of Verb + Object preposition as part of the verb.
Passive Voice: Subject + Helping Verb (had been) Active Voice Passive Voice
+ third form of Verb + by + Object (a) He looks at the The painting is looked at by
Active Voice Passive Voice painting. him.
(a) She had finished the The work had been finished (b)The parents spoke to The children were spoken to
work. by her. their children. by the parents.
(b)They had invited the The students had been (ii) Sometimes, in the Passive Voice, a preposition other than
students. invited by them. “by” is used,
Active Voice Passive Voice (b)Can he do the work? Can the work be done by
(a) Rita knows me. I am known to Rita. him?
(b)It is time to send for It is time for the musician to (iii) If the question in the Active Voice begins with has/have/
the musician. be sent for. had, the form of the verb in the Passive Voice is:
(E) PASSIVE OF IMPERATIVE SENTENCES Has/Have/Had + been + Past Participle
(i) An imperative sentence in Passive Voice begins with the Active Voice Passive Voice
work “let”. The form of the verb is: (a) Had you seen the Had the boy been seen by
Let + be + past participle boy before? you before?
Active Voice Passive Voice (b)Has Priya invited you? Have you been invited by
(a) Shut the door. Let the door be shut. Priya?
(b)Tell the students to sit. Let the students be told to (iv) If the question in the Active Voice begins with what/
sit. where/why/when, the form of the verb in the Passive
(ii) We can use another method to change the sentences voice is:
into the Passive form. Question word + is/am/are/was/were + past participle
Active Voice Passive Voice of the verb.
(a) Please help me. You are requested to help Note: If the Active Voice sentence begins with “who”, then it
me. is changed into “By whom” in the Passive Voice.
(b)Always keep to You are always advised to Active Voice Passive Voice

K
the left. keep to the left. (a) Who wrote the letter? By whom was the letter
(c) Stand up on the bench. You are ordered to stand up written?
on the bench. (b)Where did you pick it? Where was it picked by
(F) PASSIVE OF INTERROGATIVE SENTENCES you?
(i) If the question in the Active Voice begins with Do/Does/ (c) When do they deliver When is the mail delivered
Did, the form of the verb in the Passive Voice is: the mail? by them?
Is/are/was/were + Past Participle of the Verb. (G) VERBS OF INCOMPLETE PREDICATION
Active Voice Passive Voice Whenever a transitive verb of incomplete predication
(a) Do you speak French? Is French spoken by you? (i.e. a verb which needs a complement in addition to an
(b)Did India win the Was the match won by object) is changed from the Active Voice into the Passive

KUNDAN
match? India? Voice, the object (and never the complement) becomes
(ii) If the question in the Active Voice begins with modal the subject in the Passive Voice.
auxiliary the form of the verb in the Passive Voice is: Active Voice Passive Voice
Modal auxiliary + be + Past Participle of the Verb (a) They elected him He was elected secretary by
Active Voice Passive Voice secretary. them.
(a) Must we finish this Must this work be finished (b)The people proclaimed He was proclaimed king by
work? by us? him king. the people.

EXERCISE
Directions: A sentence has been given in Active Voice/ 3. This book was written by an Irishman.
Passive Voice. Out of the four alternatives suggested, select (a) An Irishman wrote this book.
the one which best expresses the same sentence in Passive/ (b) An Irishman has written this book.
Active Voice and mark your answer in the Answer-Sheet. (c) An Irishman had written this book.
1. The bag was packed by his mother. (d) An Irishman writes this book.
(a) His mother has packed the bag. 4. The electricians test the fire alarm.
(b) His mother packed the bag. (a) The fire alarm was tested by the electricians.
(c) His mother pack the bag. (b) The fire alarm has been tested by the electricians.
(d) His mother packs the bag. (c) The fire alarm is tested by the electricians.
2. Football is played by the children. (d) The fire alarm is being tested by the electricians.
(a) The children are playing football. 5. The girls can play handball.
(b) The children have played football. (a) Handball could be played by the girls.
(c) The children played football. (b) Handball can be played by the girls.
(d) The children play football.
(c) Handball is to be played by the girls. (b) Why haven’t you finished the exercises on time?
(d) Handball is played by the girls. (c) Why aren’t you finishing the exercises on time?
6. The teacher closes the window. (d) Why didn’t you finished the exercises on time?
(a) The window is closed by the teacher. 17. Last year 2,000 new units had been produced by the time
(b) The window is being closed by the teacher. we introduced the new design.
(c) The window will be closed by the teacher. (a) We had produced 2,000 new units when we
(d) The window is to be closed by the teacher. introduced the new design last year.
7. They expected us to build the road. (b) We had produced 2,000 new units by the time we
(a) We expected to build the road. introduced the new design last year.
(b) We have expected to build the road. (c) We produced 2,000 new units by the time we
(c) We are expected to build the road. introduced the new design last year.
(d) We were expected to build the road. (d) We have produced 2,000 new units by the time we
8. Snow covered the mountains. introduced the new design last year.
(a) The mountains are covered with snow. 18. Casual clothes must not be worn.
(b) The mountains have been covered with snow. (a) You must not wear casual clothes.
(c) The mountains were covered with snow. (b) You should not wear casual clothes.
(d) The mountains are to be covered with snow. (c) You do not wear casual clothes.
9. The professor is going to show the students an old bone. (d) You will not wear casual clothes.

K
(a) The students will be shown an old bone. 19. $400,000 in profit has been reported this year by the
(b) The students are going to be shown an old bone. company.
(c) The students are going to see an old bone. (a) The company has reported $400,000 in profit this
(d) The students are going to be seen an old bone. year.
10. Who said it? (b) The company had reported $400,000 in profit this
(a) It is said by whom? year.
(b) It was said by whom? (c) The company is reported $400,000 in profit this year.
(c) It was being said by whom? (d) The company reported $400,000 in profit this year.
(d) It has been said by whom? 20. The test will be given at five o’clock this afternoon.
11. Rick told Sue to give up smoking. (a) The school would give the test at five o’clock this
(a) Sue was told to give up smoking. afternoon.

KUNDAN
(b) Sue was asked to give up smoking. (b) The school gives the test at five o’clock this
(c) Sue has been told to give up smoking. afternoon.
(d) Sue told to give up smoking. (c) The school will give the test at five o’clock this
12. People said that the President is ill. afternoon.
(a) It was said that the President was ill. (d) The school will be giving the test at five o’clock this
(b) It was said that the President had been ill. afternoon.
(c) It was said that the President has been ill. 21. You should look into this matter.
(d) It was said that the President is ill. (a) This matter has been looked into by you.
13. The instructions have been changed. (b) This matter may be looked into by you.
(a) Someone changed the instructions. (c) This matter should be looked into by you.
(b) Someone has changed the instructions. (d) This matter into looked by you.
(c) Someone has been changed the instructions. 22. Do you imitate others?
(d) Someone have changed the instructions. (a) Are others imitated by you?
14. She will have to be taught. (b) Are others being imitated by you?
(a) Someone teach her. (c) Were others imitated by you?
(b) Someone will teach her. (d) Have others been imitated by you?
(c) Someone will have to teach her. 23. He was arrested on a charge of theft, but for lack of
(d) Someone will need to teach her. evidence he was released.
15. This car was manufactured in Japan by Toyota. (a) He was arrested on a charge of theft, but of evidence
(a) Toyota had manufactured this car in Japan. he was released for lack.
(b) Toyota has manufactured this car in Japan. (b) The police arrested him on a charge of theft, but for
(c) Toyota is manufacturing this car in Japan. lack of evidence he was released.
(d) Toyota manufactured this car in Japan. (c) The police arrested him on a charge of theft, but for
16. Why aren’t the exercises being finished on time? lack of evidence released him.
(a) Why don’t you finish the exercises on time? (d) None of these.
24. A stone struck me on the head. (c) Why has time been wasted by you?
(a) I was struck by a stone on the head. (d) Why is time being wasted by you?
(b) My head was struck by a stone. 34. It is time to ring the bell.
(c) I had been struck by a stone on the head. (a) It is time the bell rings.
(d) I was struck on the head by a stone. (b) It is being time to ring the bell.
25. Who can question Gandhi’s integrity? (c) It is time for the bell to ring.
(a) By whom Gandhi’s integrity can be questioned? (d) It is time for the bell to be rung.
(b) By whom can Gandhi’s integrity be questioned? 35. We hope that we shall win the match.
(c) Gandhi’s integrity can be questioned by whom? (a) The match is hoped to be won.
(d) Who could have questioned Gandhi’s integrity? (b) Match winning is our hope.
26. He asked me to finish the work in time. (c) It is hoped that the match will be won by us.
(a) I was asked that I should finish the work in time. (d) Winning the match is hoped by us.
(b) He asked me that I should finish the work in time. 36. I have watched a movie today.
(c) I was being asked to finish the work in time. (a) A movie has been watched by me today.
(d) I was asked to finish the work in time by him. (b) A movie had been watched by me today.
27. The residents celebrated the Republic Day. (c) A movie is being watched by me today.
(a) The Republic Day is celebrated by the residents. (d) A movie was being watched by me today.
(b) The Republic Day was celebrated by the residents. 37. They say that women live longer than men.
(c) The Republic Day has been celebrated by the

K
(a) Women are said to have lived longer than men.
residents. (b) Women are said to live longer than men.
(d) Celebration of the Republic Day was done by the (c) Women live longer than men.
residents. (d) Women have said to live longer than men.
28. A lion may be helped even by a little mouse. 38. Does your mum pick you up?
(a) A little mouse may even help a lion. (a) Have you picked up by your mum?
(b) Even a little mouse may help a lion. (b) Will you be picked up by your mum?
(c) A little mouse can even help a lion. (c) Are you picked up by your mum?
(d) Even a little mouse ought to help a lion. (d) Were you picked up by your mum?
29. I know him. 39. They wear blue shoes.
(a) He is known by me. (a) Blue shoes were worn by them.

KUNDAN
(b) He was known to me. (b) Blue shoes have been worn by them.
(c) He has been known by me. (c) Blue shoes will be worn by them.
(d) He is known to me. (d) Blue shoes are worn by them.
30. His subordinates accused him of various offences. 40. They don’t help you.
(a) They accused him of various offences. (a) You will not be helped by them.
(b) It was accused by his subordinates that he has done (b) You are not helped by them.
various offences. (c) You were not helped by them.
(c) His subordinates accused that he had done various (d) You may not helped by them.
offences. 41. Why did you not agree to my proposal?
(d) He was accused of various offences by his (a) Why was my proposal not agreed to?
subordinates. (b) Why was my proposal not agreed by you?
31. He was obliged to resign. (c) Why my proposal not agreed by you?
(a) He was made to resign. (d) Why was my proposal not agreed to by you?
(b) To resign was his obligation. 42. A lion does not eat grass however hungry he may be.
(c) Circumstances obliged him to resign. (a) Grass is not eaten by a lion however hungry he may
(d) Resignation obliged by him. be.
32. This shirt cannot be worn any longer. (b) Grass is not being eaten by a lion however hungry
(a) I cannot wear this shirt any longer. he may be.
(b) Wearing of this shirt any longer is not possible. (c) Grass is eaten not by a lion however hungry he may
be.
(c) This shirt is too worn out to be worn any longer.
(d) Grass is being not eaten by a lion however hungry
(d) This worn out shirt cannot be worn any longer.
he may be.
33. Why are you waste time?
43. The teacher punished the boys who had not done their
(a) Why is time wasted by you?
home work.
(b) Why is time been wasted by you?
(a) The boys who had not done their home work had 53. They will not show the new film.
been punished by the teacher. (a) The new film will not be shown by them.
(b) The boys were punished by the teacher who had not (b) The new film would not be shown by them.
done their home work (c) The new film is not be shown by them.
(c) The boys who had not done their home work were (d) The new film cannot be shown by them.
punished by the teacher. 54. We have not agreed to this issue.
(d) The boys who had not done their home work were (a) This issue is not to be agreed to by us.
being punished by the teacher. (b) This issue had not been agreed to by us.
44. People claim to have seen the suspect in the city. (c) This issue has not been agreed to by us.
(a) The suspect is being seen in the city . (d) This issue can not been agreed to by us.
(b) The suspect has been seen in the city. 55. They have not caught the thieves.
(c) The suspect is claimed to have been seen in the city. (a) The thieves had not been caught by them.
(d) The suspect was seen by people in the city. (b) The thieves have not been caught by them.
45. You watch movies every week. (c) The thieves were not caught by them.
(a) Movies are watched by you every week. (d) The thieves could not be caught by them.
(b) Movies are being watched by you every week. 56. Has she phoned him?
(c) Movies have been watched by you every week. (a) Has she been phoned by him?
(d) Every week you watch movie.
(b) Was he phoned by her?

K
46. We set the table.
(c) Was she phoned by him?
(a) The table is to be set by us.
(d) Has he been phoned by her?
(b) The table is getting set by us
57. Have they noticed us?
(c) The table is set by us.
(a) Had we been noticed by them?
(d) The table is being set by us.
(b) Have we been noticed by them?
47. She pays a lot of money.
(c) Were we noticed by them?
(a) A lot of money has been paid by her.
(d) Have we noticed by them?
(b) A lot of money is to be paid by her.
58. A thief stole my car.
(c) A lot of money gets paid by her.
(a) My car has been stolen by a thief.
(d) A lot of money is paid by her.
(b) My car had stolen by a thief.
48. I draw a picture.

KUNDAN
(c) My car was stolen by a thief.
(a) A picture is to be drawn by me.
(d) My car is stolen by a thief.
(b) A picture is drawn by me.
59. They didn’t let him go.
(c) A picture is drew by me.
(a) He was not to be let go by them.
(d) A picture will be drawn by me.
(b) He was not letting go by them.
49. Will the teacher test our English?
(c) He was not let go by them.
(a) Will our English be tested by the teacher?
(d) He could not let go by them.
(b) Will our teacher test our English?
60. Our boss will sign the contract.
(c) Is our English to be tested by the teacher?
(a) The contract is to be signed by our boss.
(d) Will our English tested by the teacher?
(b) The contract will be signed by our boss.
50. Could Jenny lock the door? (c) The contract would be signed by our boss.
(a) Was the door locked by Jenny? (d) The contract could be signed by our boss.
(b) Could the door be locked by Jenny? 61. Do not put your feet on the seats.
(c) Was Jenny able to lock the door (a) One should not put feet on the seats.
(d) Can the door be locked by Jenny? (b) You must not put your feet on the seats.
51. (a) Will the company employ a new worker? (c) Feet must not be put on the seats.
(a) Will a new worker to be employed by the company? (d) Feet should not be put on the seats.
(b) Will a new worker be employed by the company? 62. You must look into this matter.
(c) Is a new worker to be employed by the company? (a) This matter has been looked into by you.
(d) Shall a new worker be employed by the company? (b) This matter may be looked into by you.
52. You ought to wash the car. (c) This matter should be looked into by you.
(a) The car has to be washed by you.
(d) This matter into looked by you
(b) The car is to be washed by you.
63. He asked me to finish the work in time.
(c) The car ought to be washed by you.
(a) I was asked that I should finish the work in time.
(d) The car has ought to be washed by you.
(b) He asked me that I should finish the work in time.
(c) I was being asked to finish the work in time. (d) The last day’s play between India and Sri Lanka were
(d) I was asked to finish the work in time by him. disrupted by rain.
64. The residents celebrated Holi. 73. Can you solve this sum quickly?
(a) Holi is celebrated by the residents. (a) Can this sum be solved by you?
(b) Holi was celebrated by the residents. (b) Can this sum solve by you?
(c) Holi has been celebrated by the residents. (c) Can this sum be solved you quickly?
(d) Celebration of Holi was done by the residents. (d) Can this sum be solved by you quickly?
65. This shirt cannot be worn any longer. 74. Saumya married Rohan.
(a) I cannot wear this shirt any longer. (a) Rohan was married for Saumya.
(b) Wearing of this shirt any longer is not possible. (b) Rohan was married to Saumya.
(c) This shirt is too worn out to be worn any longer. (c) Rohan is married to Saumya.
(d) This worn out shirt cannot be worn any longer. (d) Rohan has been married to Saumya.
66. Snow covered the mountains. 75. I want to buy a house.
(a) The mountains are covered with snow. (a) I want a house to bought.
(b) The mountains have been covered with snow. (b) I want a house to have bought.
(c) The mountains were covered with snow. (c) I want a house to be bought.
(d) The mountains are to be covered with snow. (d) I wanted a house to be bought.
76. Women like men to flatter them.
67. Why aren’t the exercises being finished on time?
(a) Men are liked by women to flatter them.

K
(a) Why don’t you finishing the exercises on time?
(b) Why haven’t you finishing the exercises on time? (b) Women like to be flattered by men.
(c) Women like that men should flatter them.
(c) Why aren’t you finishing the exercises on time?
(d) Women are liked to be flattered by men.
(d) Why didn’t you finishing the exercises on time?
77. I saw a few trees laden with fruits.
68. The test will be given at five o’clock this afternoon.
(a) The school would give the test at five o’clock this (a) I was seen a few trees laden with fruits.
(b) A few trees were saw by me laden with fruits.
afternoon.
(c) A few trees had been seen by me laden with fruits.
(b) The school gives the test at five o’clock this
(d) A few trees were seen by me laden with fruits.
afternoon.
78.Without effort nothing can be gained.
(c) The school will give the test at five o’clock this
afternoon. (a) We can gain anything with effort.

KUNDAN
(b) We can gain nothing without effort.
(d) The school will be giving the test at five o’clock this
afternoon. (c) Without effort, we cannot gain anything.
(d) We can gain only with effort.
69. The Shakuntalam was written by Kalidas.
(a) Kalidas wrote the Shakuntalam. 79. God helps those who help themselves.
(a) Those whose help themselves are helped by god.
(b) Kalidas written the Shakuntalam.
(c) Kalidas had written the Shakuntalam. (b) Those who help themselves are helped by god.
(c) Those which help themselves are helped by god.
(d) Kalidas writes the Shakuntalam.
(d) Those who help themselves were helped by god.
70. We must now deal with these problems.
80. He is not hated by me.
(a) These problems must now be dealt with by us.
(a) I do not hate him.
(b) These problems must now be dealing with by us.
(b) I do not hate he.
(c) These problems must now deal with by us.
(c) I didn’t hate him.
(d) These problems are to be dealt with by us.
71. I did not trust anybody. (d) I will not hate him.
(a) Nobody was trusted by me. 81. The electricians test the fire alarm.
(a) The fire alarm was tested by the electricians.
(b) Anybody had been trusted by me.
(c) Nobody would be trusted by me, (b) The fire alarm has been tested by the electricians.
(d) Nobody has been trusted by me. (c) The fire alarm is tested by the electricians.
72. Rain disrupted the last day’s play between India and (d) The fire alarm is being tested by the electricians.
82. The professor is going to show the students an old bone.
Srilanka.
(a) The last days play of India and Srilanka was disrupted (a) The students will be shown an old bone.
(b) The students are going to be shown an old bone.
by rain.
(c) The students are going to see an old bone.
(b) India and Sri Lanka’s play of the last day was
(d) The students are going to be seen an old bone.
disrupted by rain.
(c) The last day’s play between India and Sri Lanka was 83. The instructions have been changed.
disrupted by rain. (a) Someone changed the instructions.
(b) Someone has changed the instructions. 91. Anybody will rob you if you are not careful.
(c) Someone has been changed the instructions. (a) You will be robbed if you are not careful.
(d) Someone have changed the instructions (b) You will robbed if you are not careful.
84. The test will be given at five o’clock this afternoon. (c) You will be rob if you are not careful.
(a) The school would give the test at five o’clock this (d) You will have robbed if you are not careful.
afternoon. 92. One must do one’s duty.
(b) The school gives the test at five o’clock this (a) Duty must done.
afternoon. (b) Duty must be done.
(c) The school will give the test at five o’clock this (c) Duty must have be done.
afternoon. (d) Duty must have been done.
(d) The school will be giving the test at five o’clock this 93. someone gave her a bracelet.
afternoon (a) She was given a bracelet.
85. He was arrested on a charge of theft, but for lack of (b) A bracelet was given to her.
evidence he was released. (c) She has been given a bracelet.
(a) He was arrested on a charge of theft, but of evidence (d) She is being given a bracelet.
he was released for lack. 94. a stone struck me on the head.
(b) The police arrested him on a charge of theft, but for (a) I was struck by a stone on the head.
lack of evidence he was released.
(b) My head was struck by a stone.

K
(c) The police arrested him on a charge of theft, but for
(c) I had been struck by a stone on the head.
lack of evidence released him.
(d) I was struck on the head by a stone.
(d) None of these
95. Rama gave the beggar an old shirt.
86. He asked me to finish the work in time.
(a) An old shirt was given to Rama by the beggar.
(a) I was asked that I should finish the work in time.
(b) An old shirt was given to the beggar by Rama.
(b) He asked me that I should finish the work in time.
(c) The beggar was gave an old shirt by Rama.
(c) I was being asked to finish the work in time.
(d) An old shirt was gave to the beggar by Rama.
(d) I was asked to finish the work in time by him.
96. he asked me to wait.
87. His subordinates accused him of various offences.
(a) I was asked wait.
(a) They accused him of various offences.
(b) I was asked to wait.
(b) It was accused by his subordinates that he has done

KUNDAN
(c) I was asked to waitng
various offences.
(d) I was asked for wait.
(c) His subordinates accused that he had done various
97. We hope that we shall win the match.
offences.
(a) The match is hoped to be won.
(d) He was accused of various offences by his
(b) Match winning is our hope.
subordinates.
(c) It is hoped that the match will be won by us.
88. Have the box broken.
(d) Winning the match is hoped by us.
(a) Have the broken box.
98. They don’t help you.
(b) Break the box.
(a) You will not be helped by them.
(c) Get someone to break the box.
(b) You are not helped by them.
(d) They have broken the box.
(c) You were not helped by them.
89. The child’s shrill wail broke the silence.
(d) You may not helped by them.
(a) The silence was being broken by the child’s shrill
99. The exhibition was opened by the governor.
wail.
(a) The governor opened the exhibition.
(b) The child’s shrill wail was broken by the silence.
(b) The governor has opened the exhibition.
(c) The silence was broken by the child’s shrill wail.
(c) The governor had opened the exhibition.
(d) The silence was being broken by the child’s shrill
(d) The governor was opened the exhibition
wail.
100. English is spoken all over the world.
90. They promised Mary a new doll on her birthday.
(a) People spoke English all over the world.
(a) Mary had been promised a new doll on her birthday.
(b) People speak English all over the world.
(b) Mary was promised a new doll on her birthday.
(c) People will People speak English all over the world.
(c) Mary is promised a new doll on her birthday.
(d) People have spoken English all over the world.
(d) Mary had promised a new doll on her birthday.
Answers
1. b 2. d 3. a 4. c 5. b 51. b 52. c 53. a 54. c 55. b
6. a 7. d 8. c 9. b 10. b 56. d 57. b 58. c 59. c 60. b
11. a 12. d 13. b 14. c 15. d 61. c 62. c 63. d 64. b 65. a
16. c 17. b 18. a 19. a 20. c 66. c 67. c 68. c 69. a 70. a
21. c 22. a 23. c 24. d 25. b 71. a 72. c 73. d 74. b 75. c
26. d 27. b 28. b 29. d 30. d 76. b 77. d 78. b 79. b 80. a
31. c 32. a 33. d 34. d 35. c 81. c 82. b 83. b 84. c 85. c
36. a 37. b 38. c 39. d 40. b 86. d 87. d 88. c 89. c 90. b
41. d 42. a 43. c 44. c 45. a 91. a 92. b 93. a 94. a 95. b
46. c 47. d 48. b 49. a 50. b 96. b 97. c 98. b 99. a 100. b

Direct and Indirect Speech

K
The words of a speaker are reported in two ways:
(i) Direct Speech: When we quote the speaker’s
actual words. These words are put within inverted
commas (“ ”).
(ii) Indirect (or Reported) Speech: When we report
what the speaker said without quoting his exact
words.
Direct: He said to me, “I am reading a book.”
Ex.: Dipu said, “I have bought a red pen.”
(Direct Speech)
Dipu said that he had bought a red pen.
(Indirect Speech)
(d) When the reporting verb is in the past tense, the
present perfect continuous becomes the past
perfect continuous.
Ex.: He said, “I have been doing the job for a month.”
Indirect: He told me that he was reading a book. (Direct Speech)

KUNDAN
Rule I: When the reporting or principal verb is in the present He said that he had been doing the job for a month.
tense, the tenses of the direct speech do not change. (Indirect Speech)
Ex.: He says, “I am busy.” (Direct Speech) (e) The simple past changes to the past perfect.
He says that he is busy. (Indirect Speech) Ex.: You said, “she wrote a letter.”
Rule II: When the reporting or principal verb is in the future (Direct Speech)
tense, the tenses of the direct speech do not change. You said that she had written a letter.
Ex: He will say, “I am fine.” (Direct Speech) (Indirect Speech)
He will say that he is fine. (Indirect Speech) (f) The past continuous changes to the past perfect
Rule III: When the reporting or principal verb is in the past continuous.
tense, all present tenses of the direct speech are Ex.: He said, “I was writing a letter to the principal.”
changed into the corresponding past tenses. (Direct Speech)
(a) When the reporting verb is in the past tense, the He said that he had been writing a letter to the
simple present in the direct speech becomes the principal. (Indirect Speech)
simple past. (g) The past perfect in the direct speech does not
Ex.: He said, “I am happy.” (Direct Speech) change.
He said that he was happy. (Indirect Speech) Ex.: The students said, “We had completed the work.”
(b) When the reporting verb is in the past tense, the (Direct Speech)
present continuous in the direct speech becomes The students said that they had completed the work.
the past continuous. (Indirect Speech)
Ex.: The boy said, “My master is writing letters.” (h) The past perfect continuous in the direct speech
(Direct Speech) does not change.
The boy said that his master was writing letters. Ex.: She said, “I had been doing my work for two
(Indirect Speech) months.” (Direct Speech)
(c) When the reporting verb is in the past tense, the She said that she had been doing her work for
present perfect becomes the past perfect.
two months. (Indirect Speech)
Rule IV: When the reporting or principal verb is in the First person pronoun will change according to the
past tense, the shall/will of the future tense is subject.
changed to should/would Second person pronoun will change according to
Ex.: The student said, “I shall do the work.” the object.
(Direct Speech) There will be no change in third person pronoun.
The student said that he should do the work. Ex.: He said, “I am busy.” (Direct Speech)
(Indirect Speech) He said that he was busy. (Indirect Speech)
Note: If the direct speech consists of a sentence expressing She said to me, “I don’t believe you.” (Direct speech)
universal truth or habitual action, there is no change She told me that she didn’t believe me.
in tense. (Indirect Speech)
Ex.: The teacher said, “The earth revolves round the He said to me, “I am right but you are wrong.”
sun.” (Direct Speech) (Direct Speech)
The teacher said that the earth revolves round the He told me that he was right but I was wrong.
sun. (Indirect Speech) (Indirect Speech)
Rule V: When direct speech is changed into indirect speech, INDIRECT NARRATION OF INTERROGATIVE
the tense of certain words also changes. SENTENCES
Direct Speech Indirect Speech
In reporting questions, the indirect speech is introduced by
(a) now then

K
verbs such as asked, enquired etc.
(b) here there
(c) ago before Ex.: He said to me, “What are you doing?” (Direct Speech)
(d) thus so She asked me what I was doing. (Indirect Speech)
(e) this that “Where do you live?”, asked the stranger.
(f) these those (Direct Speech)
(g) hence thence The stranger asked/enquired where I lived.
(h) today that day
(Indirect Speech)
(i) tonight that night
(j) yesterday the day before INDIRECT NARRATION OF IMPERATIVE SENTENCES
(k) tomorrow next day Ex.: Ram said to Arjun, “Go away.”

KUNDAN
(l) last night the night before (Direct Speech)
(m) next day the following day
Ram ordered Arjun to go away. (Indirect Speech)
Ex.: He said, “I am glad to be here this evening.”
(Direct Speech) He said to him, “Please wait here until I return.”
He said that he was glad to be there that evening. (Direct Speech)
(Indirect Speech) He requested him to wait there until he returned.
Rule VI: There is change in pronouns when direct speech is (Indirect Speech)
changed into indirect speech.

EXERCISE
Directions: A sentence has been given in Direct/Indirect (b) She said that the lesson had already started when he
form. Out of the four alternatives suggested, select the one had arrived.
which best expresses the same sentence in Indirect/Direct (c) She said that the lesson had already started when he
form. arrived.
1. She said, “I was teaching earlier.” (d) She said that the lesson already started when he had
(a) She said that she was teaching earlier. arrived.
(b) She said that she had been teaching earlier. 3. He said that he had waited for an hour.
(c) She said that she has been teaching earlier. (a) He said, “I have been waited for an hour.”
(d) She said that she is teaching earlier. (b) He said, “I have been waiting for an hour.”
2. She said, “The lesson had already started when he (c) He said, “I have waited for an hour.”
arrived.” (d) He said, “I waited for an hour.”
(a) She said that the lesson already started when he 4. Kiran asked him if he had read the letter.
arrived. (a) Kiran asked, “Did you read the letter?”
(b) Kiran asked, “Have you read the letter?” (b) Deb told Salma that she really has been a nice
(c) Kiran asked, “Are you read the letter?” roommate.
(d) Kiran asked, “Do you read the letter?” (c) Deb told Salma that she really was a nice roommate.
5. He said, “I do not wish to see any of you; go away.” (d) Deb told Salma that she is really a nice roommate.
(a) He said angrily that he did not wish to see any of 13. Mrs. Taylor wondered, “Where is my wallet?”
them and asked them to go away. (a) Mrs. Taylor wondered where her wallet was.
(b) He said angrily that he had not wished to see any of (b) Mrs. Taylor wondered where was her wallet.
them and asked them to go away. (c) Mrs. Taylor wondered where her wallet is.
(c) He said angrily that he did not wished to see any of (d) Mrs. Taylor wondered where is her wallet.
them and asked them to go away. 14. Sumit said to Shashi, “My father owns a pub in Mumbai.”
(d) He said angrily that he did not wish to see any of (a) Sumit told Shashi that his father owns a pub in
them and asked them to went away. Mumbai.
6. My teacher often says to me, “If you don’t work hard (b) Sumit told Shashi that his father owned a pub in
you will fail.” Mumbai.
(a) My teacher often says to me that if I don’t work hard (c) Sumit told Shashi that his father has owned a pub in
I would fail. Mumbai.
(b) My teacher often says to me that if I didn’t work (d) Sumit told Shashi that his father had a pub in Mumbai.
hard I would fail. 15. Aman said to Sunita , “Will you marry me?”
(c) My teacher often says to me that if I didn’t work (a) Aman asked Sunita if she would marry him.

K
hard I will fail. (b) Aman asked Sunita if she will marry him.
(d) My teacher often says to me that if I don’t work hard (c) Aman asked Sunita that will she marry him.
I will fail. (d) Aman asked Sunita whether she would marry him.
7. John asked, “Where does Paul live?” 16. Hari asked Sunil, “Do you like going to the swimming
(a) John wondered where Paul lived. pool?”
(b) John wondered where Paul lives. (a) Hari asked Sunil that did he like going to the swimming
(c) John wondered where does Paul live. pool.
(d) John wondered where Paul live. (b) Hari asked Sunil if he liked going to the swimming
8. Liz said, “Does John have eggs for breakfast?” pool.
(a) Liz wondered if John has eggs for breakfast. (c) Hari asked Sunil if he likes going to the swimming

KUNDAN
(b) Liz wondered if John have eggs for breakfast. pool.
(c) Liz wondered if John had eggs for breakfast. (d) Hari asked Sunil that does he like going to the
(d) Liz wondered if John has had eggs for breakfast. swimming pool.
9. The campers asked Mrs Taylor, “Can we spend the 17. Kailash said, ‘Why is Bhavana always in a bad mood?”
evening in town?” (a) Kailash wondered why Bhavana is always in a bad
(a) The campers asked Mrs. Taylor if they could spend mood.
the evening in town. (b) Kailash wondered why Bhavana has always bad
(b) The campers asked Mrs. Taylor if they can spend mood.
the evening in town. (c) Kailash wondered why Bhavana was always in a
(c) The campers asked Mrs. Taylor if they would spend bad mood.
the evening in town. (d) Kailash wondered why does Bhavana has always
(d) The campers asked Mrs. Taylor if they could spent bad mood.
the evening in town. 18. Pooja said to Amit, “Are you glad to be in Lucknow?”
10. Rohit said, “Ritu isn’t fun to be with”. (a) Pooja asked Amit if he is glad to be in Lucknow.
(a) Rohit said that Ritu is not fun to be with. (b) Pooja asked Amit if he was glad to be in Lucknow.
(b) Rohit said that Ritu has not been fun to be with. (c) Pooja asked Amit that was he glad to be in Lucknow.
(c) Rohit said that Ritu had not fun to be with. (d) Pooja asked Amit that is he glad to be in Lucknow.
(d) Rohit said that Ritu was not fun to be with. 19. Nisha said, “My brother’s got a Ferrari.”
11. Jack said, “I wonder why Walter always wears a cap.” (a) Nisha said that her brother has got a Ferrari.
(a) Jack wondered why Walter always wears a cap. (b) Nisha said that her brother had a Ferrari.
(b) Jack wondered why Walter is always wearing a cap. (c) Nisha said that her brother has a Ferrari.
(c) Jack wondered why Walter always wore a cap. (d) Nisha said that her brother had got a Ferrari.
(d) Jack wondered why Walter always has to wear a cap. 20. Juhi said to Suman, “You can come and stay in my flat.”
12. Deb said, “Salma, you really are a nice roommate.” (a) Juhi told Suman that she can come and stay in her
(a) Deb told Salma that she really is a nice roommate. flat.
(b) Juhi told Suman that she could come and stay in her (a) My friend told that from one of the windows of his
flat. flat he could be seeing the Qutub Minar.
(c) Juhi told Suman that she should come and stay in (b) My friend told that from one of the windows of his
her flat. flat he can see the Qutub Minar.
(d) Juhi told Suman that she may come and stay in her (c) My friend said that from one of the windows of his
flat. flat he could see the Qutub Minar.
21. Manisha said, ‘How long does Anjali swim for?” (d) My friend told that from one of the windows of his
(a) Manisha wondered how long does Anjali swim for. flat he would see the Qutub Minar.
(b) Manisha wondered how long Anjali swam for. 28. The teacher said, “Don’t sleep late and miss the train.”
(c) Manisha wondered how long did Anjali swim for. (a) The teacher advised us not to sleep late and missed
(d) Manisha wondered how long Anjali swims for. the train.
22. Shalini said, “I bought a house in Ranchi last year.” (b) The teacher advised us not to sleep late and miss the
(a) Shalini said that she bought a house in Ranchi the train.
year before. (c) The teacher advised us not sleep late and missed the
(b) Shalini said that she had bought a house in Ranchi train.
the year before. (d) The teacher advised us do not to sleep late and
(c) Shalini said that she has bought a house in Ranchi missed the train.
last year.
29. He asked me, “Do you like this?” and I said, “Yes.’

K
(d) Shalini said that she has bought a house in Ranchi
(a) He asked me if he liked it and I said that I did.
the year before.
(b) He asked me if I had liked it and I said that I did.
23. She said that she was going to the market.
(c) He asked me if I like it and I said that I did.
(a) She said, “I was going to the market.”
(d) He asked me if I liked it and I said I did.
(b) She said, “I am going to the market.”
30. She said, “I’ve lost my key. Can I borrow yours?”
(c) She said, “I have been going to the market.”
(d) She said, “I would be going to the market.” (a) She said that she has lost her key and asked me
24. His friends said that they would go home the following whether she could borrow mine.
Sunday. (b) She said that she had lost her key and asked me
(a) His friends said, “We can go home next Sunday.” whether she could borrow mine.
(b) His friends said, “We will go home the following (c) She said that she had been lost her key and asked me

KUNDAN
Sunday.” whether she could borrow mine.
(c) His friends said, “We will go home next Sunday.” (d) She said that she lost her key and asked me whether
(d) His friends said, “We should go home next Sunday.” she could borrow mine
25. The teacher told Mark he had been very regular in his 31. He said, “Please let me go.”
work (a) He said that let him go.
(a) The teacher said to Mark, “You have been very (b) He requested them to let him go.
regular in your work.” (c) He requested them that they should let him go.
(b) The teacher said to Mark, “You are very regular in (d) He said to them that please let him go.
your work.” 32. He said, “Honesty is the best policy.”
(c) The teacher said to Mark, “You had been very regular (a) He told that honesty was the best policy.
in your work.” (b) He said that honesty has been the best policy.
(d) The teacher said to Mark, “You are being very regular (c) He told that honesty had been the best policy.
in your work.” (d) He said that honesty is the best policy.
26. The Chief Minister said, “What a terrible disaster the 33. “Please don’t go away.”, she said.
earthquake is!” (a) She said to please her and not go away.
(a) The Chief Minister cried that the earthquake was a (b) She told me not to go away.
terrible disaster. (c) She begged that I not go away.
(b) The Chief Minister expressed with surprise that the (d) She begged me not to go away.
earthquake was a terrible disaster. 34. She said, ‘I had already eaten’.
(c) The Chief Minister exclaimed with sorrow that the (a) She said that she had eaten.
earthquake was a terrible disaster. (b) She said that she had already eaten.
(d) The Chief Minister told in pain that the earthquake (c) She said that she has been already eaten.
was a terrible disaster. (d) She said that she would have already eaten.
27. “From one of the windows of my flat I can see the Qutub 35. She said to him, “Be careful.”
Minar”, said my friend. (a) She said to him that be careful.
(b) She told him to be careful. (c) We said, “We watched a truly absorbing movie”
(c) She asked him to take care. (d) We said, “We watch a truly absorbing movie”
(d) She told him he should be careful. 46. He said, “I will teach him a lesson to remember me by.”
36. She said to him, “Leave my house at once.” (a) He said he will teach him a lesson to remember him
(a) She ordered him leave his house at once. by.
(b) She ordered him to leave my house at once. (b) He said he is to teach him a lesson to remember him
(c) She told to him to leave the house. by.
(d) She ordered him to leave his house at once. (c) He said he would teach him a lesson to remember
37. “Hello! How are you?” him by.
(a) He greeted me and asked how was I. (d) He said he has taught him a lesson to remember him
(b) He greeted me and asked me how I had been. by.
(c) He greeted me and asked me how I was. 47. I warned that at I could not tolerate her coming late.
(d) He greets me and asks me how I was. (a) I said to her, “You can no longer tolerate my coming
38. He said to us, “Let us have some coffee.” late.”
(a) He proposed us that we should have some coffee. (b) I said to her, “I can no longer tolerate your coming
(b) He proposed to us that they should have some coffee. late.”
(c) He proposed to us that we should have some coffee. (c) I said to her, “I cannot tolerate you coming late any
(d) He proposed to us that we should had some coffee. longer.”

K
39. he said, “ I will come here again tomorrow.” (d) I said to her, “I cannot tolerate your late coming any
(a) He said that he will go there again the next day. longer.”
(b) He said that he should go there again the next day. 48. Everybody said, “How well she sings!”
(c) He said that he would go there again tomorrow. (a) Everybody told us that she sings very well.
(d) He said that he would go there again the next day. (b) Everybody exclaimed that she sings very well.
40. He said, “After the class I had to return home.” (c) Everybody exclaimed that she sang very well.
(a) He said that after the class he had had to return (d) Everybody told us that how she sang very well.
home. 49. “May you live long and prosper”, said the old lady to
(b) He said that after the class he had to return home. her son.
(c) He said that after the class they had to return home. (a) The old lady blessed her son with long life and
(d) He said that after the class he has returned home. wished him prosperity.

KUNDAN
41. He said he goes for a walk every morning. (b) The old lady prayed for her son’s long life and
(a) He said, “I went for a walk every morning.” prosperity.
(b) He said, “I go for a walk every morning.” (c) The old lady prayed for her son and said that he
(c) He said, “I will go for a walk every morning.” might live long and prosper.
(d) He said, “He goes for a walk every morning.” (d) The old lady blessed her son and prayed for his long
42. I wondered how many discoveries went unheeded. life and prosperity.
(a) I said, “How many discoveries have gone unheeded.” 50. Kiran said to Kailash, “When do you intend to pay back
(b) I said, “How many discoveries went unheeded.” the money you borrowed from me?”
(c) I said, “Do discoveries go unheeded?” (a) Kiran enquired Kailash when did he intend to pay
(d) I said, “How many discoveries go unheeded?” back the money she borrowed from him.
43. He said that it used to be a lovely, quiet street. (b) Kiran enquired Kailash when he intended to pay back
(a) He said, “It was a lovely, quiet street.” the money she borrowed from him.
(b) He said, “It used to be a lovely, quiet street.” (c) Kiran enquired Kailash when he intended to pay back
(c) He said, “It has been a lovely, quiet street.” the money she had borrowed from him.
(d) He said, “It always used to be a lovely, quiet street.” (d) Kiran enquired when Kailash did intended to pay
44. He said he had done amazingly well that day. back the money she borrowed from him.
(a) He said, “I did amazingly well today.” 51. I said to my friend, “Good Morning. Let us go for a
(b) He said, “I am done amazingly well today.” picnic today.”
(c) He said, “I have done amazingly well today.” (a) I told good morning to my friend and asked to go for
(d) He said, “I am doing amazingly well today.” a picnic that day.
45. We said we had been watching a truly absorbing movie. (b) I wished my friend good morning and proposed that
(a) We said, “We were watching a truly absorbing movie” we should go for a picnic that day.
(b) We said, “We are watching a truly absorbing movie” (c) I wished my friend good morning and proposed that
they should go for a picnic that day.
(d) I told good morning to my friend and suggested to (c) I said I shall certainly take care of all eventualities.
go for a picnic that day. (d) I said I will certainly take care of all eventualities.
52. The new student asked the old one, “Do you know my 60. He said, ‘I will teach him a lesson to remember me by’.
name?” (a) He said he will teach him a lesson to remember him
(a) The new student asked the old one if he knew his by.
name. (b) He said he would teach him a lesson to remember
(b) The new student asked the old one that whether he him by.
knew his name. (c) He said he taught him a lesson to remember him by.
(c) The new student asked the old one did he know his (d) He said he has to teach him a lesson to remember him
name. by.
(d) The new student asked the old one if he knows his 61. He said, “I do not wish to see any of you; go away.”
name. (a) He said angrily that he did not wish to see any of
53. The kids yelled in a loud voice, “We love our family.” them and asked them to go away.
(a) The kids yelled loudly that they loved our family. (b) He said angrily that he had not wished to see any of
(b) The kids yelled in a loud voice that they loved their them and asked them to go away.
family. (c) He said angrily that he did not wished to see any of
(c) The kids yelled in a loud voice that they loved our them and asked them to go away.
family. (d) He said angrily that he did not wish to see any of

K
(d) The kids were shouting loudly that they loved our them and asked them to went away.
family. 62. Sumit said to Shashi, “My father owns a pub in Mumbai.”
54. He said to me, “Please visit my company tomorrow.” (a) Sumit told Shashi that his father owns a pub in
(a) He said to me to visit his company tomorrow. Mumbai.
(b) He requested me to visit his company tomorrow. (b) Sumit told Shashi that his father owned a pub in
(c) He requested me to visit his company the next day. Mumbai.
(d) He said to me to visit his company the next day. (c) Sumit told Shashi that his father has owned a pub in
55. He enquired, ‘When do you intend to pay me?’ Mumbai.
(a) He enquired when I intend to pay him. (d) Sumit told Shashi that his father had a pub in Mumbai.
(b) He enquired when I had intended to pay him. 63. He said, “We have done our work.”

KUNDAN
(c) He enquired when I intended to pay him. (a) He said that he had done his work.
(d) He enquired when I have intended to pay him. (b) He said that they have done his work.
56. He said, ‘Oh! that’s a nuisance.’ (c) He said that they had done his work.
(a) He exclaimed that it was a nuisance. (d) He said that they should do their work.
(b) He exclaimed that this was a nuisance. 64. “How did it get here?” she wanted to know.
(c) He exclaimed that it is a nuisance. (a) She wanted to know how did it get here.
(d) He exclaimed it to be a nuisance. (b) She wanted to know how it had got there.
57. You said, “I killed a rat rather wickedly.” (c) She wanted to know how did it got there.
(a) You said you killed a rat rather wickedly. (d) She wanted to know how it get there.
(b) You said you have killed a rat rather wickedly. 65. The police said to thief, “Don’t move.”
(c) You said you had killed a rat rather wickedly. (a) The police ordered the thief not to move.
(d) You said you did kill a rat rather wickedly. (b) The police ordered the thief he should not move.
58. They said, “We have put up an extraordinary (c) The police told the thief that he did not move.
performance indeed.” (d) The police ordered to the thief to not move.
(a) They said they have put up an extraordinary 66. My friend said “I am leaving today.”
performance indeed. (a) My friend said that he is leaving today.
(b) They said they had put up an extraordinary (b) My friend said that he was leaving today.
performance indeed. (c) My friend said that he leaves today.
(c) They said they put up an extraordinary performance (d) My friend said that he was leaving that day.
indeed. 67. The boy promised that he would do it the next day.
(d) They said they could put up an extraordinary (a) The boy promised, “I will do it tomorrow.”
performance indeed. (b) The boy said, “I would do it tomorrow.”
59. I said, “I shall certainly take care of all eventualities.” (c) The boy promised, “I shall do it tomorrow.”
(a) I said I should certainly take care of all eventualities. (d) The boy said, “I should do it tomorrow.”
(b) I said I could certainly take care of all eventualities.
68. They requested the teacher to let them go home. 75. Manisha said, ‘How long does Anjali swim for?”
(a) They said to the teacher, “Shall we go home?” (a) Manisha wondered how long does Anjali swim for.
(b) They said to the teacher, “Let me go home.” (b) Manisha wondered how long Anjali swam for.
(c) They said to the teacher, “Let us go home.” (c) Manisha wondered how long did Anjali swim for.
(d) They said to the teacher, “Allow us to go home.” (d) Manisha wondered how long Anjali swims for.
69. He enquired whether his name was Ahmed. 76. She shouted, “What a funny man you are?”
(a) He said to him, “Is your name Ahmed or not?” (a) She exclaimed that she was a very funny man.
(b) He said to him, “Is not your name Ahmed?” (b) She exclaimed that I was a very funny man.
(c) He said to him, “Is your name not Ahmed?” (c) She exclaimed if I was a very funny man.
(d) He said to him, “Is it your name Ahmed?” (d) She exclaims that I was a funny man.
70. “Hurry up”, he said to his servant, “don’t waste time” 77. ”You must be tired after such a trip.”
(a) He told his servant to hurry up and not to waste (a) He said we must have been tired after such a trip.
time. (b) He said we must be tired after that trip.
(b) He told his servant to hurry and don’t waste time. (c) He said we must be tired after such a trip.
(c) He told his servant to hurry up and do not waste (d) He said we would be tired after such a trip.
time. 78. “Don’t hesitate,” he said.
(d) He told his servant to hurry and should not waste (a) He persuaded me to do not hesitate.
time. (b) He persuaded me not to hesitate.

K
71. I asked Kiran if she would lend me her book. (c) He persuaded me so that I do not to hesitate.
(a) I said to Kiran, “Will she lend me your book?” (d) He persuaded me that I should not hesitate.
(b) I said to Kiran, “Will she lend me her book?” 79. “Don’t smoke,” the doctor warned my father.
(c) I said to Kiran, “Will you lend me her book?” (a) The doctor warned my father not to smoke.
(d) I said to Kiran, “Will you lend me your book?” (b) The doctor asked my father that he should not smoke.
72. The Prince said, “ It gives me great pleasure to be here (c) The doctor warned my father so that he does not
this evening.” smoke.
(a) The Prince said that it gives him great pleasure to be (d) The doctor warned my father to not to smoke.
there that evening. 80. “Will you make coffee?” he said.
(b) The Prince said that it gave him great pleasure to be (a) He asked me if I could make coffee.
there in the evening. (b) He asked me if I would make coffee.

KUNDAN
(c) The Prince said that it gave him great pleasure to be (c) He asked me if I will make coffee.
there that evening. (d) He asked me if I had made coffee.
(d) The Prince said that it would give him great pleasure 81. The prisoner said to the officer, “Please let me go.”
to be there that evening. (a) The prisoner said to the officer that let him go
73. “I believe”, said he, “that we are in this country among a (b) The prisoner requested the officer to let him go.
people whom we like and who like us.” (c) The prisoner requested the officer that they should
(a) He said that he believed that they were in that country let him go.
among a people whom they like and who like them. (d) The prisoner said to the officer that please let him go
(b) He said that he believed that they were in that country 82. He said to us, “let us have some coffee.”
among a people whom they liked and who like them. (a) He proposed us that we should have some coffee.
(c) He said that he believed that they had been in that (b) He proposed to us that they should have some coffee.
country among a people whom they liked and who (c) He proposed to us that we should have some coffee.
liked them. (d) He proposed to us that we should had some coffee.
(d) He said that he believed that they were in that country 83. He said he goes for a walk every morning.
among a people whom they liked and who liked them. (a) He said, “I went for a walk every morning.”
74. Juhi said to Suman, “You can come and stay in my flat.” (b) He said, “I go for a walk every morning.”
(a) Juhi told Suman that she can come and stay in her (c) He said, “I will go for a walk every morning.”
flat. (d) He said, “He goes for a walk every morning.”
(b) Juhi told Suman that she could come and stay in her 84. Everybody said, “How well she writes!”
flat. (a) Everybody told us that she writes very well.
(c) Juhi told Suman that she should come and stay in (b) Everybody exclaimed that she writes very well.
her flat. (c) Everybody exclaimed that she wrote very well.
(d) Juhi told Suman that she may come and stay in her (d) Everybody told us that how she wrote very well.
flat.
85. He said, “All that glitters is not gold.” (c) She said that she went to watch a movie the day
(a) He told that all that glitters was not gold. before.
(b) He said that all that glitters has not been gold. (d) She said that she went to watch a movie yesterday.
(c) He told that all that glitters cannot be gold. 94. He said, ‘Oh! that’s a nuisance.’
(d) He said that all that glitters is not gold. (a) He exclaimed that it was a nuisance.
86. Poonam said, “I saw Ambuj at the theatre on Monday.” (b) He exclaimed that this was a nuisance.
(a) Poonam said she saw Ambuj at the theatre on (c) He exclaimed that it is a nuisance.
Monday. (d) He exclaimed it to be a nuisance.
(b) Poonam said she has seen Ambuj at the theatre on 95. “Please don’t disturb me”, she said.
Monday. (a) She said to please her and not disturb.
(c) Poonam said she had seen Ambuj at the theatre on (b) She told me not to disturb her.
Monday. (c) She begged that I not disturb her.
(d) Poonam said she have seen Ambuj at the theatre on (d) She begged me not to disturb her.
Monday. 96. Mita said to Sita, “Where are you going?”
87. Will you come?” she asked me. (a) Mita asked sita where is she going.
(a) She asked me if I will come. (b) Mita asked sita where she was going.
(b) She asked me if I would come. (c) Mita asked sita where has she been going.
(c) She asked me if I would have come. (d) Mita asked sita where did she go.

K
(d) She asked me if I came. 97. Arun said, “Arjun has gone home?”
88. “Did he marry Shilpi?” she said. (a) Arun said that Arjun has gone home.
(a) She wondered whether he would marry Shilpi. (b) Arun told that Arjun had gone home.
(b) She wondered whether he had married Shilpi. (c) Arun told that Arjun went home.
(c) She wondered whether he married Shilpi. (d) Arun said that Arjun is going home.
(d) She wondered whether he will marry Shilpi. 98. Teacher said, “Never make mischief again.”
89. What time did it start?” he said. (a) The teacher warned me never to make mischief again.
(a) He wanted to know what time it had started. (b) The teacher warned me never make mischief again.
(b) He wanted to know what time it started. (c) The teacher warns me never to make mischief again.
(c) He wanted to know what time it has started. (d) The teacher had warned me never to make mischief

KUNDAN
(d) He wanted to know what time it starts. again.
90. “Why won’t he do it?” she said. 99. She said, “The lesson had already started when he
(a) She wondered why he won’t do it. arrived.”
(b) She wondered why he will not do it. (a) She said that the lesson already started when he
(c) She wondered why he wouldn’t do it. arrived.
(d) She wondered why he didn’t do it. (b) She said that the lesson had already started when he
91. He said, “I shall go as soon as possible.’ had arrived.
(a) He said that he should go as soon as possible. (c) She said that the lesson had already started when he
(b) He said that he would go as soon as possible. arrived.
(c) He said that he will go as soon as possible. (d) She said that the lesson already started when he had
(d) He said that he shall go as soon as possible. arrived.
92. He said that he came to see them. 100. Hariasked Sunil,“Do you like going to the swimming
(a) He said, “I come to see them.” pool?”
(b) He said, “I have came to see them.” (a) Hari asked Sunil that did he like going to the swimming
(c) He said, “I have come to see them.” pool.
(d) He said, “I came to see them.” (b) Hari asked Sunil if he liked going to the swimming
93. She said, “ I went to watch a movie yesterday.” pool.
(a) She said that she had gone to watch a movie the day (c) Hari asked Sunil if he likes going to the swimming
before. pool.
(b) She said that she did go to watch a movie the day (d) Hari asked Sunil that does he like going to the
before. swimming pool.
Answers
1. b 2. c 3. c 4. b 5. a 51. b 52. a 53. b 54. c 55. c
6. d 7. a 8. c 9. a 10. d 56. a 57. c 58. b 59. a 60. b
11. c 12. c 13. a 14. b 15. a 61. a 62. b 63. c 64. b 65. a
16. b 17. c 18. b 19. d 20. b 66. d 67. c 68. c 69. b 70. a
21. b 22. b 23. b 24. c 25. a 71. d 72. c 73. d 74. b 75. b
26. c 27. c 28. b 29. d 30. b 76. b 77. c 78. b 79. a 80. b
31. b 32. d 33. d 34. b 35. b 81. b 82. c 83. b 84. c 85. d
36. d 37. c 38. c 39. d 40. b 86. c 87. b 88. c 89. a 90. c
41. b 42. d 43. b 44. c 45. a 91. b 92. a 93. a 94. a 95. d
46. c 47. b 48. c 49. c 50. c 96. b 97. b 98. a 99. c 100. b

Common Errors

K
Directions: In each of the following exercises we have
sentences given in four parts. Read each sentence to find out
whether there is any error in it. The error, if any, will be in
one part of the sentence. The number of that part is the
answer. If there is no error, the answer is (e). (Ignore the
errors of punctuation, if any.)
1. (a) Magic realism is one/ (b) of the latest/ (c) addition to
good literature published in recent times./(d) No error
12. (a) Neither Prannoy nor his wife /(b) were aware / (c) of
the arrangements made for their journey. /(d) No error
13. (a) The man who I have often mentioned, /(b) is one
whose friendship I could /(c) wish to acquire because he
possesses my esteem./ (d) No error
14. (a) After the Civil War Harriet Tubman, herself an escaped
slave,/ (b) continued her efforts on behalf of former
slaves,/ (c) helping to educate freedmen, supporting
2. (a) Even though he found the subject /(b) rather children and she was assisting impoverished old people./

KUNDAN
interesting Manoj could not / (c) manage good marks / (d) No error
(d) in the examination./ (e) No error 15. (a) Never before in the history of music / (b) have musical
3. (a) I now understand why / (b) Sanjay did not told/ (c) superstars been able to command / (c) so extraordinary
me the reason why he was late. / (d) No error fees / (d) of the kind they do today. / (e) No error
4. (a) He got to the top /(b) and was very disappointed /(c) 16. (a) Researchers have demonstrated that homing pigeons
when he found that someone else has reached it first. /(b) can sense changes in the earth’s magnetic field, see
/(d) No error light waves that people cannot see, /(c) and can detect
5. (a) Our rich men, /(b) to say the least, /(c) did not advance low-frequency sounds from kilometres away. /(d) No error
the moral struggle of passive resistance as much the 17. (a) The sale of government-surplus machinery /(b) will
poor. / (d) No error begin at 9 a.m. /(c) and continue until the supply lasts.
6. (a) You are learning English / (b) for the last one year / (c) /(d) No error
but you show no improvement whatsoever. / (d) No error 18. (a) I am not / (b) one of those / (c) who believe everything
7. (a) My friend being unwilling to attend / (b) the court at I hear. / (d) No error
an early hour of the morning, sent a letter explaining /(c) 19. (a) There are many reasons /(b) that the whole character
why could he not obey. /(d) No error of the twentieth century / (c) should be very different /
8. (a) Such a life as this /(b) is far conducive to health than from that of the nineteenth. /(d) No error
that of the man /(c) who rises late. /(d) No error 20. (a) Disregard for odds and complete confidence / (b) in
9. (a) We had swam /(b) across the river before /(c) the sun one’s self / (c) have produced many of our successes. /
set. /(d) No error (d) No error
10. (a) I knew your college library /(b) was running 21. (a) No one /(b) who has seen him lecture in the class /(c)
chaotically but only recently did I discover /(c) how bad can deny that Mr Mehrotra has an interest and aptitude
the situation is. /(d) No error for teaching. /(d) No error
11. (a) If you know what it is /(b) that you want out of life / 22. (a) No less / (b) than fifty mariners / (c) were killed in the
(c) it is easy to find out what different kinds of job offers explosion. /(d) No error
this. /(d) No error
23. (a) No sooner the advertisement appeared /(b) in the Indian cricketers have mastered the art of making easy
newspapers /(c) than there was a rush at the booking things difficult. / (d) No error
window. /(d) No error 46. (a) The boy in grey hair / (b) looked a bit sad / (c) while
24. (a) The house with all its / (b) furnitures and exotic plants the rest of the audience were in a gay mood. / (d) No
was sold /(c) for Rs 50,000. / (d) No error error
25. (a) He made a great fuss /(b) about having an 47. (a) You will not get / (b) a banana or two here;/ (c) this
investigation, / (c) but it all ended with smoke. / (d) No fruitseller sells bananas in the dozen. / (d) No error
error 48. (a) On what I know / (b) of him, I can / (c) easily affirm
26. (a) The crew were /(b) on board / (c) and they soon that he is a good husband. / (d) No error
busied themselves in preparing to meet the storm. /(d) 49. (a) Kumble has not turned / (b) a word-class spinner
No error overnight ; / (c) his skill comes of practice. / (d) No error
27. (a) The long-awaited moment at last came and we set out 50. (a) If the work is / (b) without your capacity, / (c) why not
for the station, / (b) as merry a band of children / (c) as I take the help of others? (d) No error
have ever seen before or since. / (d) No error 51. (a) Children visiting the / (b) park are amused / (c) by the
28. (a) In these days of inflation /(b) a ten rupee’s note / (c) monkeys’play in the cages. / (d) No error
will not buy you even an ordinary meal. / (d) No error 52. (a) The enmity of the / (b) two groups has reached a level /
29. (a) Well, I spent six or seven years / (b) after high school (c) where reconciliation has become impossible. / (d) No
/ (c) trying to find a job for me / (d) but could not succeed error

K
in it. / (e) No error 53. (a) To make him succeed, / (b) the correct thing to do / (c)
30. (a) I am pleased to sanction / (b) one special increment / is to punish him until he does not try. / (d) No error
(c) to all the employees with this month. /(d) No error 54. (a) If he would not have / (b) confessed it himself / (c) the
31. (a) Our school would / (b) have won the match / (c) if crime could scarcely have been known to him. / (d) No
only we would have concentrated. / (d) No error error
32. (a) Ajay asked the dealer / (b) what the price / (c) of that 55. (a) The purpose of this book, / (b) however, is not to discuss
bicycle was and whether it is really made in Germany. / / (c) these basic issues in its various aspects. / (d) No error
(d) No error 56. (a) My uncle / (b) forbade me not to / (c) go through the
contents of his letter. / (d) No error
33. (a) The other day / (b) I told you / (c) about this book;
57. (a) How does he earn money / (b) is more important than
you may take. / (d) No error

KUNDAN
/ (c) how much he earns. / (d) No error
34. (a) Elders have said / (b) that we should not entertain /
58. (a) Manisha is / (b) more prettier / (c) than her sister.
(c) a too high opinion of our wisdom. / (d) No error
/ (d) No error
35. (a) He copied the letter/ (b) word by word / (c) and kept
59. (a) In the harbour great ships / (b) lay at anchor / (c)
it as a proof / (d) of his innocence. / (e) No error
bearing the names of faraway places like Aberdeen and
36. (a) He is a MA / (b) of the Delhi University / (c) and is
even Singapore. / (d) No error
expected to know his subject well. / (d) No error
60. (a) School offers many opportunities of meeting / (b)
37. (a) He is / (b) more liberal but / (c) not so competent as
helpful people, reading useful books, / (c) and obtain
his elder brother . / (d) No error
information about a variety of public careers. / (d) No
38. (a) Prabhash is a tall gentleman / (b) and can therefore / error
(c) play basketball better than others. / (d) No error 61. (a) All one can gather / (b) from the children / (c) are that
39. (a) He orders for / (b) every new book / (c) that is there were a loud noise and smoke. / (d) No error
published in Bihar. / (d) No error 62. (a) Politicians in particular / (b) get carried away / (c) by
40. (a) Since he has never kept his promise, / (b) I conclude analogies and draw unwarranted conclusions from them.
that / (c) he is nothing else than a liar . / (d) No error / (d) No error
41. (a) The beggar stood / (b) at the gate / (c) in the hope of 63. (a) The percentage of the labour force / (b) that is
receiving alms. / (d) No error unemployed / (c) has dropped sharply this month even
42. (a) I sat in the office / (b) to welcome the visitors / (c) but though it may be only temporarily. / (d) No error
nobody came for the whole day. / (d) No error 64. (a) In ancient times, / (b) Nubia was the principal corridor/
43. (a) We lived within the British / (b) before Independence (c) where there were cultural influences transmitted
and (c) we are yet to get out of the slave mentality. / (d) between Black Africa and the Mediterranean Basin.
No error / (d) No error
44. (a) I was not there / (b) when the incident took place / (c) 65. (a) Jayaprakash Narayan was among the last of a
but the news came to me by a friend. / (d) No error generation / (b) of freedom fighters who led the nation
45. (a) The match could have been / (b) won in ease / (c) but through decades of change / (c) so profound, many
young Indians are not able to imagine, much less 86. (a) He only died / (b) a week ago / (c) but it appears as if
remember, what slavery was like. / (d) No error it were ages. /(d) No error
66. (a) I know that / (b) Jaigopal is more efficient / (c) than 87. (a) This matter / (b) must be considered / (c) from every
any man in our organisation. / (d) No error points of view. / (d) No error
67. (a) The annual survey of chemistry published by the 88. (a) Before you / (b) enter the temple, / (c) you should
American Chemical Society / (b) attributes the vast take out your shoes. / (d) No error
change in warfare / (c) to the airplane and, above all, to 89. (a) Since we have / (b) only one life / (c) to live, let us
the motor fuel of today. / (e) No error enjoy. / (d) No error
68. (a) The successful self-employed man / (b) invariably 90. (a) Although I am living / (b) in Delhi for the / (c) last
works harder / (c) and worries most than the man on a seven years, I have not seen Purana Qila. / (d) No error
salary. / (d) No error 91. (a) That Lear was eager/ (b) to hear his own praise / (c)
69. (a) Mr Churchill must be / (b) the first to rejoice that / (c) becomes evident when he tells Cordelia, “Nothing will
both Sir Samuel Hoare and the Premier had vindicated so come from nothing.” / (d) No error
emphatically. / (d) No error 92. (a) I must start / (b) by dawn / (c) to reach the station in
70. (a) I am working / (b) on this job / (c) since last Monday. time. / (d) No error
/ (d) No error 93. (a) I got the information / (b) just now / (c) that the Kalka
71. (a) World affairs have surprises in store / (b) for Express is due to 4 p.m. / (d) No error
whomever tries / (c) to read the future from his knowledge 94. (a) Sati was practised / (b) among the Hindus / (c) but

K
of the past and from the signs of the present. / (d) No women’s activists of today cannot tolerate such
error nonsense anymore, / (d) No error
72. (a) Our neighbours are / (b) old-fashioned and / (c) are 95. (a) The fever has taken / (b) a turn for / (c) the better from
very strictly with their children . / (d) No error yesterday. / (d) No error
73. (a) Last year there was flood, / (b) the river was
96. (a) I agree that / (b) nobody besides you / (c) could have
overflowing, / (c) and everyone run to save his life. / (d)
borne so much burden. / (d) No error
No error
97. (a) Beside Madan and Daman, / (b) there were five / (c)
74. (a) He was not only involved / (b) in her kidnapping / (c)
more boys present in the class. / (d) No error
but also in her murder. / (d) No error
98. (a) Rana Pratap, a true Rajput, (b) subsisted with grass /
75. (a) As he had taken / (b) only a few sips / (c) there was
(c) but did not give up his independence. / (d) No error
still little water left in the glass. / (d) No error

KUNDAN
99. (a) Divide the mango / (b) in four parts / (c) and distribute
76. (a) One of the members / (b) expressed doubt if / (c) the
them among your queens one each. / (d) No error
minister was an atheist. / (d) No error
100. (a) The old lady / (b) can hardly walk / (c) and is now
77. (a) In a report / (b) issued by Indian Statistical Institute,
confined within her house. /(d) No error
/ (c) the Iron and Steel Industry is spending more than
any other Indian industry on fighting pollution. / (d) No 101. (a) The car flew off the road / (b) and fell into the valley
error / (c) because Amit was driving faster. / (d) No error
78. (a) The retiring principal / (b) asked his old pupils / (c) to 102. (a) Anita was / (b) unhappy about the results / (c) of the
take the interest in the school after his retirement./ (d) discussion she has with Rajani. / (d) No error
No error 103. (a) Taking pity / (b) on the mouse, / (c) the sage
79. (a) The mission provides /(b) able service to all / (c) the transformed it into a cat. / (d) No error
needy people in this area during last few years. / (d) No 104. (a) You have not care / (b) to see that your / (c) expenditure
error was more than your salary. / (d) No error
80. (a) I missed the train / (b) which I usually catch / (c) and 105. (a) Jayesh had hardly / (b) started reading the magazine
had to travel on the next. / (d) No error / (c) than the managing director came to see him. / (d) No
81. (a) A hammer, some nails, / (b) and a few other tools / (c) error
was all that he used for mending the boat. /(d) No error 106. (a) Unless I do not get / (b) some tea / (c) I shall not be
82. (a) I am fed up / (b) of working from morning / (c) till night able to do any more work. / (d) No error
in this damp room. / (d) No error 107. (a) Both of my / (b) children, a daughter and a son, / (c)
83. (a) This book resembles / (b) to that / (c) and I often get always quarrel with one another. / (d) No error
confused about which one I am pulling out. / (d) No error 108. (a) Every man and woman / (b) should vote / (c) for the
84. (a) There are not / (b) some good restaurants / (c) in this candidate of their choice. / (d) No error
part of the town. / (d) No error 109. (a) He had consulted not only the opposition parties /
85. (a) Due to illness / (b) he was unable / (c) to go on a (b) but also accommodated their views / (c) while
holiday. /(d) No error dropping this controversial clause. / (d) No error
110. (a) The escape of David to France / (b) was successfully 128. (a) All history shows that nations active in international
arranged, but no sooner / (c) was this did than George politics / (b) are continously preparing, / (c) actively
was captured and held prisoner. /(d) No error involved in, or recovering from organised violence in
111. (a) People are just likely / (b) to be failures through / (c) the form of war. / (d) No error
a lack of jobs as through sheer incompetence. / (d) No 129. (a) We invited applications / (b) for sixteen posts / (c)
error but we have receive 20,000 applications. / (d) No error
112. (a) Nowadays several / (b) plastic products are superior 130. (a) It is the duty / (b) of every right-thinking citizen / (c)
/ (c) than similar products made from rubber. / (d) No to try to make the whole world a happier place to live.
error / (d) No error
113. (a) Sorry for the inconvenience / (b) caused by our 131. (a) Our laxity/ (b) in duty / (c) increases with our aversion
delayed delivery / (c) but the dispatcher is already for work. / (d) No error
reprimanded for this lapse./ (d) No error 132. (a) Though he lives in Tamil Nadu, / (b) he speaks / (c)
114. (a) The patient admitted yesterday / (b) did not responded not only Tamil but Telugu as well. / (d) No error
/ (c) to the treatment prescribed by the doctor for him. / 133. (a) It is / (b) difficult for / (c) anyone to past time thus.
(d) No error / (d) No error
115. (a) Both the alternatives /(b) have their merits and 134. (a) I will put on / (b) a note in this regard / (c) for your
demerits / (c) when it comes to their actual implementation. consideration and necessary decision. / (d) No error
/ (d) No error 135. (a) He wasn’t rich / (b) by any means, / (c) although he

K
116. (a) Luckily, the prisoner agreed / (b) to accompany the never turned away anyone who needed help./ (d) No
stranger and / (c) they slowly made their way down to error
the bottom of the cliff. / (d) No error 136. (a) Scarcely had I / (b) finished washing the car / (c) than
117. (a) As more and more subjects take the Rorschach test, the master came and asked me to clean the floor of the
/(b) the body of information tying styles of response house. / (d) No error
/(c) with specific problems or tendencies grows and the 137. (a) Your scissor / (b) are blunt; / (c) my razor is sharp. /
predictive power of the test increases. / (d) No error (d) No error
118. (a) It is a special feature of cell aggregation in the 138. (a) Mr Sharma has set up / (b) another school with a view
developing nervous system / (b) that in most regions of / (c) to compete against his rival. / (d) No error
the brain / (c) the cells not only adhere to one another / 139. (a) Until you / (b) remain restless / (c) you cannot

KUNDAN
and also adopt some preferential orientation. / (d) No concentrate. / (d) No error
error 140. (a) He can / (b) only be cured / (c) by a surgical operation.
119. (a) The residents’ opposition to the spraying programme / (d) No error
/ (b) has rekindled an old debate / (c) among those who 141. (a) Politicians who are wise / (b) profit from / (c) the
oppose the use of pesticides and those who feel that dissensions of the bureaucrats. / (d) No error
pesticides are necessary to save the trees. / (d) No error 142. (a) That Birbal was / (b) a contemporary with Akbar / (c)
is well-established from the Akbar-Birbal jokes. / (d) No
120. (a) He made it / (b) appear to the police / (c) as if his
error
condition is very serious. / (d) No error
143. (a) The cat appears / (b) to have / (c) originated from the
121. (a) Cannot one do / (b) what one / (c) likes to do with his
East. /(d) No error
own ? / (d) No error
144. (a) The solution / (b) does not hint / (c) the possibility of
122. (a) There’s Mr Shanu, / (b) whom they say / (c) is the
an alternative answer./ (d) No error
best singer in the country. / (d) No error
145. (a) As far as I think, / (b) the dress he wore / (c) was not
123. (a) The Renaissance is one of the most / (b) interesting /
appropriate to the occasion. /(d) No error
(c) period in the history of architecture and indeed, of art
146. (a) The ideas / (b) he propagates / (c) are subversive to
in general. / (d) No error
discipline. / (d) No error
124. (a) By this time next week / (b) my child not only / (c) will 147. (a) A mother, of whichever species / (b) she may be, / (c)
have learnt numbers but will also master the alphabet. can never lose affection to her son. / (d) No error
/ (d) No error 148. (a) Your capacity of / (b) facing difficult situations / (c)
125. (a) It gives me / (b) great pleasure / (c) in calling on Mr will be tested when you have to live on your own. / (d)
Smith to address you. / (d) No error No error
126. (a) My brother has gone / (b) to Madras; / (c) he will 149. (a) India can / (b) hardly stand / (c) any comparison to
return back in a week’s time. / (d) No error the US. / (d) No error
127. (a) A majority of the / (b) students / (c) believes that the 150. (a) She has an experience / (b) for three years / (c) and is
terminal examination is unnecessary. / (d) No error therefore eligible for the scholarship. /(d) No error
151. (a) Piyush was the one person / (b) who could somehow 169. (a) He has not undergone / (b) any technical training in
manage / (c) to working in that section for such a long the games / (c) but he plays good than most
time. / (d) No error professionals. /(d) No error
152. (a) He was driving slowly / (b) down the highway / (c) 170. (a) My sermon on the meaning of manna / (b) in the
when a cat ran across and he had to brake all of a sudden. wilderness / (c) can be adapted on / (d) almost any
/ (d) No error occasion, joyful or distressing./ (e) No error
153. (a) Watch how careful / (b) the sparrow knits / (c) the 171. (a) It is the first time / (b) in the history of India that / (c)
straws into one another to form a nest. / (d) No error people from all over India flocked to see a naked fakir. /
154. (a) Most of the third world / (b) country are experiencing (d) No error
/ (c) the ethnic or communal problems in various degrees. 172. (a) The money-minded people believe / (b) that it is
/ (d) No error foolish to exert themselves for such study / (c) and
155. (a) The manager asked the worker / (b) why was he / (c) brainwork which cannot be converted into cash. / (d) No
again disturbing the schedule of production./ (d) No error
error 173. (a) They talked to Ajit / (b) and tried to / (c) convince he
156. (a) It is necessary to check every passenger for security about their point of view. /(d) No error
/ (b) and do not load his luggage / (c) on to the aircraft 174. (a) The lovers had great fun / (b) and then walked / (c)
unless he identifies it. / (d) No error besides each other in silence. /(d) No error
157. (a) The Secretary and Treasurer / (b) were / (c) not present 175. (a) He won’t / (b) return the money / (c) that he borrowed,

K
at today’s meeting. /(d) No error will he ? / (d) No error
158. (a) He used /(b) very inaccurate language /(c) for he 176. (a) A large-scale exchange of / (b) nuclear weapons will
knew no better. / (d) No error produce / (c) unprecedented amounts of radiation that
159. (a) I cannot understand that / (b) how you can think/ (c) can penetrate into the biological tissue. / (d) No error
of going to Kashmir without woollen clothes. / (d) No 177. (a) Troy was taken by Greeks; / (b) this formed the basis
error / (c) of a story which has become famous./ (d) No error
160. (a) With a lot of persuasion / (b) Mrinal was able to get / 178. (a) I have had to work / (b) at the fountain for almost / (c)
(c) the professor to agree to review his article. / (d) No ten hours before it could start functioning well. / (d) No
error error
179. (a) If the teacher / (b) is good the students/ (c) will respond
161. (a) No slogan was so popular / (b) in the poorer population
positively to them. / (d) No error

KUNDAN
/ (c) as “Garibi Hatao”, which reflected the government
180. (a) The teaching staff had taken / (b) lectures regularly /
policies. / (d) No error
(c) had they known that the Principal would know about
162. (a) The programme is / (b) being telecasted / (c) from one
it. / (d) No error
of the Doordarshan Kendras./ (d) No error
181. (a) One learns / (b) to respect / (c) oneself in the army.
163. (a) Unlike the Second World War, when long voyages
/ (d) No error
home aboard troopships gave soldiers / (b) a chance to
182. (a) The Governor will preside / (b) at the meeting / (c) in
talk out their experiences / (c) and time to absorb them,
which a decision regarding the statehood of Uttarakhand
Vietnamese returnees often came home by jet, singly or
will be taken. / (d) No error
in small groups. /(d) No error
183. (a) You cannot / (b) believe / (c) his words because he is
164. (a) Since 1965 there are four times as many Black college
untruthful man. / (d) No error
students enrolled, / (b) and the one million Black people
184. (a) While trying / (b) to swim, / (c) the mouth must be
/ (c) in college today represent 11 per cent of all college
kept above water. / (d) No error
students. / (d) No error
185. (a) The captain as well as / (b) five other members / (c) of
165. (a) In cold-water habitats, certain invertebrates / (b) and the team were present. / (d) No error
fish convert starches into / (c) complex carbohydrates 186. (a) Please specify / (b) in your letter / (c) what kind of a
called glycerols, in effect manufacturing its own camera you want. / (d) No error
antifreeze. / (d) No error 187. (a) It was a / (b) major earthquake / (c) and the house’s
166. (a) The two first rows / (b) in the auditorium / (c) were roof fell down. / (d) No error
reserved for the press. / (d) No error 188. (a) When she / (b) could not / (c) answer his question, he
167. (a) Each of the students / (b) have done / (c) well in the called her as a fool. / (d) No error
internal examination. / (d) No error 189. (a) Ram as well as Shyam / (b) was invited / (c) but both
168. (a) Men, women and children — in a word, the whole men have not come. /(d) No error
population / (b) of the parish / (c) had turned out to 190. (a) Let you and I / (b) come to an agreement / (c) that we
welcome the new Vicar. /(d) No error will share the space adjoining the building. / (d) No error
191. (a) Being courteous, / (b) I accepted / (c) the invitation 196. (a) He became so infatuated / (b) of her charm / (c) that
for the dinner. /(d) No error he was ready to do anything for her sake. /(d) No error
192. (a) Is there any / (b) exemption of income tax / (c) if you 197. (a) He said that / (b) he was not / (c) liable to his wife’s
invest in mutual funds ? (d) No error debts. /(d) No error
193. (a) This is nothing new; / (b) we have become / (c) 198. (a) Are you so sure / (b) about your success / (c) that
accustomed with instances of corruption./ (d) No error you are not taking the examination again? / (d) No error
194. (a) Being accomplished with / (b) the art of dancing, / (c) 199. (a) It was evident from / (b) the umpire’s controversial
she must not let her talent go waste. / (d) No error decisions / (c) that he was partial with his own country.
195. (a) He pleaded for some more time / (b) to prove his / (d) No error
innocence / (c) but the court was deaf before all his 200. (a) I am/ (b) anxious of / (c) the outcome of this game.
pleas. / (d) No error / (d) No error

Answers
1. c 2. b 3. b 4. c 5. c 101. c 102. c 103. d 104. a 105. c
6. a 7. c 8. b 9. a 10. c 106. a 107. c 108. c 109. a 110. c

K
11. c 12. b 13. a 14. c 15. c 111. a 112. c 113. c 114. b 115. d
16. c 17. c 18. c 19. b 20. d 116. d 117. c 118. c 119. c 120. c
21. c 22. a 23. a 24. b 25. d 121. c 122. b 123. c 124. c 125. d
26. c 27. c 28. b 29. c 30. c 126. c 127. c 128. b 129. c 130. c
31. c 32. c 33. c 34. c 35. b 131. c 132. c 133. c 134. a 135. c
36. a 37. b 38. a 39. a 40. d 136. c 137. a 138. c 139. a 140. b
41. d 42. c 43. a 44. c 45. b 141. d 142. b 143. c 144. b 145. d
46. a 47. c 48. a 49. c 50. b 146. c 147. c 148. a 149. c 150. b
51. c 52. a 53. c 54. a 55. c 151. c 152. d 153. a 154. b 155. b
56. b 57. a 58. b 59. d 60. c 156. b 157. b 158. d 159. a 160. c
61. c 62. d 63. c 64. c 65. c 161. a 162. b 163. a 164. a 165. d

KUNDAN
66. c 67. d 68. c 69. c 70. a 166. a 167. b 168. d 169. c 170. c
71. b 72. c 73. c 74. a 75. c 171. a 172. c 173. c 174. c 175. d
76. b 77. a 78. c 79. a 80. c 176. d 177. a 178. a 179. c 180. a
81. c 82. d 83. b 84. b 85. a 181. c 182. d 183. c 184. c 185. c
86. a 87. c 88. c 89. c 90. a 186. c 187. c 188. c 189. c 190. a
91. a 92. b 93. c 94. b 95. c 191. c 192. b 193. c 194. a 195. c
96. b 97. a 98. b 99. b 100. c 196. b 197. c 198. b 199. c 200. b

Sentence Improvement
Directions: In each sentence below, a group of words 2. Anyone can now gather a vast amount of information if
has been given in bold. Below each, four choices (a), (b), (c), you have access to the internet.
and (d) are given. Pick out the one which can substitute the (a) they have access
bold group of words correctly, without changing the meaning (b) access can be available
of the sentence. (c) he or she has access
1. It was them who had left before I arrived. (d) one of them have access
(a) they who had left before I had arrived 3. She prepares tea, works on computer and then doing her
(b) them who had went before I arrived homework.
(c) them who had went before I had arrived (a) doing then her homework
(d) they who had left before I arrived (b) then is doing her homework
(c) doing her homework then (a) to not refrain from (b) to resort to
(d) then does her homework (c) to refrain from (d) to be refrained from
4. The grocer hadn’t hardly any of those kind of soap. 13. What happens to all those aboard the bus was not known.
(a) had hardly any of those kinds (a) What happened of
(b) hadn’t hardly any of those kinds (b) This is what happens to
(c) had hardly any of those kind (c) What is that happens to
(d) had scarcely any of that kind (d) What happened to
5. My teacher believed that I would get over my problems 14. Despite of their efforts to tarnish our campaign, we
and get the results I would be producing before. managed to emerge victorious.
(a) would have been producing (a) Despite their (b) Despite of the
(b) would have produced (c) Despite for their (d) Despite off their
(c) had been produced 15. The singer had been left the auditorium before the
(d) had been producing audience stood up.
6. If you are thinking of investing money, isn’t it makes (a) had been leaving (b) was left
sense to consult your father? (c) leaves (d) would leave
(a) is it not making (b) doesn’t it make 16. They came to the conclusion that the members should
(c) does it make (d) is it making unitedly undertook the assignments for the next month.
7. In addition to communicated his innocence through his (a) should be united undertook

K
speech, the minister attacked the opposition for its (b) should unitedly undertake
malicious campaign. (c) should be unitedly undertaken
(a) of communicating his innocence (d) should be unitedly undertake
(b) to having communicated his innocence 17. One of my drawbacks is that I do not have to tolerance of
(c) to communicating his innocence ambiguity.
(d) to have his innocence communicated (a) do not have (b) cannot have to
8. Technology must use to feed the forces of change. (c) am not (d) did not have to
(a) must be used to feed 18. They should have carefully calculated what they stand
(b) must have been using to feed to gain from the deal.
(c) must use having fed (a) should have been careful in calculating about
(d) must be using to feed (b) should be carefully calculated

KUNDAN
9. Unemployment has growth rapidly since the advent of (c) shall have to carefully calculate
computers in our country. (d) should have carefully calculate
(a) Rapid unemployment has grown 19. They are so poor that they struggle to make both ends to
(b) Unemployment has grown rapid meet.
(c) Unemployment grows rapidly (a) to making both ends meet
(d) Unemployment has been rapidly grown (b) to make both ends for meeting
10. The Senator rose up to say that, in his opinion, he thought (c) to make both ends meet
the WTO Bill should be passed on unanimously. (d) for making both ends to meet
(a) rose to say that he thought the WTO Bill should be 20. He never has and ever will take a decision against his
passed conscience.
(b) rose up to say that the WTO Bill should be passed (a) has taken nor will ever take
on (b) had taken and will ever take
(c) rose to say that, in his opinion, he thought that the (c) has and never will take
WTO Bill should be passed (d) had and ever will take
(d) rose to say that, in his opinion, the WTO Bill should 21. Law-breakers should severely punished when they are
be passed on found out.
11. The courts are actively to safeguard the interests of the (a) should be severe punished
commuters. (b) should have severely punished
(a) are actively to safeguarding (c) shall have severe punishment
(b) have been actively safeguarding (d) should be severely punished
(c) have to active in safeguarding 22. They examined the twins very carefully but failed to
(d) are actively in safeguarding detect any difference in them.
12. Because of his poor memory, the teacher has advised (a) some difference in
him not to refrain from cramming. (b) some difference between
(c) any difference between 30. Bacon believes that the medical profession should be
(d) any difference among permitted to ease and quicken death where the end would
23. Mr Pant, the defence minister and who is also the otherwise only delay for a few days and at the cost of
secretary of the party, has been chosen as the great pain.
representative for the talks. (a) be delayed for a few days
(a) since he is the secretary of the party (b) be delayed for a few days and
(b) also being the secretary of the party (c) be otherwise only delayed for a few days and
(c) secretary of the party (d) otherwise only delayed for a few days
(d) in addition, who is the secretary of the party 31. Though his actions were severe criticised, he didn’t
24. He marvelled at Shah Jahan’s love for architecture and lose his temper.
admiring the monuments built by him. (a) were severe criticism
(a) admire the monuments being built (b) were being of severe criticism
(b) admired the monuments built (c) had severe criticise
(c) admire the building of monuments (d) were severely criticised
(d) admired building the monuments 32. It was he, not me, who decided to postpone the programm
25. Since the advent of internet trading at the beginning of (a) he, not I
this decade, the capital markets took a giant stride (b) him, not me
forward in our country. (c) he, not mine

K
(a) this decade saw the capital markets taking (d) him, not I
(b) this decade, the capital markets have taken 33. This candidate has none of the characteristics possess
(c) this decade, the capital markets had taken by the deserving ones.
(d) this decade, the capital markets will take (a) from the characteristics possessed by
26. Only people who are good at these kind of activities will (b) with the characteristics possess by
be considered for the appointment. (c) of the characteristics possess with
(a) these kind of activity (d) of the characteristics possessed by
(b) this kinds of activity 34. Those who are indecisive can be readily persuading to
(c) these kinds of activities change their mind again.
(d) the kind activities like these (a) can be ready persuading

KUNDAN
27. If you are on a three-monthly software design project (b) can be readily persuaded
and you have put together a program that solves part of (c) could be ready to persuade
the problem in two weeks, show it to your boss without (d) can be readily persuasive
delay. 35. We should realise that the chance of accident may be as
(a) and you have put together a program that has solved great or greater in our homes than in a train or on the
part of the problem in two weeks road.
(b) and, in two weeks, you have put together a program (a) may be at least as great in our homes as
that solves part of the problem (b) was at least as great as in homes
(c) and, in two weeks, you put together a program that (c) can be as great or greater in our homes than
solved only part of the problem (d) may be at least as great or greater in our homes than
(d) and you put together a program that solves part of 36. In 1994 the batsman scored thrice as many runs against
the problem in two weeks. Australia as that had in 1993.
28. Some believed Hrithik’s acting is no different from the (a) than he had (b) than he did
other actors. (c) as he had (d) as he did
(a) not different from 37. The possibility of cyclones are regarded by most Orissa
(b) not different than the residents with fear and desperation.
(c) no different from that of the (a) are regarded by most of the residents of Orissa as
(d) not differing from the (b) is regarded by most Orissa residents with
29. If you would have completed the assignment, I would (c) is regarded by most of the residents of Orissa with
have granted you leave. (d) is regarded by most Orissa residents as
(a) Had you been 38. Primitive man believing that a crime created an imbalance
(b) If you would be which could be rectified only by punishing the
(c) If you had wrongdoer.
(d) If you would have been (a) believes that a crime created
(b) believed that a crime created
(c) believed that a criminal created (b) similarly as those that inspired belief
(d) believed that a crime creates (c) similarly as those that had inspired belief
39. Everybody depend upon him scoring runs every time (d) similar to those that inspire belief
India goes to bat. 48. On seeing him breathe with difficulty, I asked him since
(a) depends upon him scoring how long he had been ailing.
(b) depend upon him score (a) how long he had been ailing for
(c) depends upon his scoring (b) how long he had been ailing
(d) depending upon his scoring (c) since how long he was ailing
40. The floods that came last year was devastating more (d) since when he ailed
than the ones this year. 49. They asked him to keep studying till they had returned.
(a) were more devastating (a) until they had returned
(b) was devastated more (b) till their returning
(c) were devastating more (c) till they return
(d) were more devastated (d) till they returned
41. Most of the boys and girls like to see these kind of 50. Of the two brothers, I think the oldest one is the taller.
movies. (a) the eldest one is the taller
(a) this kind of movies (b) the elder one is the taller
(b) these kind of movie (c) the older one is the taller

K
(c) these kinds of movies (d) the oldest one is the tallest
(d) this kind of movie 51. If you have power or money you misuse and if you don’t
42. Because of the changing times, we expect to study more have it you yearn for it.
than our teachers have. (a) you misuse it when you don’t
(a) should expect to study more than our teachers (b) you misuse it and if you don’t
(b) are expected to study as much as our teachers study (c) you misuse so that you don’t
(c) are expected to study more than our teachers did (d) you misuse and if you can’t
(d) expected to study more than our teachers 52. With the number of vehicles on the roads going up fast,
43. It is difficult to prevent assault against environment as the traffic in the metros have become heavy and
those that are against animals. uncontrollable.
(a) those against (a) has become heavily

KUNDAN
(b) it is against (b) had become heavy
(c) preventing those against (c) have become heavily
(d) it is to prevent those against (d) has become heavy
44. There is no need of being anxious about him; he knows 53. The tehelka.com revelations and their aftermath
his target and will certainly achieve it. have shaked up entrenched political-bureaucratic
(a) for being anxious of him networks in New Delhi.
(b) to be anxious about him (a) its aftermath has shaken up
(c) for being anxious about him (b) their aftermaths have shaked up
(d) to feeling his anxiety (c) its aftermath has shook
45. The success of a captain depends upon the tact in which (d) their aftermath have shaken up
he is able to get the best out of his team members. 54. The editor cut up all the long words from his article and
(a) that makes him enabled made the text easier.
(b) in which he enables (a) cut up some of the
(c) with which he is able (b) cut across all the
(d) in which he has been able (c) cut out all the
46. Soon after the first show I returned to my quarter, where (d) cut back all
I was surprised to find my uncle waiting for me. 55. The man is mortal and ultimately he has to leave
(a) to my quarter, where I everything here, yet how irrationally he remains attached
(b) to my quarter in which I was to his worldly possessions.
(c) to my quarter, where I had been (a) The man is immortal
(d) to my quarters, where I was (b) Man is considered to be mortal
47. Scientific medicine had at first to combat superstitions (c) Man is mortal
similar to those that had inspired belief in witchcraft. (d) Though the man is mortal
(a) like the ones that inspired belief
56. I already thrice have read this story. 66. When I receive the letter, the date for the interview was
(a) I have already read this story thrice. already over.
(b) I thrice have already read this story. (a) I received the letter
(c) I have thrice already read this story. (b) the letter I had received
(d) I have already read thrice this story. (c) I had to receive letter
57. All of the candidates except Manindra and I intends on (d) I was receiving the letter
choosing cricket as hobby. 67. Facing with the prospect that the RBI withholds approval
(a) me intends on choosing for the UTI-GTB merger, GTB has played safe by walking
(b) me intends to choose out of the merger.
(c) me intend to choose (a) Faced with the prospect of the RBI withholding
(d) I intend to choose (b) Facing the prospect that the RBI withholds
58. He has requested for an interview of the principal this (c) In the face of the prospect of the RBI withholding
morning. (d) On the face of it, the RBI withholds
(a) an interview by the principal 68. The chairman approved the recommendations to the
(b) an interview with the principal committee with partial modifications.
(c) the interview of the principal (a) by the committee for
(d) an interview from the principal (b) of the committee with
59. Can you tell me where does your uncle live? (c) to the committee by

K
(a) where your uncle does live (d) at the committee in
(b) whether your uncle does live 69. Enzymes help develop a person’s appetite, but it is
(c) where your uncle lives unharmful to their bodies.
(d) where lives your uncle (a) and it is not harmful to their bodies
60. This milk has got a funny taste — it can have gone sour. (b) and it is not harmful to the body
(a) can go sour (b) had soured (c) but they are not harmful to the body
(c) can have gone sourly (d) may have gone sour (d) and they are not harmful to the body
61. All the newcomers enjoyed at the welcome party. 70. Having tired of play he sat down to rest.
(a) enjoyed in (a) Having tired with (b) Having tired by
(b) themselves enjoyed at (c) Being tired of (d) Having been tiring by

KUNDAN
(c) enjoyed themselves at 71. I have never been hearing from him since he left this city.
(d) enjoyed thoroughly at (a) was never heard
62. Most of the politicians in the town, they supported the (b) have never been heard
striking employees of the factory. (c) had never been heard
(a) Most of the politicians in the town supported (d) have never heard
(b) Most of the politicians in the town provided support 72. If you have examined the proposals, you must have found
for that his scheme is preferable than that of anyone else.
(c) The politicians in the town, most of them supported (a) preferable than for
(d) Of the politicians in the town, mostly supported (b) preferably more than that of
63. The Bill on the Right to Information is being examined (c) preferable for than
by a committee; is it? (d) preferable to that of
(a) hasn’t it (b) isn’t it 73. The terrorists were not in a yielding mood but ultimately
(c) won’t it (d) doesn’t it they were given in.
64. He is afraid that his past will be a deterrent when he will (a) were given up (b) gave in
seek fresh admission to the college. (c) had been given up (d) had been given in
(a) when he seek fresh admission 74. Finding fault with others is the business of those who
(b) when he seeks fresh admission are nothing else to do.
(c) when he shall seek fresh admission (a) are nothing to do elsewhere
(d) when he should seek fresh admission (b) have nothing else to do
65. He went to the flooded area to save his friend who was (c) do not have nothing else to do
stranding there. (d) had nothing to be done
(a) had been stranded 75. Kailash Satyarthi has championed the cause from child
(b) had to strand labourers for the last several years.
(c) was being stranding (a) has been championing the cause for
(d) has been stranding (b) has been championing the cause of
(c) had been championed the cause for (a) an out-of-court settlement seems ruled out
(d) had championed the cause for (b) an out-of-curt settlement seems to has been ruled
76. Our business firms were full aware of the Y2K crises. out
(a) were fully aware into (c) an out-of-court settlement seem ruled out
(b) have been full aware of (d) out-of-court settlement seems to have ruled out
(c) were fully aware of 86. When I managed to convince him that I want to get home
(d) had been fully aware at quickly, he agreed to leave me early.
77. To finish a long journey, persistent is needed. (a) I want to get home quickly
(a) persistence is needful (b) that I want to get to home quickly
(b) persistence is what one needs (c) that I wanted to get home quick
(c) persistent is what one needs (d) that I wanted to get home quickly
(d) one needs to be persisted 87. Robin Singh is oldest than the other player in the Indian
78. Moby Dick is the story of a man’s battle with a white cricket team.
whale and the story would take place mainly in the south (a) older than any other
seas. (b) oldest than other
(a) the story has taken place mainly (c) the oldest among the
(b) the story took place primarily (d) elder than the other
(c) the story is set to take place mainly 88. Even after requesting him, Sachin did not tell us how he

K
(d) the story takes place mainly bats so well.
79. Holding the fatal telegram in her closed fist, the distraught (a) Even on requesting him
mother fell back on the couch. (b) Even we had requested him
(a) fell out with (b) fell back (c) Though after requesting him
(c) fell back for (d) fell on (d) Even after being requested
80. The quality of coal mined from State X is less than State 89. Cutting small savings rates is conducive for the growth
Y. of the economy.
(a) is less than that from (a) To cut small savings rates is conducive for
(b) is lower than (b) Cutting small savings rates is conducive to
(c) is lower than that from (c) Cutting small savings rate is conducive for
(d) is lower than that of (d) Cutting small savings rates are conducive to

KUNDAN
81. He likes living in small towns rather than crowded cities. 90. It is definitely kind of this attitude that is required in this
(a) rather than that in crowded cities day and age.
(b) rather than in crowded cities (a) one of this kind of attitude
(c) rather crowded cities (b) this kind of attitude
(d) better than crowded cities (c) one of this kinds of attitude
82. Last fifteen days he has not been attending classes and (d) kind of these attitudes
he cannot make up for it now. 91. The manager got very angry and order immediate
(a) In the last fifteen days dismissal of the employee who had committed the
(b) Fifteen last days mistake.
(c) Of the last fifteen days (a) order immediately (b) ordered immediately
(d) For the last fifteen days (c) ordered immediate (d) ordering immediate
83. Tansen is occupying an important position in the world 92. I have told the sculptor to put on all the best in the statue
of music for many generations. which he was carving.
(a) occupies (a) to put in (b) to put up
(b) has occupied (c) to put down (d) to put
(c) has been occupying 93. The finance ministry has initiated talks with the law and
(d) is being occupied power ministries on an arbitrator being appointed.
84. He is not in a position to use the digital diary given by (a) for an arbitrator being appointed
you to him. (b) on the appointment of an arbitrator
(a) given to him (b) given by you (c) to appoint an arbitrator
(c) you gave to him (d) given by him to you (d) so that an arbitrator be appointed
85. The Centre has decided to take Enron’s arbitration notice 94. You should apologise for your inability to attending the
head-on and chances of out-of-court settlement have wedding.
seem ruled out. (a) inability about attending
(b) being inability to attending (c) keep the colony clean
(c) being unable to attend (d) for clean and keep the colony
(d) unability to attend 104. Any one who known to India’s metros knows the
95. Unless you show me how to do it, I should not be able to meaning of pollution.
solve the problem. (a) knowing India’s metros knows
(a) shall not be able (b) is known to India’s metros knows
(b) should not be unable (c) knows India’s metros knows
(c) shall not be unable (d) knew India’s metros knowing
(d) should be unable 105. Shook by the challenge thrown by Reliance Infocom,
96. Nothing is so easy than operating the new-generation BSNL is planning to give its customers a better deal.
machines; they are user-friendly in true sense. (a) Shaken by the challenge thrown by Reliance Infocom
(a) Nothing is easier as (b) Since Reliance Infocom shook it by throwing a
(b) Nothing is so easy as challenge
(c) Nothing is so easier than (c) Thrown away by the shaking challenge of Reliance
(d) Nothing is more easier than Infocom
97. Despite being tried his best to win the minority votes, (d) Shook up by the challenge thrown by Reliance
he could not be successful. Infocom
(a) Despite his best trying 106. If Indian people are united, this nation could become a

K
(b) Despite of his best trial source with strength for the entire world.
(c) In spite of being tried his best (a) can become a source with
(d) Despite trying his best (b) could become a resource of
98. His suggestions were so trivial and hence nobody paid (c) can become resourceful for
any attention to them. (d) can become a source of
(a) so trivial that and hence 107. All the efforts were direct to promote harmony among
(b) very trivial and hence so various groups of people.
(c) too trivial to and hence (a) were directing to promote
(d) very trivial and hence (b) were directed at promote
99. But for your time of helping, the mission would still (c) were directed to promote

KUNDAN
have remained incomplete. (d) were directed to promoting
(a) But for your timely help 108. What did happen there is not something that agitates
(b) Because of your timely helping me.
(c) Despite your time of helping (a) What does happen
(d) But your time for helping (b) What would have happened
100. He failed in his attempt to disperse the mob before the (c) What happened
miscreants sets the fire on the bus. (d) What should have happened
(a) set the bus on fire 109. We cannot ignore the contribution of Kapil Dev for
(b) setting fire on the bus bringing about change in our bowling pattern.
(c) set fire on the bus (a) for bringing in
(d) set the fire on to the bus (b) in bringing about
101. The patient was saved from the jaws of death because of (c) in bringing of
the doctor arrived on time. (d) for he brought over
(a) of the doctor being arrived 110. Such loopholes can be founded whenever similar laws
(b) of the arrival of the doctor exist.
(c) the doctor arrived (a) could have been founded
(d) of the doctor had arrived (b) can be found
102. How can you take the support of your voters for granted (c) could be founded
without being loyally to them? (d) could have found
(a) being loyally for (b) been loyally to 111. The Supreme Court was never bothering to care about
(c) being loyal to (d) loyally being to the sufferings of the masses.
103. Every resident must help the Residents’ Welfare (a) was never bothered to
Association for keep up the colony clean. (b) never bothered to
(a) for upkeep clean the colony (c) never was bothering to
(b) for keeping the colony cleanliness (d) was never bothering to
112. India’s outlook on the world is consisting of these (b) had been known
various elements. (c) knew
(a) consists of (b) is consisted of (d) was known
(c) is consisted by (d) has been consisting 123. The presence of politicians in celebrating the life of a
113. That match-fixing is a universal phenomenon is evident spiritual conqueror is an anomaly because few among
from events that taking place around the globe. them rise above the formality of making speeches and
(a) taken place over (b) taking place in transforming words into action.
(c) that take place on (d) that take place around (a) but transforming words into action
114. The exploitation of employees in a factory needs be (b) and transform action from words
condemned. (c) and transform words into action
(a) needs to be condemned (d) that transform words into action
(b) needs condemned 124. The construction of great temples which had seemingly
(c) needs condemnation have presented great engineering difficulties, was
(d) need be condemned relatively easy.
115. The fact-finding team which had been at the area found (a) had seemingly been presented
villagers giving information to the police. (b) had been seemed present
(a) which had been for (c) was seemingly presented
(b) that led to (d) would seemingly have presented

K
(c) which led to 125. Finishing his homework, he went out to play.
(d) that had been to (a) His homework finished
116. The new facts he had discovered has not changed my (b) His homework having finished
opinion. (c) Having finished his homework
(a) could not change (b) could not be changed (d) Finished his homework
(c) has not been changed (d) cannot be changed 126. One of the characteristic of an officer is to make decision
117. On firm ground the tent had held in place by pegs driven in the most demanding of the situations.
into the ground. (a) One of the characteristics of
(a) has been held at (b) was held in (b) Most of the characteristics of
(c) was being held with (d) should be held at (c) Some of the characteristics of
118. Inaugurating the 2600th birth year celebrations of (d) One of the characteristics by

KUNDAN
Bhagwan Mahavir, the prime minister has well done to 127. Whenever we went to the canteen, it was mandatory for
restate the fundamental tenets of ahimsa and each of us to pay our own bills.
intolerance. (a) to pay my own bill
(a) has well done in restating (b) paying our own bill
(b) has done well to restate (c) are paying our own bills
(c) has done well in restating (d) to pay his own bill
(d) has done good in restating 128. Even if a work is tedious, I prefer doing it myself than
119. He has still studied the matter and is still collecting related depending upon another person.
reports. (a) than depending upon the other
(a) is still studying (b) has studied still (b) than depend upon another
(c) still has studied (d) will have still studied (c) to depending upon another
120. I overheard him saying something to me when I was (d) rather than depending upon another
quit. 129. Being a demanding exam, he had to put in much more
(a) was almost quit (b) was about to quit time than in other exams.
(c) had about to quit (d) had been quitting (a) With a demanding exam
121. They have not only troubled us with their erratic (b) It being a demanding exam
behaviour but also with unnecessary disturbances. (c) Being a demanded exam
(a) not only troubled us by (d) As a demanding exam
(b) troubled us not only with 130. We are happy to recommend that his son to be considered
(c) troubled not only to us with for the post.
(d) not only been troubled us by (a) considers for
122. It was strange that no one was knowing who the culprit (b) be considered with
was. (c) be considered for
(a) had been knowing (d) may consider for
131. Plummeting stock markets have discouraged investors 141. The plight of the Gujarat earthquake victims must be
to make their own investment decision. seen to believe.
(a) from making their own investment decision (a) have been seen to believe
(b) to make their own investment decisions (b) have been seen for believing
(c) of making their own investment decisions (c) have seen for belief
(d) from making their own investment decisions (d) be seen to be believed
132. No sooner he ordered for food than the waiter was ready 142. The truth is that the UGC with the help of certain
with it. handpicked educationists are offering cash incentives
(a) No sooner had he order to cash-strapped universities to start these courses.
(b) He ordered no sooner (a) who are offering cash incentives to
(c) He no sooner having ordered (b) is offering cash incentives for
(d) No sooner did he order (c) is offering cash incentives to
133. I am working on this book since last January. (d) are offering incentives in cash to
(a) was working 143. The Election Commission announced that elections to
(b) have been worked the Assemblies will be held on May 10.
(c) being worked (a) should be held from (b) will be held from
(d) have been working (c) would be held on (d) are being held on
134. The modifications made by them in the draft were so 144. He was quite sure that none of them were aware of the

K
drastic that the entire emphasis had been shifted. truth.
(a) shall have been shifted (a) were aware from (b) was aware of
(b) had shifted (c) were beware of (d) had aware of
(c) was being shifted 145. Vedic astrology is not only one of the main subjects of
(d) had been shifting our traditional and classical knowledge, but this is the
135. Many of our top managers today feel so frustrated and discipline which let us knowing the events happening
powerless as the rest of us. in human life and in the universe on time scale.
(a) so frustrating and powerless as (a) this is the discipline which lets us know
(b) quite frustrating and powerless as (b) this is the discipline that let us know
(c) so frustrated and powerless like (c) this being the discipline whom let us know

KUNDAN
(d) as frustrated and powerless as (d) this is the discipline that lets us to knowing
136. Despite his best efforts, the prime minister could not 146. Shocked of seeing corrupt persons partying in his house,
convince the rebels by changing their decision. the honest minister committed suicide.
(a) and changed their (b) to change their (a) Shockingly seen (b) Shocked at seeing
(c) with changing their (d) in changing his (c) Shocked by seeing (d) Seeing as a shock
137. The sight of the accident was so frightened that the 147. No sooner did he finish writing the article than his friend
bystanders could not utter a single word. had arrived there.
(a) so very frightening because (a) had arrived thereat (b) had had his arrival there
(b) so frightening that (c) had been arrived there (d) arrived there
(c) extremely frightening as 148. When your father inquired about your marks, you lied to
(d) extremely frightened him; have you not?
138. He travelled by bus but would have travelled by car to (a) haven’t you (b) couldn’t you
save time. (c) have you (d) didn’t you
(a) must have travelled (b) should be travelling 149. As soon as I will get my salary, I will repay your loan.
(c) could be travelling (d) should have travelled (a) The sooner I will get
139. The passing-out parade was held yesterday as they were (b) As soon as I get
for years. (c) No sooner than I get
(a) as they have been for (b) so were they for (d) As soon as I shall get
(c) as they are for (d) as they were before 150. Before the advent of the internet, television has been the
140. By the time he had won his commission, Mohan had to major source of entertainment.
start seeking employment elsewhere. (a) television was the major source of entertainment
(a) had started seeking (b) television has been the major source for
(b) were started seeking entertainments
(c) had been started to seek (c) the major source of entertainment is the television
(d) were to have started seeking (d) television was there a major source from
entertainment
Answers
1. d 2. c 3. d 4. a 5. d 76. c 77. b 78. d 79. d 80. c
6. b 7. c 8. a 9. b 10. a 81. b 82. d 83. b 84. b 85. c
11. b 12. c 13. d 14. a 15. d 86. d 87. a 88. d 89. b 90. b
16. b 17. a 18. c 19. c 20. a 91. c 92. a 93. b 94. c 95. a
21. d 22. c 23. c 24. b 25. b 96. b 97. d 98. d 99. a 100. a
26. c 27. b 28. c 29. c 30. c 101. c 102. c 103. c 104. a 105. a
31. d 32. a 33. d 34. b 35. a 106. d 107. d 108. c 109. b 110. b
36. c 37. c 38. b 39. c 40. a 111. b 112. a 113. d 114. a 115. d
41. d 42. c 43. b 44. b 45. c 116. a 117. b 118. b 119. a 120. b
46. d 47. a 48. b 49. d 50. b 121. b 122. c 123. c 124. d 125. c
51. b 52. d 53. d 54. c 55. c 126. a 127. d 128. c 129. b 130. c
56. a 57. c 58. b 59. c 60. d 131. d 132. d 133. d 134. b 135. d
61. c 62. a 63. b 64. b 65. a 136. b 137. b 138. d 139. a 140. a
66. a 67. a 68. b 69. d 70. c 141. d 142. c 143. c 144. b 145. a
71. d 72. d 73. b 74. b 75. b 146. b 147. d 148. d 149. b 150. a

K
KUNDAN

Vous aimerez peut-être aussi